You are on page 1of 150

ACHIEVE IAS MCQS SERIES - A 41 DAY PLANT TO COVER

HISTORY NCERTS
www.achieveias.co.in, YouTube Channel: http://youtube.com/c/AchieveIAS Telegram Channel:
http://t.me/Achieve_Ias, Mail: achieveias21@gmail.com, Contact Number: 8968920720
https://upscpdf.com
Day https://upscpdf.com
NCERT https://upscpdf.com
Chapters to be Covered
1. 6th Class Chapter 1 to 4
2. 6th Class Chapter 5 to 8
3. 6th Class Chapter 9 and 10
4. 6th Class Chapter 11 and 12
5. 7th Class Chapter 1 to 3
6. 7th Class Chapter 4
7. 7th Class Chapter 5
8. 7th Class Chapter 6 and 7
9. 7th Class Chapter 8
10. 7th Class Chapter 9 and 10
Day NCERT Chapters to be Covered
11. 8th Class Chapter 1 and 2
12. 8th Class Chapter 3
13. 8th Class Chapter 4
14. 8th Class Chapter 5 and 6
15. 8th Class Chapter 7 and 8
16. 8th Class Chapter 9 and 10
17. 8th Class Chapter 11 and 12
18. 9th Class All Chapters
19. 10th Class Chapter 3
20. 10th Class Chapter 4-6
Day NCERT Chapters to be Covered

Google it:- https://upscpdf.com


ACHIEVE IAS MCQS SERIES - A 41 DAY PLANT TO COVER
HISTORY NCERTS
www.achieveias.co.in, YouTube Channel: http://youtube.com/c/AchieveIAS Telegram Channel:
http://t.me/Achieve_Ias, Mail: achieveias21@gmail.com, Contact Number: 8968920720
21.
https://upscpdf.com 10https://upscpdf.com
th Class Chapter https://upscpdf.com
7 and 8
22. 12th Class Theme 7, Chapter 3
23. 12th Class Theme 8, Chapter 4
24. 12th Class Theme 8, Chapter 4
25. 12th Class Theme 9, Chapter 5
26. 12th Class Theme 10, Chapter 1
27. 12th Class Theme 10, Chapter 1
28. 12th Class Theme 11, Chapter 2
29. 12th Class Theme 11 and 12, Chapter
2 and 3
30. 12th Class Theme 12, Chapter 3
31. 12th Class Theme 13, Chapter 3
32. 12th Theme 13
33. 12th Theme 13
34. 12th Theme 14
35. 12th Theme 15
36. 12th Theme 1
37. 12th Theme 2
38. 12th Theme 3
39. 12th Theme 4
40. 12th Theme 5
41 12th Theme 6

Google it:- https://upscpdf.com


www.achieveias.co.in, YouTube Channel: http://youtube.com/c/AchieveIAS Telegram Channel: http://t.me/Achieve_Ias,
Mail: achieveias21@gmail.com, Contact Number: 8968920720
https://upscpdf.com https://upscpdf.com https://upscpdf.com
ACHIEVE IAS HISTORY NCERT MCQ SERIES, DAY 1, SOLUTIONS

Q1: With reference to the history of India, consider the following statement:
1. Wheat and Barley were the first crop to be grown.
2. Garo hills and banks of river Kaveri were some of the areas where agriculture developed.

Select the correct answer using the codes given below:


(a) 1 only (b) 2 only (c) Both 1 and 2 (d) Neither 1 Nor 2

Explanation: Answer is (a)

Wheat and Barley were the first crop to be grown in the region of Sulaiman and Kirthar hills to the
northwest of India about 8000 years ago.People also began rearing animals like sheep, goat, and
cattle, and lived in villages there. The Garo hills to the north-east and the Vindhyas in central India
were some of the areas where agriculture developed. There is no such reference for river Kaveri.
The places where rice was first grown are to the north of the Vindhyas.

Q2.Consider the following statement:


1. Traces of ash have been found around Kurnool caves.
2. An inscribed stone was found in Rosetta,a town on the north coast of Egypt.
3. Tools made from limestone were found in caves of France.

Select the correct answer using the codes given below:


(a) 1 only (b) 1 and 3 only (c) 1 and 2 only (d) 1, 2 and 3

Explanation: Answer is (c)

In Kurnool caves of Andhara Pradesh, traces of ash have been found, suggesting that people were
familiar with the use of fire. Rosetta is a town on the north coast of Egypt, and here an inscribed stone
was found, which contained inscriptions in three different languages and scripts (Greek, and two
forms of Egyptian). Tools made from limestone were found in Hunsgi, Karnataka. A number of early
Palaeolithic sites were found here.

Q3: which of the following is not a Palaeothic site?

(a) Bhimbetka caves (b) Hunsgi (c) Mehrgarh (d) Kurnool

Explanation: Answer is (c)

Bhimbetka caves, Hunsgi, Kurnool are Palaeolithic sites while Mehrgarh is a Neolithic site located in
present day Pakistan.

Google it:- https://upscpdf.com


www.achieveias.co.in, YouTube Channel: http://youtube.com/c/AchieveIAS Telegram Channel: http://t.me/Achieve_Ias,
Mail: achieveias21@gmail.com, Contact Number: 8968920720
https://upscpdf.com https://upscpdf.com https://upscpdf.com

Q4:Consider the following pairs:


Grains and bones Site
1. Wheat, barley, sheep, Mehrgarh (in present day-
goat, cattle. Pakistan)

2. Millet, cattle, sheep, Chirand (in present-day


goat, pig. Bihar)
3. Wheat and lentil. Gufkral (in present-day
Kashmir)
4. Rice, fragmentary Koldihwa (in present-day
animal bone. Uttar Pradesh)

Which of the pairs given above is/are correctly matched?


(a) 1, 2 and 3 (b) 1, 3 and 4 (c) 2, 3 and 4 (d) 1, 2 ,3 and 4

Explanation: Answer is (b)

Archaeologists have found traces of Wheat, barley, sheep, goat,Cattle in Mehrgarh (in present day-
Pakistan).Wheat and lentil in Gufkral (in present-day Kashmir).Rice, fragmentary and animal bone in
Koldihwa (in present-day Uttar Pradesh). However Millet and remains of cattle, sheep, goat, pig were
found in Hallur (in present-day Andhra Pradesh)and in Chirand wheat, green gram, barley
buffalo,ox were found. Several burial sites have been found at Mehrgarh. In one instance, the dead
person was buried with goats, which were probably meant to serve as food in the next world.

Q5: Traces of mortars and pestles have been found in:

(a)Mehrgarh (b)Burzahom (c)Daojali Hading (d) Gufkral

Explanation: Answer is (c)


Daojali Hading is an important Neolithic site in Dima Hasao District of Assam, India on the hills near
Brahmaputra valley close to routes leading into China and Myanmar. Extensive digging at this site
has yielded polished stone tools, ceramics and kitchen items such as grinders, pestles and mortars.

Q6: which metal was unknown to Indus Valley Civilisation?

(a) Iron (b) Copper (c) Gold (d) Silver

Explanation: Answer is (a)

The Indus valley people knew the use of Copper, Bronze, Silver, Gold but not iron. Copper and

Google it:- https://upscpdf.com


www.achieveias.co.in, YouTube Channel: http://youtube.com/c/AchieveIAS Telegram Channel: http://t.me/Achieve_Ias,
Mail: achieveias21@gmail.com, Contact Number: 8968920720
https://upscpdf.com https://upscpdf.com https://upscpdf.com
bronze were used to make tools, weapons, ornaments and vessels.While Gold and silver were used
to make ornaments and vessels. Iron was discovered in Late Harappan (Cemetery H) culture.

Q7: Which one of the following statement is not correct about Indus Valley Civilization:
(a) Evidence of some form of irrigation have been found.
(b) Embroidered garments were unknown in Indus civilization.
(c) Many of the cities had covered drains.
(d) Mohenjodaro,Harappa, and Lothal had store houses.

Explanation: Answer is (b)

A stone statue of an important man found from Mohenjodaro shows him wearing an embroidered
garment. This gives evidence about use of embroidered garments. Evidence of some form of
irrigation have been found in Dholavira, Gujrat Houses have been found with a separate bathing area,
and some had wells to supply water. And one of the notable feature is that many of these cities had
covered drains. Also remains of store houses have been found in Mohenjodaro,Harappa, and Lothal.
A platform was built in towns where goods were checked and stored.

Q8: In the context of indus valley civilization , which of the following statement is correct
about faience?

(a) Faience were an artificially produced material and were used to make beads, bangles, earrings,
and tiny vessels
(b) Faience is a naturally found mineral used in making vessels.
(c) Faience was a kind of stone tool used in pot making.
(d) Faience were stone weights found in Lothal.

Explanation: Answer is (A)


Faience were artificially produced material made by melting quartz rock and then regrinding the
glassy frit to make a paste that is then fired once again.These were used in makeing beads, bangles,
earrings, and tiny vessels. Use of faience was also common in Egyption civilization.

Q9: Which of the finding is common for ancient cities Kalibangan and Lothal?

(a) Dockyard (b) Bead making factory (c) Fire altars (d) Ploughed field

Explanation: Answer is (c)

Dockyard, bead making factory along with sea trade centre were found in Lothal, Gujrat.
Fire altars were found in both Kalibangan and Lothal showing practice of cult of sacrifice.
Ploughed field were found in Kalibangan, Rajasthan.

Google it:- https://upscpdf.com


www.achieveias.co.in, YouTube Channel: http://youtube.com/c/AchieveIAS Telegram Channel: http://t.me/Achieve_Ias,
Mail: achieveias21@gmail.com, Contact Number: 8968920720
https://upscpdf.com https://upscpdf.com https://upscpdf.com
Q10: Match the following:

Column A Column B
1. GOLD A. Rajasthan
2. TIN B. Afghanistan
3. COPPER C. Karnataka
4. PRECIOUS STONE D. Gujarat

(a) 1.C 2.B 3.D 4. A


(b) 1.B 2.C 3.D 4.A
(c) 1. C 2.B 3.A 4.D
(d) 1 .A 2.B 3.D 4.A

Explanation: Answer is (c)

The Harappans probably got copper from present-day Rajasthan, and even from Oman in West Asia.
Tin, which was mixed with copper to produce bronze, may have been brought from present-day
Afghanistan and Iran. Gold could have come all the way from present-day Karnataka, and
Precious stones from present-day Gujarat, Iran and Afghanistan.

Google it:- https://upscpdf.com


www.achieveias.co.in, YouTube Channel: http://youtube.com/c/AchieveIAS Telegram Channel: http://t.me/Achieve_Ias,
Mail: achieveias21@gmail.com, Contact Number: 8968920720
https://upscpdf.com https://upscpdf.com https://upscpdf.com
ACHIEVE IAS MCQ SERIES, HISTORY DAY 2, SOLUTIONS

Q1: With reference to vedic history of India, consider the following statements:
1. In Rigveda, hymns are called as Suktas which means well said.
2. Some of the hymns in the Rigveda are in the form of dialogues between a sage named
Vishvamitra, and two rivers, Beas and Yamuna.
3. Iron was unknown in Later vedic period.
4. Dasyus were the opponent of Aryans who did not perform sacrifices.

Which of the statements given above are correct?


(a) 1, 2 and 4. (b) 1 and 4 only (c)1, 3 and 4 (d)All of the above

Explanation: Answer is (b)

Hymns in the Rigveda are in the form of dialogues between a sage named Vishvamitra, and two
rivers, Beas and SUTLEJ. Later vedic period was the beginning of the use of Iron introduced
around 1000BCE and is mentioned as Krishna/Shyama Ayas. The Rigveda includes more than
a thousand hymns, called sukta or “well-said”. These hymns are in praise of various gods and
goddesses. Sometimes, the people who composed the hymns described themselves as Aryas and
called their opponents Dasas or Dasyus. These were people who did not perform sacrifices, and
probably spoke different languages. Later, the term dasa (and the feminine dasi) came to mean
slave.

Q2: Which of the following statement is correct about Megaliths?

(a) Megaliths were a kind of stone tool used in megalithic culture.


(b) These were stone boulders mainly used to mark burial sites.
(c) Megaliths were material used for making vessels.
(d) Megaliths were minerals mainly found in banks of river sutlej.

Explanation: Answer is (b)


Megaliths were stone boulders (literally big stones) carefully arranged by people, and were used to
mark burial sites. The practice of erecting megaliths began about 3000 years ago, and was prevalent
throughout the Deccan, south India, in the north-east and
Kashmir. Megalith is a large, often undressed stone, that has been used in the construction of various
types of Neolithic, Chalcolithic or Bronze Age monuments, during the period 4500-1000 BCE.

Q3. Consider the following statement:


1. Archaeologists have found oracle bones at Inamgaon site.
2. Oracle bones are some of the first evidence of writing in China.
3. They were used to predict the future.

Which of the statements given above are correct?

Google it:- https://upscpdf.com


www.achieveias.co.in, YouTube Channel: http://youtube.com/c/AchieveIAS Telegram Channel: http://t.me/Achieve_Ias,
Mail: achieveias21@gmail.com, Contact Number: 8968920720
https://upscpdf.com https://upscpdf.com https://upscpdf.com
(a)1 and 2 only (b)1 and 3 only (c)2 and 3 only (d) All of the above

Explanation: Answer is (c)

Oracle bones are some of the first evidence of writing in China.They were used to predict the future.
Kings got scribes to write questions on the bones — would they win battles? Would the harvest be
good? Would they have sons?The bones were then put into the fire, and they cracked because of the
heat. Then fortunetellers studied these cracks, and tried to predict the future. Inamgaon is a site on
the river Ghod, a tributary of the Bhima in Maharastra.No oracle bone have been found there.
Archaeologists have found seeds of wheat, barley, rice, pulses, millets, peas and sesame. Bones of a
number of animals, many bearing cut marks that show they may have been used as food, have also
been found. These include cattle, buffalo, goat, sheep, dog, horse, ass, pig, sambhar, spotted deer,
blackbuck, antelope, hare, and mongoose, besides birds, crocodile, turtle, crab and fish. There is
evidence that fruits such as ber, amla, jamun, dates and a variety of berries.

Q4. Consider the following:


1. Rig Veda
2. Samaveda
3. Yajurveda
4. Atharvaveda.

Which of the following is/are not a later vedic book?


(a) 1 only (b) 1 and 4 only (c) 3 and 4 only (d)All of the above

Explanation: Answer is (a)

Samaveda, Yajurveda and Atharvaveda, are often called later Vedic, because they were composed
after the Rigveda. Some other books were composed by priests, and described how rituals were to
be performed. They also contained rules about society.

Q5. Which of the following is not a Mahajanapada?

1. Vajji
2. Malla
3. Vatsa
4. Assaka

Choose from the following Options:


(a) 1 and 4 only (b) 2 and 3 only (c) 4 only (d)None of the above.

Explanation: Answer is (d)

All are Mahajanapadas. There were sixteen of such Mahajanapadas: Kasi, Kosala, Anga, Magadha,
Vajji, Malla, Chedi, Vatsa, Kuru, Panchala, Machcha, Surasena, Assaka, Avanti, Gandhara and
Kamboja.
Google it:- https://upscpdf.com
www.achieveias.co.in, YouTube Channel: http://youtube.com/c/AchieveIAS Telegram Channel: http://t.me/Achieve_Ias,
Mail: achieveias21@gmail.com, Contact Number: 8968920720
https://upscpdf.com https://upscpdf.com https://upscpdf.com
Kasi is a region settled around Varanasi. It has a predominant position among the sixteen
Mahajanapadas. Matsya Purana and Alberuni talk immensely about Kasi read as Kausika and
Kaushika there. Also we come to know much about Kasi from the folklores of the Jatakas.

Kosala comprises of Shravasti, Kushavati, Saket and Ayodhya. Also it constituted of the modern
cities of Oudh (Awadh), Uttar Pradesh. Ayodhya was under the control of the Kosala king Prosenjit. A
neighboring state was the famous Magadha.

Anga was one of the earliest of all. This was around the Gangetic plains. This state was known by
various names such as Malini, Champapuri, Kala malini, Champa malini etc. it was in the Atharva
Veda that the Angas were first mentioned.

Magadha was a powerful kingdom and was run by Bimbisara and Ajatshatru, his son. As per the
Vedas Magadha is the 'semi Brahman' state. In the later Vedic scriptures and texts it was referred to
as Kikata. Also it is popular by various names as Magadhapura, Brihadrathapura, Vasumati etc.

Vajji comprised of many different social groups and villages. It was a confederation of many clans
such as the Licchavis, the Vedehans, the Jnatrikas and most importantly the Vajjis. Its capital was
located at Vaishali.

Malla has been mentioned in the Buddhist and the Jain works. It existed in a republic of nine
territories. The Mallas were known for their bravery and warlike temperament. They were conquered
by Magadha after Buddha's death.

Chedi people existed on the southern part of the Yamuna River. The capital of Chedis was Suktimati
and the Rigveda gives details of this Mahajanapada. It was ruled by Sisupala and it was this city
where the Pandavas (from Mahabharata) chose to spend the thirteenth year of their exile.

Vatsa or also Vamsa followed a monarchial form of government. The capital of this Mahajanapada
was located at Kausambi. It became a hub of all the economical activities, business and trade.

Kuru basically belonged to the Puru-Bharata family. These were the people who originated from
Kurukshetra. They are believed to have shifted to the republic form of government in the fifth of the
sixth century BCE.

Panchala was divided into two parts: Uttara Panchala and Dakhsina Panchala with Chhatravati and
Kampilya there capitals respectively.

Machcha was located to the south of Kuru and west to the river of Yamuna. As per the Pali literature
the Machchas are generally linked with the Surasena. Its capital was Viratanagara.

Surasena witnessed great metamorphism in religion. Its capital was Mathura. Earlier Lord Krishna
was worshipped here later the disciples of Buddha took over this Mahajanapada.

Google it:- https://upscpdf.com


www.achieveias.co.in, YouTube Channel: http://youtube.com/c/AchieveIAS Telegram Channel: http://t.me/Achieve_Ias,
Mail: achieveias21@gmail.com, Contact Number: 8968920720
https://upscpdf.com https://upscpdf.com https://upscpdf.com
Assaka also known as Ashmaka, this was situated in the southern part of the country. Its capital was
located at Potali.

Avanti lay in the western India. This kingdom nurtured Buddhism immensely. Its capital was known
as Ujjaini. Initially its capital was Mahissati which later was integrated into Ujjaini. Avanti later
dissolved in the Magadha Empire.

Gandhara comprised of the Gandharas who were believed to be excellently tarind in the art of war
and have been mentioned in the Atharva Veda. The Gandhara are included in the Uttarapatha by the
Puranic and Buddhist traditions.

Kamboja was believed to have consisted of the areas around the Hindukush. It is mentioned in the
great epic Mahabharata in many excerpts.

Q6: Consider the following statement:


1. Both the kshatriyas and the vaishyas could perform sacrifices.
2. Rajagriha (present-day Rajgir) in Bihar was the capital of Magadha for several years.
3. No evidence of punch marked coins in Mahajanapada period.

Which of the statements given above are correct?

(a) 1 and 2 only (b) 1 and 3 only (c) 2 and 3 only (d) All of the above.

Explanation: Answer is (a)


The first varna was that of the brahmin. Brahmins were expected to study (and teach) the Vedas,
perform sacrifices and receive gifts. In the second place were the rulers, also known as kshatriyas.
They were expected to fight battles and protect people. Third were the vish or the vaishyas. They
were expected to be farmers, herders, and traders. Both the kshatriyas and the vaishyas could
perform sacrifices.Last were the shudras, who had to serve the other three groups and could not
perform any rituals. Evidences have been found that Some payments were probably made using
punch marked coins.

Q7. Consider the following:

1. Bodh Gaya
2. Sarnath
3. Kusinara
4. Ujjain

Which of the following is not associated to the life of Buddha?

(a) 3 only (b) 4 only (c) 3 and 4 only (d) None of the above

Explanation: Answer is (b)

Bodh Gaya, Sarnath and Kusinara are associated with buddha's life.
Google it:- https://upscpdf.com
www.achieveias.co.in, YouTube Channel: http://youtube.com/c/AchieveIAS Telegram Channel: http://t.me/Achieve_Ias,
Mail: achieveias21@gmail.com, Contact Number: 8968920720
https://upscpdf.com https://upscpdf.com https://upscpdf.com
Lumbini: Lumbini is currently located in Kapilavastu district of Nepal. It is birth place of Buddha. At the
time of birth of Buddha, Lumbini was a part of Shakya Janapada, which was a republic.

Bodhgaya: It is located in Bihar on the bank of river Neranjana {this river was known as Uruwela at
that time}. It is known for place of enlightment of Buddha.

Sarnath
Sarnath {also known as Mrigadava, Migadaya, Rishipattana, Isipatana} is the deer park where
Gautama Buddha delivered his first sermon or Dhammachakraparivartan Sutra. At the time of
Buddha, it was a part of Kashi Janapada.

Kushinagar: Kushinara or Kushinagara is located in Kushinagar district of Uttar Pradesh. It is the site
of Buddha’s death and mahaparinirvana. At the time of Buddha’s death, it was a capital of Malla
janapada.

Sravasti: Sravasti was located in Uttar Pradesh around area of Balrampur in modern Uttar Pradesh. It
is closely asscoiated with the life of Buddha because Buddha had spent 24 Chaturmasas {implies 24
years because one year as only one Chaturmasa between Ashadha to Kartika}. Thus, we can say
that most of monastic life of Buddha was spent in Shravasti. In Buddha’s times, Shravasti was capital
of Kosala Kingdom. Shravasti is also birthplace of Jaina Tirthankar Sambhavanath, and thus is
important for Jains also.

Sankasya: It’s current location is Farrukhabad district of Uttar Pradesh. It has some faiths of
Buddhism that Buddha after is death descended from heaven here.

UJJAIN: Is a famous Hindu pilgrimage centre.It is not associated with buddha's life, however
according to the Sinhalese Buddhist tradition, king Ashoka's children Mahendra and Sanghamitra,
who preached Buddhism in modern Sri Lanka, were born in Ujjain.

Q8: Consider the following statement:


1. The Buddha belonged to a small gana known as the Sakya gana, and was a Kshatriya.
2. As per Upanishads ultimately, both the atman and the brahman were one.
3. There is no mention of women thinkers in Upanishads.
4. As per vanaprastha ashram one had to live in the forest and meditate.

Which of the statements given above are correct?

(a)1 and 2 only (b)1,3 and 4 only (c)1 ,2 and 4 only (d)All of the above.

Explanation: Answer is (c)


Buddha was born in Lumbini, a part of Shakya gana and he was a kshatriya. Many of Upanishads
thinkers felt that there was something permanent in the universe that would last even after death.
They described this as the atman or the individual soul and the brahman or the universal soul. They
believed that ultimately, both the atman and the brahman were one. Most Upanishadic thinkers
were men, especially brahmins and rajas. Occasionally, there is mention of women thinkers, such as

Google it:- https://upscpdf.com


www.achieveias.co.in, YouTube Channel: http://youtube.com/c/AchieveIAS Telegram Channel: http://t.me/Achieve_Ias,
Mail: achieveias21@gmail.com, Contact Number: 8968920720
https://upscpdf.com https://upscpdf.com https://upscpdf.com
Gargi, who was famous for her learning, and participated in debates held in royal courts. Four
ashramas were recognised: brahmacharya, grihastha, vanaprastha and samnyasa. Brahmin,
kshatriya and vaishya men were expected to lead simple lives and study the Vedas during the early
years of their life (brahmacharya). Then they had to marry and live as householders (grihastha). Then
they had to live in the forest and meditate (vanaprastha). Finally, they had to give up everything
and become samnyasins.

Q9: It is an early type of Buddhist monastery consisting of an open court surrounded by open
cells accessible through an entrance porch. Theses were originally constructed to shelter the
monks when it became difficult for them to lead the wanderer’s life.

Which of the following have been addressed in above statement:

(a) Vihara (b) Stupa (c) chaitya (d) sangha

Explanation: Answer is (a)

Vihara, early type of Buddhist monastery consisting of an open court surrounded by open cells
accessible through an entrance porch. The viharas in India were originally constructed to shelter the
monks during the rainy season, when it became difficult for them to lead the wanderer’s life.

Stupa, Buddhist commemorative monument usually housing sacred relics associated with the
Buddha or other saintly persons. The hemispherical form of the stupa appears to have derived from
pre-Buddhist burial mounds in India.

Chaitya was a shrine or a buddhist place of worship, where the buddhist devotees used to assamble
for prayers and worship.

Sangha, Buddhist monastic order, traditionally composed of four groups: monks, nuns, laymen, and
laywomen. The sangha is a part—together with the Buddha and the dharma (teaching)—of the
Threefold Refuge, a basic creed of Buddhism. The sangha originated in the group of disciples who
renounced the worldly life to wander with the Buddha and listen to his teachings. After the Buddha’s
death his disciples continued to live together as a community, wandering from place to place, living
off the receipt of alms. The sangha originated in the group of disciples who renounced the worldly life
to wander with the Buddha and listen to his teachings. After the Buddha’s death his disciples
continued to live together as a community, wandering from place to place, living off the receipt of
alms.

FACTS TO REMEMBER:

(1) Avesta, also called Zend-avesta, sacred book of Zoroastrianism containing its cosmogony, law,
and liturgy, the teachings of the prophet Zoroaster.

(2)Vinaya Pitaka - contains the rules of the Order of Buddhist monks which must be observed for
achieving purity of conduct. Hence, it is also called the Book of Discipline. It, in turn, has three books
viz., Suttavibhanga, Khandhaka, and Parivara.
Tripitaka also known as the Threefold Basket, include - Vinaya Pitaka, Sutta Pitaka, and
Abhidhamma Pitaka.
Google it:- https://upscpdf.com
www.achieveias.co.in, YouTube Channel: http://youtube.com/c/AchieveIAS Telegram Channel: http://t.me/Achieve_Ias,
Mail: achieveias21@gmail.com, Contact Number: 8968920720
https://upscpdf.com https://upscpdf.com https://upscpdf.com
(3)Jainas: They were followers of Mahavira, who had to lead very simple lives, begging for food.
They had to be absolutely honest, and were especially asked not to steal. Also, they had to observe
celibacy. And men had to give up everything, including their clothes.

Q10: Consider the following statement:


1. Most of Ashoka’s inscriptions were in Prakrit and were written in the Brahmi script.
2. Dhamma mahamatta were officials appointed by Ashoka who went from place to place teaching
people about dhamma.
3. The State Emblem of India is a representation of the Lion Capital of Ashoka, Sarnath.
4. Devanampiya (Sanskrit Devanampriya meaning Beloved of the Gods) and Piyadasi were other
names of Ashoka.

Which of the statements given above are correct?


(a)1 and 2 only (b)1,3 and 4 only (c)1 ,2 and 4 only (d)All of the above.

Explanation: Answer is (d)

Most of Ashoka’s inscriptions were in Prakrit and were written in the Brahmi script.Both were
common among his subjects.Ashoka was the first ruler who tried to take his message to the people
through inscriptions.

Brahmi script used on the Indian Subcontinent and in Central Asia, during the final centuries BCE and
the early centuries CE.The best-known Brahmi inscriptions are the rock-cut edicts of Ashoka in north-
central India, dated to 250-232 BCE. The script was deciphered in 1837 by James Prinsep, an
archaeologist, philologist, and official of the British East India Company.

People in the Asoka's empire followed different religions, and this sometimes led to conflict. Ashoka
felt it was his duty to solve these problems. So, he appointed officials, known as the dhamma
mahamatta who went from place to place teaching people about dhamma. Besides, Ashoka got
his messages inscribed on rocks and pillars instructing his officials to read his message to those who
could not read it themselves.

Lion capital is most celebrated Ashokan pillars, erected at Sarnath, the site of Buddha’s First Sermon
where he shared the Four Noble Truths (the dharma or the law). Currently, the pillar remains where it
was originally sunk into the ground, but the capital is now on display at the Sarnath Museum. It is
this pillar that was adopted as the national emblem of India.

The various names of Ashoka include Buddhashakya, Dharmasoka, Devanampiya (Sanskrit


Devanampriya meaning Beloved of the Gods) and Piyadasi.

Google it:- https://upscpdf.com


www.achieveias.co.in, YouTube Channel: http://youtube.com/c/AchieveIAS Telegram Channel: http://t.me/Achieve_Ias, Mail:
achieveias21@gmail.com, Contact Number: 8968920720
https://upscpdf.com https://upscpdf.com https://upscpdf.com
ACHIEVE IAS MCQ SERIES, HISTORY, DAY 3, SOLUTIONS

Q1: Consider the following pairs:


1. Grihapatis Independent Farmers
2. Uzhavar Ordinary Ploughmen
3. Vellalar Large Landowners
4. Kadaisiyar Landless Labourers
5. Gramabhojaka Village Headman

With reference to the history of north and south India, Which of the pairs given above is/are correctly
matched?
(a) 1, 2 and 3 (b) 1, 3 and 4 (c) 2, 3, 4 and 5 (d) All of the above

Explanation: Answer is (d)

In the Tamil region, large landowners were known as vellalar, ordinary ploughmen were known as
uzhavar, and landless labourers, including slaves, were known as kadaisiyar and adimai. In the
northern part of the country, the village headman was known as the grama bhojaka. Usually, men
from the same family held the position for generations. In other words, the post was hereditary. The
grama bhojaka was often the largest landowner. Apart from the gramabhojaka, there were other
independent farmers, known as grihapatis, most of whom were smaller landowners. And then there
were men and women such as the dasa karmakara, who did not own land, and had to earn a living
working on the fields owned by others.

Q2: With reference to the history of India, which of the given statement is not correct about
Mathura?
(a) Mathura was located at the cross roads of two major routes of travel and trade — from the
northwest to the east and from north to south
(b) Mathura was known for extremely fine sculpture in ancient time.
(c) Mathura was the second capital of the Kushanas.
(d) Roman lamps, glassware and gems have been found in Mathura.

Explanation: Answer is (d)


Arikamedu, the ancient Roman trade centre (Puducherry) on the right bank of Ariyankuppam River. It
has a long history that dates back to the second century B.C. Arikamedu was a coastal settlement
where ships unloaded goods from distant lands. A massive brick structure, which may have been a
warehouse,was found at the site. Roman lamps, glassware and gems have been found at the
site.

Mathura has been an important settlement for more than 2500 years. It was important because it was
located at the cross roads of two major routes of travel and trade — from the northwest to the east
and from north to south. There were fortifications around the city, and several shrines. Farmers and
herders from adjoining areas provided food for people in the city. Mathura was also a centre where
some extremely fine sculpture was produced. Around 2000 years ago Mathura became the second
capital of the Kushanas. Mathura was also a religious centre — there were Buddhist monasteries,
Jaina shrines, and it was an important centre for the worship of Krishna. Several inscriptions on
surfaces such as stone slabs and statues have been found in Mathura. Generally, these are short
inscriptions, recording gifts made by men (and sometimes women) to monasteries and shrines.

Google it:- https://upscpdf.com


www.achieveias.co.in, YouTube Channel: http://youtube.com/c/AchieveIAS Telegram Channel: http://t.me/Achieve_Ias, Mail:
achieveias21@gmail.com, Contact Number: 8968920720
https://upscpdf.com https://upscpdf.com https://upscpdf.com
These were made by kings and queens, officers, merchants, and crafts persons who lived in the city.
For instance, inscriptions from Mathura mention goldsmiths, blacksmiths, weavers, basket makers,
garland makers, perfumers.

Q3: Which of the following is not correct regarding ring wells?


(a) Ring wells were a type of structure in which rows of pots, or ceramic rings arranged one on top of
the other.
(b) Ring wells have been used as toilets, as drains and as garbage dumps.
(c) Ring wells were absent in Mauryan period.
(d) Ring wells were found in individual houses

Explanation: Answer is (c)

In many cities, archaeologists have found rows of pots, or ceramic rings arranged one on top of the
other. These are known as ring wells. These seem to have been used as toilets in some cases, and
as drains and garbage dumps. These ring wells are usually found in individual houses. In an
excavation at Purana Qila in 2014, Ring wells were discovered dating back to Mauryan period. As per
Archaeological Survey of India (ASI), it is a drain, probably of a kitchen in the Maurya period.

Q4: Which of the following statement is not correct about city Bharuch, Gujrat?
(a) It is the second-oldest city of India having continuous habitation.
(b) In Greek and Roman it was called Barygaza.
(c) It was location for import only.
(d) It is mentioned extensively as a major trading partner of the Roman world.

Explanation: Answer is (c)


Bharuch was known for both imports and exports. As mentioned by the Greeks, the imports into
Barygaza were wine, copper, tin, lead, coral, topaz, cloth, gold and silver coins. Exports from the town
included plants from the Himalayas, ivory, agate, carnelian, cotton, silk and perfumes. Bharuch is the
oldest city of Gujarat. It is also the second-oldest city of India having continuous habitation, first being
Kashi (Varanasi). Bharuch has a known history for about 8000 years. It was known to the Greeks and
Romans as Barygaza, and probably had a settlement of Greek traders. As one southern terminus of
the Kamboja-Dvaravati Route, it is mentioned extensively as a major trading partner of the Roman
world.

Q5: Which of the following was a very important seaport of the ancient Chola kingdom?

(a) Kaveripattinam (b) Madurai (c) Motupalli (d) Bharuch

Explanation: Answer is (a)


Kaveripattinam, the chief port of the ancient Chola kingdom, was situated at the mouth of Kaveri
river. It is today identified with Puhar, a town in the Nagapattinam district of Tamil Nadu.
Kaveripattinam is also known as Kaveripaddinam, and Kaveripumpattinam. Once the capital of the
early Chola rulers, Kaveripattinam has been described in various Sangam works and also
in Ptolemy’s Geography and the Periplus of the Erythrean Sea. According to a Sinhalese source,
spacious harbour of Kaveripattinam was built by soldiers captured by the great Chola
King Karikalan in a raid on Ceylon. During the first three centuries of the Christian era,
Kaveripattinam was a prosperous city inhabited by rich merchants, traders, craftsmen and sailors.
Madurai was the capital of the Pandyas. Motupalli , now in Krishna district, was an important seaport
in kakatiya kingdom. Bharuch is the oldest city of Gujarat. It is also the second-oldest city of India
Google it:- https://upscpdf.com
www.achieveias.co.in, YouTube Channel: http://youtube.com/c/AchieveIAS Telegram Channel: http://t.me/Achieve_Ias, Mail:
achieveias21@gmail.com, Contact Number: 8968920720
https://upscpdf.com https://upscpdf.com https://upscpdf.com
having continuous habitation, first being Kashi (Varanasi). Bharuch has a known history for about
8000 years.

Q6: which of the given statement is not correct about Kushanas?


(a) Kushanas had control over silk route.
(b) Peshawar and Mathura were two major centres of their power.
(c) Kushanas issued gold coins.
(d) Kanishka was the founder of this empire.

Explanation: Answer is (d)


Kujula Kadphises is regarded as founder of Kushana Empire. He amalgamated the five clans of
Yue-chi tribe and laid the foundation of unified kushana empire. Kanishka was the most powerful ruler
of the Kushana Empire. The capital of his empire was Purushpura (Peshawar). Under his rule,
Kushana Empire extended from Uzbekistan, Tajikistan to Mathura and Kashmir. Kanishka was the
successor of Vima Kadphises, as demonstrated by an impressive genealogy of the Kushan kings,
known as the Rabatak inscription. Kushanas were best-known for controlling the Silk Route.
They ruled over central Asia and north-west India around 2000 years ago. Their two major centres
of power were Peshawar and Mathura. Taxila was also included in their kingdom. During their rule,
a branch of the Silk Route extended from Central Asia down to the seaports at the mouth of the river
Indus, from where silk was shipped westwards to the Roman Empire. The Kushanas were amongst
the earliest rulers of the subcontinent to issue gold coins. These were used by traders along the
Silk Route.

Q7: Consider the following pairs:

1. Pepper Black gold


2. Silk China
3. Muvendar Three cheifs
4. Dakshinapatha Satavahanas

Which of the pairs given above is/are correctly matched?


(a) 1,2 and 3 (b) 1,3 and 4 (c) 2,3 ,4 and 5 (d) All of the above

Explanation: Answer is (d)


South India was famous for gold, spices, especially pepper, and precious stones. Pepper was
particularly valued in the Roman Empire, so much so that it was known as black gold. Techniques of
making silk were first invented in China around 7000 years ago. While the methods remained a
closely guarded secret for thousands of years, some people from China who went to distant lands on
foot, horseback, and on camels, carried silk with them. The paths they followed came to be known as
the Silk Route. Sangam poems mention the muvendar. This is a Tamil word meaning three chiefs,
used for the heads of three ruling families, the Cholas, Cheras, and Pandyas who became powerful in
south India around 2300 years ago. Around 200 years later a dynasty known as the Satavahanas
became powerful in western India. The most important ruler of the Satavahanas was Gautamiputra
Shri Satakarni. We know about him from an inscription composed by his mother, Gautami Balashri.
He and other Satavahana rulers were known as lords of the dakshinapatha, literally the route leading
to the south, which was also used as a name for the entire southern region.

Google it:- https://upscpdf.com


www.achieveias.co.in, YouTube Channel: http://youtube.com/c/AchieveIAS Telegram Channel: http://t.me/Achieve_Ias, Mail:
achieveias21@gmail.com, Contact Number: 8968920720
https://upscpdf.com https://upscpdf.com https://upscpdf.com
Q8: consider the following statement:
(a) Kanishka organized 4th Buddhist council, where scholars met and discussed important matters.
(b) Buddhacharita, the biography of Buddha was composed by Ashvaghosha.

Which of the statements given above are correct?


(a) 1 Only (b) 2 Only (c) Both 1 and 2 (d) Neither 1 nor 2

Explanation: Answer is (c)


Kanishka was a great patron of Buddhism. He organized 4th Buddhist council at Kashmir, presided by
Vasumitra. During this council the collection of Buddhist texts took place and the commentaries were
engraved on copper sheets. During this council Buddhism was divided into Mahayana and Hinayana.
Kanishka had invaded Patliputra and had taken the Buddhist monk Asvaghosa to Purushpura.
Ashvaghosha. composed a biography of the Buddha, the Buddhacharita. He lived in kanishka’s court.

Q9: which of the following statement is not true about Mahayana Buddhism?
(a) The term Mahayana is a Sanskrit word which literally means "Lesser Vehicle".
(b) It believes in the heavenliness of Buddha and Idol worship of Buddha.
(c) Buddhist schools embedded in China, Korea and Japan belong to the Mahayana tradition.
(d) It believed in concept of Bodhisattvas.

Explanation: Answer is (a)


The term Mahayana literally means "Great Vehicle". Hinayana means "Lesser Vehicle".
This sect of Buddhism believes in the heavenliness of Buddha and believes in Idol Worship. Mahayan
sect spread from India to several other nations such as China, Korea, Japan, Taiwan, Nepal, Tibet,
Bhutan and Mangolia. Mahayana believes in Mantras. Its main principles were based on the
possibility of universal liberation from suffering for all beings. That’ why, this sect is called Mahayana
(The Great Vehicle). Its principles are also based on the existence of Buddhas and Bodhisattvas
embodying Buddha nature. It allows salvation through having faith and committing oneself to Buddha.

Hinayana Hinayana follows the original teaching of Buddha. It emphasizes individual salvation
through self-discipline and meditation.

Major difference between Hinayana and Mahayana


Mahayana worships the bodhisattvas and read the Mahayana sutras while Hinayanists do not
perform these. Mahayana Buddhism followers think that The Buddha is a God because they think that
the Buddha came down to earth to help people cross the sea of life. So the Buddha can be
worshipped as a God because he is eternal and comes down to earth. On the other hand Hinayana
Buddhists think that The Buddha was a Human instead of a God because they think The Buddha was
simply a man who found a way to Nirvana.
Hinayana Buddhism Mahayana Buddhism
1-Followed as a teaching or 1-Followed with reference to higher beings, more like a
Philosophy. religion.

2-Found mostly in the South and West


2-Found mostly in the North and West, covering China,
covering Indochina and Ceylon
Korea, Japan, and Tibet.
(Sri-Lanka).

3-Early work written in Pali (e.g.


3-Early texts are in Sanskrit (e.g. Karma, Dharma)
Kamma, Dhamma).

Google it:- https://upscpdf.com


www.achieveias.co.in, YouTube Channel: http://youtube.com/c/AchieveIAS Telegram Channel: http://t.me/Achieve_Ias, Mail:
achieveias21@gmail.com, Contact Number: 8968920720
https://upscpdf.com https://upscpdf.com https://upscpdf.com
Q10: which of the following Chinese Buddhist pilgrim never visited India?

(a) Fa Xian (b) Xuan Zang (c) I-Qing (d) Lao Zu

Explanation: Answer is (d)


Faxian was a Chinese Buddhist monk and translator who traveled by foot from Ancient
China to Ancient India, visiting many sacred Buddhist sites in Central Asia, the Indian
subcontinent and Southeast Asia between 399-412 to acquire Buddhist texts. His journey is
described in his important travelogue, A Record of Buddhist Kingdoms, Being an Account by the
Chinese Monk Fa-Xian of his Travels in India and Ceylon in Search of the Buddhist Books of
Discipline. Other transliterations of his name include Fa-Hien, and Fa-hsien.
Xuan Zang was a Chinese Buddhist monk, scholar, traveller, and translator who travelled to India in
the seventh century and described the interaction between Chinese Buddhism and Indian
Buddhism during the early Tang dynasty. During the journey he visited many sacred Buddhist sites in
what are now Pakistan, India, Nepal, and Bangladesh. He was born in what is now Henan province
around 602, from boyhood he took to reading religious books, including the Chinese classics and the
writings of ancient sages.
I-Qing, was also a Chinese Buddhist monk who came to India about 50 years after Xuan Zang.

Lao Zu (also Laozi or Lao-Tzu) is a legendary figure in Chinese folk history. It is believed that Lao Zu
lived around 500 BC. He is most famous as the author of the Tao Te Ching — and as such, the
founder of the religion/philosophy Taoism. Lao zu was not a Buddhist and he had never visited India.

IMPORTANT FACTS: The beginning of Bhakti: Deities such as Shiva, Vishnu, and Shakti became a
central feature of later Hinduism and gained massive in importance. These deities were worshipped
through Bhakti, an idea that became very popular at this time. Bhakti is generally understood as a
person’s devotion to his or her chosen deity. Anybody, whether rich or poor, belonging to the
so-called ‘high’ or ‘low’ castes, man or woman, could followthe path of Bhakti. The idea of Bhakti is
present in the Bhagavad Gita, a sacred book of the Hindus.

Google it:- https://upscpdf.com


www.achieveias.co.in, YouTube Channel: http://youtube.com/c/AchieveIAS Telegram Channel: http://t.me/Achieve_Ias,
Mail: achieveias21@gmail.com, Contact Number: 8968920720
https://upscpdf.com https://upscpdf.com https://upscpdf.com
ACHIEVE IAS HISTORY MCQ SERIES, DAY 4, SOLUTIONS

Q1. With reference to the history of India, which of the given statement is not correct about
Prashastis?

a. Prashastis were a form of poetry or ornate prose written in praise of the rulers.
b. Samudragupta’s prashasti was composed by his court poet Harishena in Sanskrit.
c. Samudragupta has been described as maharaj-adhiraja in his Prashastis.
d. Samudragupta built Allahabad pillar to inscribe his Prashastis.

Answer: D

Samudragupta’s Prashasti was inscribed on the Ashokan pillar at Allahabad. It was of course not built
by him. The Allahabad pillar is an Ashoka Stambha, one of the Pillars of Ashoka, erected by Ashoka,
Emperor of the Maurya dynasty, who reigned in the 3rd century BCE.

Q2. Consider the following statement:


1. The rulers of Aryavarta, surrendered to Samudragupta after being defeated and he then allowed
them to rule again.
2. He was defeated by the ruler belonging to the Chalukya dynasty, Pulakeshin II.
3. Descendants of the Kushanas and Shakas were uprooted, and their kingdoms were made a part of
Samudragupta’s empire.

Which of the following statement is correct regarding conquests of Samudragupta?


(a) 2 and 3 only (b) 1 and 3 only
(c) All of the above (d) None of the above.

Answer. D

Rulers of Aryavarta, the nine rulers who were uprooted, and their kingdoms were made a part of
Samudragupta’s empire. Harshavardhana, the ruler of Vardhana dynasty was defeated by the ruler
belonging to the Chalukya dynasty, Pulakeshin II. The descendants of the Kushanas and Shakas,
and the ruler of Sri Lanka, who submitted to him and offered daughters in marriage.

Q3. With reference to the history of India, consider the following statement:
1. Samudragupta, was the first ruler of the Gupta dynasty to adopt the grand title of maharaj-adhiraja.
2. Nagaram was an organisation of merchants controlled by rich samantas.
3. Fa Xian visited india during reign of Harshavardhana
4. Aihole, was the capital of the Chalukyas.

Which of the given statement is correct?


(a) 1 and 3 Only (b) 2 and 4 only
(c) 4 only (d) None of the above

Answer: C

Google it:- https://upscpdf.com


www.achieveias.co.in, YouTube Channel: http://youtube.com/c/AchieveIAS Telegram Channel: http://t.me/Achieve_Ias,
Mail: achieveias21@gmail.com, Contact Number: 8968920720
https://upscpdf.com https://upscpdf.com https://upscpdf.com
Only statement 4 is correct. Chandragupta,the father of Samudragupta was the first ruler of the Gupta
dynasty to adopt the grand title of maharaj-adhiraja nagaram was an organisation of merchants. It is
likely that these assemblies were controlled by rich and powerful landowners and merchants. Fa Xian
visited india during reign of Chandragupta II.

Q4. With reference to the courtiers of ancient history which of the given statement is not
correct?
a. Ravikirti, a poet belonged to court of the Pallavas.
b. Banabhatta was the court poet of Harshavardhana.
c. Kalidasa the poet, and Aryabhata the astronomer were in the court Chandragupta 2
d. Harishena, the court poet of Samudragupta also held the post of kumar-amatya

Answer is (a)

Ravikiriti belongs to the Chalukya dynasty. Ravikirti was the court poet of Pulakeshin II, the best-
known Chalukya ruler. Ravikirti composed prashastis in praise of Pulakeshin II. Rest are correct

Q5. Consider the following:

1. Nagara shreshthi - chief judicial officer of the city


2. Sarthavaha - leader of the merchant caravans
3. Prathama-kulika - chief craftsman

Which among these are correctly matched?


(a) 1 and 2 only (b) 2 and 3 only
(c) All of the above (d) None of the above

Answer is (b)

Important men probably had a say in local administration. These included the nagarashreshthi or
chief banker or merchant of the city, the sarthavaha or leader of the merchant caravans, the
prathama-kulika or the chief craftsman, and the head of the kayasthas or scribes.

Q6. In context of the inscriptions of the Pallavas, consider the following statement:
1. Sabha was a local assembly which looked after irrigation, agricultural operations, making roads,
local temples, etc.
2. The ur was a village assembly found in areas where the land owners were not brahmins.

Which of the given statement is correct?


(a)1 only (b)2 only
(c) All of the above (d)None of the above

Answer is (c)

The inscriptions of the Pallavas mention a number of local assemblies. These included the sabha,
which was an assembly of brahmin land owners. This assembly functioned through subcommittees,
Google it:- https://upscpdf.com
www.achieveias.co.in, YouTube Channel: http://youtube.com/c/AchieveIAS Telegram Channel: http://t.me/Achieve_Ias,
Mail: achieveias21@gmail.com, Contact Number: 8968920720
https://upscpdf.com https://upscpdf.com https://upscpdf.com
which looked after irrigation, agricultural operations, making roads, local temples, etc. The ur was a
village assembly found in areas where the land owners were not brahmins. And the nagaram was an
organisation of merchants. It is likely that these assemblies were controlled by rich and powerful
landowners and merchants. Many of these local assemblies continued to function for centuries.

Q7. Which of the following book is the story of merchant Kovalan and a courtesan Madhavi?
a. Cīvaka Cintāmaṇi
b. Valayapathi
c. Kuṇṭalakēci.
d. Silappadikaram

Answer is (d)
A famous Tamil epic, the Silappadikaram, was composed by a poet named Ilango, around 1800
years ago. It is the story of a merchant named Kovalan, who lived in Puhar and fell in love with a
courtesan named Madhavi, neglecting his wife Kannagi. Later, he and Kannagi left Puhar and went to
Madurai, where he was wrongly accused of theft by the court jeweller of the Pandya king. The king
sentenced Kovalan to death. Kannagi, who still loved him, was full of grief and anger at this injustice,
and destroyed the entire city of Madurai. All of these four tamil epics are among “five
great Tamil epics”

Q8. Consider the following statement:

1. The word stupa means a mound.


2. The small box placed at the centre or heart of the stupa mostly contains bodily remains of the
Buddha or his followers or things they used.

Which of the given statement is correct?


(a) 1 Only (b) 2 Only
(c) All of the above (d) None of the above

Answer is (c)

The word stupa means a mound. While there are several kinds of stupas, round and tall, big and
small, these have certain common features. Generally, there is a small box placed at the centre or
heart of the stupa. This may contain bodily remains (such as teeth, bone or ashes) of the Buddha or
his followers, or things they used, as well as precious stones, and coins.

Q9. Who among them is composer of the famous ancient epic, Manimekalai?
a. Ilango
b. Sattanar
c. Kalidasa
d. Tiruttakkatēvar

Answer is (b)

Google it:- https://upscpdf.com


www.achieveias.co.in, YouTube Channel: http://youtube.com/c/AchieveIAS Telegram Channel: http://t.me/Achieve_Ias,
Mail: achieveias21@gmail.com, Contact Number: 8968920720
https://upscpdf.com https://upscpdf.com https://upscpdf.com
Tamil epic, the Manimekalai was composed by Sattanar around 1400 years ago. This describes the
story of the daughter of Kovalan and Madhavi. These beautiful compositions were lost to scholars for
many centuries, till their manuscripts were rediscovered, about a hundred years ago. Manimekalai is
also one of the “five great Tamil epics”

Q10. Consider the following statement:


1. Pradakshina patha, was a path laid around the temples surrounded with railings.
2. Garbhagriha, was the room where the image of the chief deity was
Placed.

Which of the given statement is not correct?


(a) 1 only (b) 2 only
(c) All of the above (d) None of the above

Answer is (a)

A path, known as the pradakshina patha, was laid around the stupa. This was surrounded with
railings. The most important part of the temple was the room known as the garbhagriha, where the
image of the chief deity was placed. It was here that priests performed religious rituals

Google it:- https://upscpdf.com


www.achieveias.co.in, YouTube Channel: http://youtube.com/c/AchieveIAS Telegram Channel: http://t.me/Achieve_Ias,
Mail: achieveias21@gmail.com, Contact Number: 8968920720
https://upscpdf.com https://upscpdf.com https://upscpdf.com
ACHIEVE IAS HISTORY MCQ SERIES, DAY 5, SOLUTIONS
Q1.which among them is not correct regarding developments in Hinduism in Medieval period?
a. Worship of new deities
b. Construction of temples by royalty
c. Emergence of the idea of bhakti
d. Brahmanas losing their royal patronage

Answer: D

During medieval period important changes occurred in what we call Hinduism today. These included
the worship of new deities, the construction of temples by royalty and the growing importance of
Brahmanas, the priests, as dominant groups in society. Their knowledge of Sanskrit texts earned the
Brahmanas a lot of respect in society. Their dominant position was consolidated by the
support of their patrons — new rulers searching for prestige. One of the major developments was
the emergence of the idea of bhakti — of a loving, personal deity that devotees could reach without
the aid of priests or elaborate rituals.

Q2. Consider the following statement:


1. First detailed and quite accurate map was of Indian Subcontinent was made by a French
cartographer
2. Definition of the words like “Hindustan” and “foreigner” were different in medieval from today.
3. Corn and Chillies were the main exports from India to Persia in the early medieval period.

Which of the following statement is correct in context of medieval period?


(a) 2 and 3 Only (b) 1 and 2 only
(c) All of the above (d) None of the above.

Answer. B

MAP 1

This map was made by al-Idrisi. Here south India is where we would expect to find north India. So this
map was not accurate at all.

MAP 2

Google it:- https://upscpdf.com


www.achieveias.co.in, YouTube Channel: http://youtube.com/c/AchieveIAS Telegram Channel: http://t.me/Achieve_Ias,
Mail: achieveias21@gmail.com, Contact Number: 8968920720
https://upscpdf.com https://upscpdf.com https://upscpdf.com

Map 2 was made nearly 600 years after the first, during which time information about the
subcontinent had changed considerably. This map seems more familiar to us and the coastal areas in
particular are surprisingly detailed. This map was made by a French cartographer. This was much
detailed and accurate. The term “Hindustan”, for example. Today we understand it as “India”, the
modern nation state. When the term was used in the thirteenth century by Minhaj-I-Siraj, a chronicler
who wrote in Persian, he meant the areas of Punjab, Haryana and the lands between the Ganga and
Yamuna. He used the term in a political sense for lands that were a part of the dominions of the Delhi
Sultan. The areas included in this term shifted with the extent of the Sultanate but the term never
included south India within it. In the medieval period a “foreigner” was any stranger who appeared say
in a given village, someone who was not a part of that society or culture. New foods and beverages
arrived in the subcontinent: potatoes, corn, chillies, tea and coffee from Arab and Persia. These were
imported to India

Q3: With reference to the history of India, consider the following:


1. Muvendavelan - a velan or peasant serving three kings.
2. Vellanvagai - land for the maintenance of a school
3. Shalabhoga - land of non-Brahmana peasant proprietors
4. Araiyar - chief

Which among these are correctly matched?


(a) 2 and 3 only (b) 1 and 4 only
(c) 2 and 4 only (d) All of the above

Answer. B

Chola kings gave some rich landowners titles like muvendavelan (a velan or peasant serving three
kings), araiyar (chief). Chola inscriptions mention several categories of land.
Vellanvagai- land of non-Brahmana peasant proprietors
Brahmadeya- land gifted to Brahmanas
Shalabhoga -land for the maintenance of a school
Devadana, tirunamattukkani-land gifted to temples
Pallichchhandam-land donated to Jaina institutions

Q4. The term ‘Delhiwal’ in medieval history is used for:


a. Rich Jaina merchants
b. Tomara Rajputs
c. Coins
d. Chahamanas

Google it:- https://upscpdf.com


www.achieveias.co.in, YouTube Channel: http://youtube.com/c/AchieveIAS Telegram Channel: http://t.me/Achieve_Ias,
Mail: achieveias21@gmail.com, Contact Number: 8968920720
https://upscpdf.com https://upscpdf.com https://upscpdf.com
Answer is (c)

Many rich Jaina merchants lived in the city and constructed several temples. Coins minted here,
called dehliwal, had a wide circulation.

Q5.Which dynasty ruled for the longest period of time in delhi sultanate?

a. Tughluq dynasty b. Sayyid dynasty


c. Lodi dynasty d. Slave dynasty

Answer is (a)

Tughluqs ruled for longest which is 93 Years (1320-1413)

Q6. The big temples of Thanjavur were built by which dynasty’s rulers?
a. Chalukya b. Cholas
c. Pandya d. Pallava

Answer is (b)

The big temples of Thanjavur and Gangaikondacholapuram,


built by Rajaraja and Rajendra, are architectural and sculptural marvels.

Q7. Founder of which dynasty was feudatory of the Pallavas before gaining power?
a. Chola b. Pratiharasc.
c. Chauhans d. Chalukyas

Answer is (a)
Vijayalaya was the founder of the Imperial Chola dynasty which was the beginning of one of the most
splendid empires in Indian history. Vijayalaya, a feudatory of the Pallava dynasty, took an opportunity
arising out of a conflict between the Pandya dynasty and Pallava dynasty in c. 850,
captured Thanjavur from Muttarayar, and established the imperial line of the medieval Chola Dynasty.
Thanjavur became the capital of the Imperial Chola Dynasty.

Q8. Consider the following statement:


1. Sultan Mahmud of Ghazni looted many wealthy temples, including that of Somnath, Gujarat.
2. Sultan Muhammad Ghori defeated Chahamana ruler Prithviraja III in very first battle.

Google it:- https://upscpdf.com


www.achieveias.co.in, YouTube Channel: http://youtube.com/c/AchieveIAS Telegram Channel: http://t.me/Achieve_Ias,
Mail: achieveias21@gmail.com, Contact Number: 8968920720
https://upscpdf.com https://upscpdf.com https://upscpdf.com
Which of the given statement is not correct?

(a) 1 only (b) 2 only


(c) All of the above (d) None of the above

Answer is (b)

Sultan Mahmud of Ghazni, Afghanistan. He ruled from 997 to 1030, and extended control over parts
of Central Asia, Iran and the north-western part of the subcontinent. He raided the subcontinent
almost every year – his targets were wealthy temples, including that of Somnath, Gujarat. The best-
known Chahamana ruler was Prithviraja III (1168-1192), who defeated an Afghan ruler named
Sultan Muhammad Ghori in 1191, but lost to him the very next year, in 1192.

Q9.Vetti and Kadamai, found in inscriptions of cholas are a kind of?


a. Tax b. Tamil literature
c. Schools for Brahmanas d. Donation given by kings to Brahmanas

Answer is (a)
The inscriptions of the Cholas who ruled in Tamil Nadu refer to more than 400 terms for different
kinds of taxes. The most frequently mentioned tax is vetti, taken not in cash but in the form of forced
labour, and kadamai, or land revenue. There were also taxes on thatching the house, the use of a
ladder to climb palm trees, a cess on succession to family property, etc.

Q10. Consider the following statement regarding Nagabhata


1. He defeated the king of Vanga, Vatsa and Matsya but got defeated by combined forces of Arabs
and Turks
2. He belonged to Gurjara-Pratiharas dynasty.
3. His prashasti was written in Sanskrit, found in Kannauj, the capital of his dynasty.

Which of the given statement is/are correct?


(a) 1 only (b) 2 and 3 only
(c) 2 only (d) 1 and 3 only

Answer is (c)

Nagabhata I (r. c. 730-760 CE) founded the imperial Gurjara Pratihara dynasty. He defeated the king
of Vanga, Vatsa and Matsya. He also defeated combined forces of Arabs and Turks. This arab
invasion was led by Junayd ibn Abd al-Rahman al-Murri, a general and governor of Sindh under
the Umayyad caliph Hisham ibn Abd al-Malik. After the death of Dharampala, Nagabhata II regained
hold over Kannuaj and made it the capital of the Gurjara Pratihara Empire. But prashasti dedicated to
him was found in Gwalior, MP.

Google it:- https://upscpdf.com


www.achieveias.co.in, YouTube Channel: http://youtube.com/c/AchieveIAS Telegram Channel: http://t.me/Achieve_Ias, Mail: achieveias21@gmail.com, Contact
Number: 8968920720

https://upscpdf.com https://upscpdf.com https://upscpdf.com


ACHIEVE IAS HISTORY MCQ SERIES, DAY 6, SOLUTIONS
Q1. Consider the following statement regarding Akbar Nama:
1. Written in three volumes by Abul Fazl.
2. The first two volumes dealt with Akbar’s ancestors.
3. Its third volume,Ain-I-Akbari deals with Akbar’s administration and other aspects also.

Which among them is correct regarding Akbar Nama?


(a) 2 Only (b) 1 and 2 only
(c) 1 and 3only (d) None of the above.

Answer: C

Akbar ordered one of his close friends and courtiers, Abul Fazl, to write a history of his reign. Abul
Fazl wrote a three volume history of Akbar’s reign titled, Akbar Nama. The first volume dealt with
Akbar’s ancestors and the second volume recorded the events of Akbar’s reign. The third volume is
the Ain-I-Akbari. It deals with Akbar’s administration, household, army, the revenues and geography
of his empire. It also provides rich details about the traditions and culture of the people living in India.
The most interesting aspect about the Ain-i Akbari is its rich statistical details about things as diverse
as crops, yields, prices, wages and revenues.

Q2. Consider the following statement:


1. In 1527 Babur defeated the Rajputs at Chanderi and established control over Agra and Delhi.
2. In 1526 Babur defeated Rana Sanga, Rajput rulers and allies at Khanua.

Which of the following statement is correct in context of Mughal history?


(a) 1 only (b) 2 only
(c) All of the above (d) None of the above.

Answer: D

BABUR 1526-1530

1526 – Defeated Ibrahim Lodi and his Afghan supporters at Panipat.


1527 – Defeated Rana Sanga, Rajput rulers and allies at Khanua.
1528 – Defeated the Rajputs at Chanderi. Established control over Agra and Delhi before his death.

Q3: With reference to the history of mughals, consider the following:


1. Bakhshi - Military paymaster
2. Faujdars – Military commanders
3. Sadr - Town police commander

Which among these are correctly matched?


(a) 2 and 3 only (b) 1 and 2 only
(c) All of the above (d) None of the above

Google it:- https://upscpdf.com


www.achieveias.co.in, YouTube Channel: http://youtube.com/c/AchieveIAS Telegram Channel: http://t.me/Achieve_Ias, Mail: achieveias21@gmail.com, Contact
Number: 8968920720

https://upscpdf.com https://upscpdf.com https://upscpdf.com


Answer: B

For the maintenance of peace and order in his province, the subadar was supported by other officers
such as the military paymaster (bakhshi), the minister in charge of religious and charitable patronage
(sadr), military commanders (faujdars) and the town police commander (kotwal).

Q4. Which of the following statement is correct in context of Mughal history?


a. First discussion in ibadat khana took place at Agra in 1570s.
b. Literal meaning of sulh-i kul was “universal religion”.
c. Jahangir was the first to frame a vision of governance around this idea of sulh-i kul.
d. Principles of sulh-i-kul was followed by Shah Jahan as well.

Answer: D

While Akbar was at Fatehpur Sikri during the 1570s he started discussions on religion with the
ulama, Brahmanas, Jesuit priests who were Roman Catholics, and Zoroastrians. These discussions
took place in the Ibadat Khana. Akbar subscribed to the idea of Sulh-i Kul or “universal peace”.
Abul Fazl helped Akbar in framing a vision of governance around this idea of Sulh-i Kul. This
principle of governance was followed by Jahangir and Shah Jahan as well.

Q5. In context of Mughal economy, consider the following statement:


1. International travellers described it as the deride land of wealth due to poverty.
2. Income inequality was very high in this period.

Which of the given statement is correct?


(a) 1 only (b) 2 only
(c) All of the above (d) None of the above

Answer: B

The administrative and military efficiency of the Mughal Empire led to great economic and commercial
prosperity. International travellers described it as the fabled land of wealth. A mere 5.6 per cent of
the total number of mansabdars – received 61.5 per cent of the total estimated revenue of the empire
as salaries for themselves and their troopers. This shows that Income inequality was very high in that
period

Q6. With reference to Mughals, which of the following statement is not correct:
a. Mansab was a grading system which determined military responsibilities.
b. Sawar were the cavalrymen maintained by the mansabdar.
c. Rank and salary were determined by a numerical value called zat.
d. By Aurangzeb’s reign the number of mansabdars declined due to his intolerance.

Answer: D

By Aurangzeb’s reign the actual revenue collected was often less than the granted sum. There was
also a huge increase in the number of mansabdars which meant a long wait before they received
Google it:- https://upscpdf.com
www.achieveias.co.in, YouTube Channel: http://youtube.com/c/AchieveIAS Telegram Channel: http://t.me/Achieve_Ias, Mail: achieveias21@gmail.com, Contact
Number: 8968920720

https://upscpdf.com https://upscpdf.com https://upscpdf.com


a jagir. These and other factors created a shortage in the number of jagirs. The term mansabdar
refers to an individual who holds a mansab, meaning a position or rank. It was a grading system used
by the Mughals to fix
(1) rank,
(2) salary and
(3) military responsibilities.

Rank and salary were determined by a numerical value called zat. The higher the zat, the more
prestigious was the noble’s position in court and the larger his salary. The mansabdar’s military
responsibilities required him to maintain a specified number of sawar or cavalrymen. The mansabdar
brought his cavalrymen for review, got them registered, their horses branded and then received
money to pay them as salary.

Q7. Consider the following statement:


1. The Sisodiya Rajputs were defeated by Mughals and their empire was annexed.
2. The Mughals believed in the rule of primogeniture.

Which of the given statement is correct?


(a) 1 only (b) 2 only
(c) All of the above (d) None of the above

Answer: D

The Mughals did not believe in the rule of primogeniture, where the eldest son inherited his father’s
estate. Instead they followed the Mughal and Timurid custom of coparcenary inheritance, or a division
of the inheritance amongst all the sons. The Sisodiya Rajputs refused to accept Mughal authority for
a long time. Once defeated, however, they were honourably treated by the Mughals, given their lands
(watan) back as assignments (watan jagir). The Sisodias were the last Rajput dynasty to form an
alliance with the Mughals, and unlike other Rajput clans, never intermarried with the Mughal imperial
family.

Q8. Consider the following statement:


1. Todar Mal was Akbar’s revenue minister.
2. Four revenue circles was created under Zabt system
3. Tax was fixed on each crop and was collected in medium production.

Which of the given statement is correct?


(a) 1 only (b) 2 only (c) 3 only (d) All of the above

Answer is (a)

Akbar’s revenue minister, Todar Mal, carried out a careful survey of crop yields, prices and areas
cultivated for a ten-year period, 1570-1580. On the basis of this data, tax was fixed on each crop in
cash. Each province was divided into revenue circles with its own schedule of revenue rates for
individual crops. This revenue system was known as zabt.

Q9. Who was not a contemporary of Akbar?


Google it:- https://upscpdf.com
www.achieveias.co.in, YouTube Channel: http://youtube.com/c/AchieveIAS Telegram Channel: http://t.me/Achieve_Ias, Mail: achieveias21@gmail.com, Contact
Number: 8968920720

https://upscpdf.com https://upscpdf.com https://upscpdf.com


a. Russian ruler, Czar Ivan VI
b. Safavid ruler of Iran, Shah Abbas
c. the ruler of Ottoman Turkey, Sultan Suleyman
d. Queen Elizabeth I of England

Answer is (a)

Russian ruler, Czar Ivan VI ruled between 1740-1741 Akbar’s contemporaries- the ruler of Ottoman
Turkey, Sultan Suleyman, also known as “al Qanuni” or the lawgiver (1520 1566); the Safavid ruler of
Iran, Shah Abbas (1588 1629); and the more controversial Russian ruler, Czar Ivan IV Vasilyevich,
also called “Ivan the terrible” (1530 1584), Queen Elizabeth I (1558 1603) of England.

Q10. In context of major campaigns by Mughals, consider the following:


1. Akbar - Golcunda
2. Shah Jahan - Qandahar
3. Aurangzeb - Ahmadnagar

Which among these are correctly matched?


(a) 2 and 3 only (b) 1 and 2 only
(c) All of the above (d) None of the above

Answer is (d)

1. Akbar - Qandahar was seized from the Safavids,


2. Shah Jahan - Ahmadnagar was annexed and the Bijapur forces sued for peace.
3. Aurangzeb - Bijapur was annexed in 1685 and Golcunda in 1687.

Google it:- https://upscpdf.com


www.achieveias.co.in, YouTube Channel: http://youtube.com/c/AchieveIAS Telegram Channel: http://t.me/Achieve_Ias, Mail:
achieveias21@gmail.com, Contact Number: 8968920720
https://upscpdf.com https://upscpdf.com https://upscpdf.com
ACHIEVE IAS HISTORY MCQ SERIES, DAY 7, SOLUTIONS
Q1. Which among them is not correct regarding developments in medieval architecture?
a. Cross fertilisation of artistic forms and architectural styles.
b. Use of Intersecting arches to make high ceiling roof.
c. Arcuate was used to support weight of windows with geometrical designs.
d. Limestone cement was increasingly used in construction.

Answer: C
Interior of temple of Govind Deva in Vrindavan, 1590. The temple was constructed out of red
sandstone and intersecting arches were used that made the high ceiling roof. The creation of large
empires that brought different regions under their rule helped in this cross fertilisation of artistic forms
and architectural styles. Limestone cement was increasingly used in construction. This was very high
quality cement, which, when mixed with stone chips hardened into concrete. The weight of the
superstructure above the doors and windows was sometimes carried by arches. This architectural
form was called “arcuate”.

Q2. Which of the following statement is not correct:


a. The temple was seen as a miniature model of the world ruled by the king and his allies.
b. Muslim Sultans and Padshahs claimed to be incarnations of god.
c. The king took the god’s name because it was auspicious and he wanted to appear like a God.
d. Places of worship provided rulers with the chance to proclaim their close relationship with God.

Answer. B
Muslim Sultans and Padshahs did not claim to be incarnations of god but Persian court chronicles
described the Sultan as the “Shadow of God”.

Q3. Consider the following pairs:


1. Kandariya Mahadeva temple– Dhangadeva
2. Rajarajeshvara temple– Rajaraja I
3. Humayun’s tomb - Akbar

Which among these are correctly matched?


(a) 2 and 3 only (b) 1 and 2 only
(c) All of the above (d) None of the above

Answer: B
1 and 2 are correctly matched. The Humayun’s tomb was commissioned by Humayun's first wife and
chief consort, Empress Bega Begum.

Q4. Which of the following statement is not correct about Qutb Minar?

Google it:- https://upscpdf.com


www.achieveias.co.in, YouTube Channel: http://youtube.com/c/AchieveIAS Telegram Channel: http://t.me/Achieve_Ias, Mail:
achieveias21@gmail.com, Contact Number: 8968920720
https://upscpdf.com https://upscpdf.com https://upscpdf.com
a. Patterns have been created using small arches and geometrical designs.
b. Bands of inscriptions under the balcony in Arabic.
c. Surface of the minar is flat but angular.
d. Use of red sandstone in building structure.

Answer: C
Pattern created under the balcony by the small arches and geometrical designs. Can you find two
bands of inscriptions under the balcony? These are in Arabic. Notice that the surface of the minar is
curved and angular. Placing an inscription on such a surface required great precision. Only the most
skilled craftsperson could perform this task. Remember that very few buildings were made of stone or
brick 800 years ago.

Q5. In context of early Medieval architecture, consider the following statement:


1. Number of windows and doors were reduced.
2. Trabeate style was used in the construction of temples, mosques, tombs.

Which of the given statement is correct?


(a) 1 only (b) 2 only
(c) All of the above (d) None of the above

Answer: B
Between the seventh and tenth centuries architects started adding more rooms, doors and windows
to buildings. Roofs, doors and windows were still made by placing a horizontal beam across two
vertical columns, a style of architecture called “trabeate” or “corbelled”. Between the eighth and
thirteenth centuries the trabeate style was used in the construction of temples, mosques, tombs and
in buildings attached to large stepped-wells (baolis)

Q6. Consider the following statements:


1. Temples were destroyed as these demonstrated other king’s power and wealth.
2. Chola king Rajendra I took away the gold statue of the Buddha when he attacked Sri Lanka.

Which of the given statement is not correct?


a. 1 only b. 2 only
c All of the above d. None of the above

Answer: B

In the early ninth century when the Pandyan king Shrimara Shrivallabha invaded Sri Lanka and
defeated the king, Sena I (831-851), the Buddhist monk and chronicler Dhammakitti noted: “he
removed all the valuables ... The statue of the Buddha made entirely of gold in the Jewel Palace”

Google it:- https://upscpdf.com


www.achieveias.co.in, YouTube Channel: http://youtube.com/c/AchieveIAS Telegram Channel: http://t.me/Achieve_Ias, Mail:
achieveias21@gmail.com, Contact Number: 8968920720
https://upscpdf.com https://upscpdf.com https://upscpdf.com
Q7. Which delhi sultanate ruler described himself as prophet Moses and Solomon?
(a) Alauddin khilji
(b) Qutbuddin Aibak
(c) Iltutmish
(d) Ghiyasuddin Tughluq

Answer: A
Alauddin khilji described himself as Moses and Solomon, the great law-givers of the past. Most of
them used to describe themselves as shadow of God.

Q8. Consider the following statement:


1. Chahar bagh means four gardens,because of their symmetricaldivision into quarters.
2. Charbagh style was brought to India by Mughals.

Which of the given statement is correct?


(a) 1 only (b) 2 only
(c) All of the above (d) None of the above

Answer: C
Chahar bagh means four gardens,because of their symmetricaldivision into quarters.Babur
describedhis interest in planning and layingout formal gardens, placed withinrectangular walled
enclosuresand divided into four quartersby artificial channels. Charbagh style was brought to India by
Mughals. Humayun’s tomb and Taj Mahal in India are the most famous examples of this style.

Q9. Which of the following statement describes ‘Pishtaq’and ‘ Hasht bihisht’ correctly?
a. Pishtaq is central towering dome and Hasht bihisht means “eight paradises”
b. Pishtaq is Tall gateway and Hasht bihisht literally means eight rooms.
c. Pishtaq is tall gateway and Hasht bihisht literally means “eight heavens”
d. Pishtaq is central towering dome and Hasht bihisht is tall gateway.

Answer: C
Tall gateway-pishtaq became important aspects of Mughal architecture, first visible in Humayun’s
tomb. Tradition known as “eight paradises” or hasht bihisht – a central hall surrounded by eight
rooms.

Q10. Which of the following statement is not correct in context of architectural developments
in shah jahan’s reign :
a. The ceremonial halls of public and private audience were described as chihil sutun.
b. Shah Jahan’s audience halls were specially constructed to resemble a mosque.
c. The emperor’s throne was placed on a pedestal.
d. Series of pietra dura inlays were present on the walls of diwan-i khas that depicted the legendary
Greek god Orpheus playing the lute

Google it:- https://upscpdf.com


www.achieveias.co.in, YouTube Channel: http://youtube.com/c/AchieveIAS Telegram Channel: http://t.me/Achieve_Ias, Mail:
achieveias21@gmail.com, Contact Number: 8968920720
https://upscpdf.com https://upscpdf.com https://upscpdf.com
Answer: D

The ceremonial halls of public and private audience (diwan-i khas or am) were carefully planned.
These courts were also described as chihil sutun or forty-pillared halls, placed within a large
courtyard. Shah Jahan’s audience halls were specially constructed to resemble a mosque. The
pedestal on which his throne was placed was frequently described as the qibla. Series of pietra dura
inlays were used behind the emperor’s throne only. It depicted the legendary Greek god Orpheus
playing the lute.

Google it:- https://upscpdf.com


www.achieveias.co.in, YouTube Channel: http://youtube.com/c/AchieveIAS Telegram Channel: http://t.me/Achieve_Ias,
Mail: achieveias21@gmail.com, Contact Number: 8968920720
https://upscpdf.com https://upscpdf.com https://upscpdf.com
ACHIEVE IAS HISTORY MCQ SERIES, DAY 8, SOLUTIONS
Q1. Consider the following statement:
1. The phrase "Fishing in Troubled Waters" was used for Masulipatnam.
2. This city was walled from all sides using interlocking technique.

Which among them is correct?


(a) 1 only (b) 2 only
(c) All of the above (d) None of the above.

Answer: a

Statement 1 is correct. As per description of Masulipatnam by William Methwold, Masulipatnam was


small, populous, unwalled and ill built town. No evidence have been found to prove that it was walled.

Q2. Consider the following statement:


1. Surat city had huge wholesale and retail textile market.
2. Ports in Bombay were developed to support Surat's sea route trade.
3. Surat was a gateway to trade with west and pilgrimage to Mecca

Which of the following statement is not responsible for prosperity of Surat city in medieval era?
(a) 1 only (b) 2 only
(c) 2 and 3 only (d) None of the above.

Answer: B

Development of Bombay was reason for decline of surat. Surat began to decline towards the end of
the seventeenth century because of many factors: the loss of markets and productivity because of the
decline of the Mughal Empire, control of the sea routes by the Portuguese and competition from
Bombay (present-day Mumbai) where the English East India Company shifted its headquarters in 1668.

Q3. Which community was associated with work of goldsmith and bronze Smith in medieval
era?
a. Bidri b. Vishwakarma
c. Saliyar d. Agrahari

Answer: b

The Craftspersons of Bidar were so famed for their inlay work in copper and silver that it came to be
called Bidri. The Panchalas or Vishwakarma community, consisting of goldsmiths, bronzesmiths,
blacksmiths, masons and carpenters, were essential to the building of temples. They also played an
important role in the construction of palaces, big buildings, tanks and reservoirs. Similarly, weavers
such as the Saliyar or Kaikkolars emerged as prosperous communities, making donations to temples.
Some aspects of cloth making like cotton cleaning, spinning and dyeing became specialised and
independent crafts.

Google it:- https://upscpdf.com


www.achieveias.co.in, YouTube Channel: http://youtube.com/c/AchieveIAS Telegram Channel: http://t.me/Achieve_Ias,
Mail: achieveias21@gmail.com, Contact Number: 8968920720
https://upscpdf.com https://upscpdf.com https://upscpdf.com
Q4. Which of the following statement is not correct in context of European effect on Indian
trade?
a. Craftspersons began to work on a system of advances.
b. Weavers no longer had the liberty of selling their own cloth.
c. Decline in the crafts of spinning, weaving, bleaching, dyeing.
d. Indian traders were forced to work as agents of European merchants.

Answer: C

The spurt in demand for goods like textiles led to a great expansion of the crafts of spinning, weaving,
bleaching, dyeing, etc. with more and more people taking them up.
This period saw the decline of the independence of Crafts Persons. They now began to work on a
system of advances which meant that they had to weave cloth which was already promised to European
agents. Weavers no longer had the liberty of selling their own cloth. European Companies used their
naval power to gain control of the sea trade and forced Indian traders to work as their agents.

Q5. Which of the following is the immediate reason responsible for decline of Masulipatnam?
(a) Dominance of Dutch
(b) Policy of Qutb Shahi rulers of Golconda
(c) Poor infrastructure and huge population
(d) Annexation Golconda by Mughal Emperor Aurangzeb

Answer: D

Masulipatnam was chief port of Golconda. When Mughal Emperor Aurangzeb annexed Golconda in
1686-1687, this caused the European Companies to look for alternatives. As the Company traders
moved to Bombay, Calcutta (present-day Kolkata) and Madras (present-day Chennai), Masulipatnam
lost both its merchants and prosperity and declined in the course of the eighteenth century, being today
nothing more than a dilapidated little town.

Q6. Which of the following statement is correct in context of Ahoms?


A. They Rejected the beliefs and notions of Hinduism.
B. They were dependent on paiks for military services.
C. Their Historical works, known as buranjis were also written in Bengali in later period.
D. Brahmanas were not given importance as Ahoms believed in their own traditions.

Answer: B

The Ahom state depended upon forced labour called paiks. The adult males served in the army during
war. At other times, they were engaged in building dams, irrigation systems and other public works.
The Ahoms also introduced new methods of rice cultivation. Ahoms worshipped their own tribal gods.
During the first half of the seventeenth century, however, the influence of Brahmanas increased.
Temples and Brahmanas were granted land by the king. In the reign of Sib Singh (1714-1744),
Hinduism became the predominant religion. But the Ahom kings did not completely give up their
traditional beliefs after adopting Hinduism. Historical works, known as buranjis, were also written – first
in the Ahom language and then in Assamese.

Q7. Which of the given statement is not correct about 11th and 12th century Indian society?
Google it:- https://upscpdf.com
www.achieveias.co.in, YouTube Channel: http://youtube.com/c/AchieveIAS Telegram Channel: http://t.me/Achieve_Ias,
Mail: achieveias21@gmail.com, Contact Number: 8968920720
https://upscpdf.com https://upscpdf.com https://upscpdf.com
(a) Rajput clans became powerful by the eleventh and twelfth centuries.
(b) Dominant tribes of Punjab, Sind and the North-West Frontier joined the lower jatis of caste society.
(c) Specialised artisans were also recognised as separate jatis by the Brahmanas.
(d) Varna system was was taken over by the system of Jatis

Answer: B

Many dominant tribes of Punjab, Sind and the North-West Frontier had adopted Islam. They continued
to reject the caste system. Jatis, rather than varna, became the basis for organising society. Among
the Kshatriyas, new Rajput clans became powerful by the eleventh and twelfth centuries. Specialised
artisans – smiths, carpenters and masons– were also recognised as separate jatis by the Brahmanas.

Q8. With reference to Gonds, which of the given statement is correct?


1. Garhs were divided in 84 villages called barhots.
2. They rejected both the caste system and orthodox Hinduism.

(a) 1 only (b) 2 only


(c) All of the above (d) None of the above

Answer is (d)

Each garh was controlled by a particular Gond clan. This was further divided into units of 84 villages
called Chaurasi. The chaurasi was subdivided into barhots which were made up of 12 villages each.
The emergence of large states changed the nature of Gond society. Their basically equal society
gradually got divided into unequal social classes. Brahmanas received land grants from the Gond rajas
and became more influential. The Gond chiefs now wished to be recognised as Rajputs. Above
statement shows that they later adapted in caste system.

Q9. Which of the following statement is not correct in context of Mughal-tribal relation?
A. Kamal Khan Gakkhar, the cheif of Gakkhars was made a noble (mansabdar) by Emperor Akbar.
B. Rani Durgawati, the queen of Garha Katanga accepted the authority of Mughals.
C. Mir Jumla, the Mughal General defeated the Ahoms in 1662.
D. Raja Man Singh, Akbar’s famous general, attacked and defeated the Cheros in 1591.

Answer: B

In 1565, the Mughal forces under Asaf Khan attacked Garha Katanga. A strong resistance was put up
by Rani Durgawati. She was defeated and preferred to die rather than surrender. Her son, too, died
fighting soon after.

Q10. In context of major tribes of medieval period, consider the following :


1. Khokhar - Punjab
2. Balochis - present day Pakistan
3. Kolis - Chattisgarh

Which among these are correctly matched?


Google it:- https://upscpdf.com
www.achieveias.co.in, YouTube Channel: http://youtube.com/c/AchieveIAS Telegram Channel: http://t.me/Achieve_Ias,
Mail: achieveias21@gmail.com, Contact Number: 8968920720
https://upscpdf.com https://upscpdf.com https://upscpdf.com
(a) 1 and 2 only (b) 2 and 3only
(c) All of the above (d) None of the above

Answer is (a)
1 and 2 are correctly matched. The Maharashtra highlands and Karnataka were home to Kolis, Berads
and numerous others. Kolis also lived in many areas of Gujarat.

Google it:- https://upscpdf.com


www.achieveias.co.in, YouTube Channel: http://youtube.com/c/AchieveIAS Telegram Channel: http://t.me/Achieve_Ias, Mail:
achieveias21@gmail.com, Contact Number: 8968920720
https://upscpdf.com https://upscpdf.com https://upscpdf.com
ACHIEVE IAS HISTORY MCQ SERIES, DAY 9, SOLUTIONS
Q1. Consider the following statement regarding Kabir:
1. Kabir’s teachings was based on tolerance towards major religious traditions.
2. The language of his poetry was a form of spoken Hindi widely understood by ordinary people.

Which among them is correct?


(a) 1only (b) 2 only (c) All of the above. (d) None of the above.

Answer: B
Statement 1 is incorrect. Kabir’s teachings were based on a complete, indeed vehement, rejection of
the major religious traditions. His teachings openly ridiculed all forms of external worship of both
Brahmanical Hinduism and Islam, the pre-eminence of the priestly classes and the caste system. The
language of his poetry was a form of spoken Hindi widely understood by ordinary people. He also
sometimes used cryptic language, which is difficult to follow.

Q2. With reference to late ancient era consider the following statement:
1. Puranic stories and local deities became popular.
2. As per Puranas only Upper Caste devotees can receive the grace of God.

Which of the following statement is not correct?


(a) 1 only (b) 2 only (c) All of the above. (d) None of the above.

Answer. B
Local myths and legends became a part of the Puranic stories, and methods of worship recommended
in the Puranas were introduced into the local cults. Eventually the Puranas also laid down that it was
possible for devotees to receive the grace of God regardless of their caste status.

Q3. Consider the following statements:


1. Virashaiva movement was started by Ramanuja.
2. They were against idol worship.

Which of the following statement is correct?


(a) 1 only (b) 2 only (c) All of the above. (d) None of the above.

Answer: B
Virashaiva movement initiated by Basavanna and his companions like Allama Prabhu and
Akkamahadevi. The Virashaivas argued strongly for the equality of all human beings and against
Brahmanical ideas about caste and the treatment of women. They were also against all forms of ritual
and idol worship.

Q4. Which of the following statement is not correct in context of Nayanars and Alvars?
a. They were saints devoted to shiva and Vishnu
b. They gave equal respect to traditions of other religions.
c. They drew upon the ideals of love and heroism as found in the Sangam literature.
d. They believed in broad based social partition.
Google it:- https://upscpdf.com
www.achieveias.co.in, YouTube Channel: http://youtube.com/c/AchieveIAS Telegram Channel: http://t.me/Achieve_Ias, Mail:
achieveias21@gmail.com, Contact Number: 8968920720
https://upscpdf.com https://upscpdf.com https://upscpdf.com
Answer: B

B is incorrect as they were sharply critical of the Buddhists and Jainas. So they had no regards for their
traditions. They believed in broad based social partition as their members came from all castes
including those considered “untouchable” like the Pulaiyar and the Panars. Nayanars (saints devoted
to Shiva) and Alvars (saints devoted to Vishnu). They drew upon the ideals of love and heroism as
found in the Sangam literature (the earliest example of Tamil literature, composed during the early
centuries of the Common Era) and blended them with the values of bhakti.

Q5. Who among them did not preached in Marathi language?


(a) Namdev (b) Eknath (c) Tukaram (d) Narsi Mehta

Answer: D
Narsi Mehta was a famous Gujarati saint. He did not preached in marathi. Mehta is a pioneer poet of
Gujarati literature.

Q6. Which of the following is true as per Philosophy of Shankara?


A. He was an advocate of dvaita philosophy.
B. He described different attributes of god.
C. According to him, the world is real and God is the creator of this world.
D. According to him knowledge is important to attain salvation.

Answer: D

A and C are part of dvaita philosophy but shankara advocated advaita philosophy. He was an advocate
of Advaita or the doctrine of the oneness of the individual soul and the Supreme God which is the
Ultimate Reality. He taught that Brahman, the only or Ultimate Reality, was formless and without any
attributes. He considered the world around us to be an illusion or maya, and preached renunciation
of the world and adoption of the path of knowledge to understand the true nature of Brahman and
attain salvation.

Q7. Consider the following statement:


1. Bandanawaz Gisudaraz of Gulbarga belongs to Qadiri tariqa Sufi order.
2. Mirabai was a disciple of Ravidas.

Which of the following statement is not correct?


(a) 1 only
(b) 2 only
(c) All of the above.
(d) None of the above.

Answer: A.

Google it:- https://upscpdf.com


www.achieveias.co.in, YouTube Channel: http://youtube.com/c/AchieveIAS Telegram Channel: http://t.me/Achieve_Ias, Mail:
achieveias21@gmail.com, Contact Number: 8968920720
https://upscpdf.com https://upscpdf.com https://upscpdf.com
Statement 2 is correct. Bandanawaz Gisudaraz of Gulbarga belongs to Chishti silsila (sufi order) mainly
popular in indian subcontinent. Qadiri tariqa Sufi order, with its many offshoots, is widespread,
particularly in the Arabic-speaking world.

Q8. With reference to Sikhs, consider the following statement?


1. Policies of Mughals was immediate reason for politicisation of Sikhs
2. Gurmukhi was developed by Guru Arjan.

Which of the following statement is correct ?


(a) 1 only (b) 2 only (c) All of the above (d) None of the above

Answer: A
In 1606, Guru Arjan was tortured and killed by the Mughal emperor Jahangir, for refusing to convert to
Islam. His martyrdom is considered a watershed event in the history of Sikhism. After the martyrdom
of Guru Arjan, his son Guru Hargobind at age eleven became the sixth Guru of the Sikhs and Sikhism
dramatically evolved to become a political movement in addition to being religious. So we can conclude
that Policies of Mughals was immediate reason for politicisation of sikhs. Gurmukhi was developed by
Guru Angad.

Q9. Consider the following:


1. Sufis
2. Siddhas
3. Nathpanthis

Rejection of rituals and practice of breath control to train mind was common in:

a. 1 and 2 only b. 2 and 3 only


c. All of the above d. None of the above

Answer: C
The Sufis often rejected the elaborate rituals. Discussion of parables, breath control, etc. under the
guidance of a master or pir was essential for training mind. These were common in Nathpanthis,
Siddhas and Yogis too.

Q10. In context of Sufism, consider the following:


1. Pir - master
2. Sama - dancing
3. Raqs - chanting of a name

Which among these is/are not correctly matched?


(a) 1 only (b) 2 only (c) 2 and 3only (d) None of the above

Answer: C

Google it:- https://upscpdf.com


www.achieveias.co.in, YouTube Channel: http://youtube.com/c/AchieveIAS Telegram Channel: http://t.me/Achieve_Ias, Mail:
achieveias21@gmail.com, Contact Number: 8968920720
https://upscpdf.com https://upscpdf.com https://upscpdf.com
Only 1 is correct. Sufis developed elaborate methods of training using zikr (chanting of a name or
sacred formula), contemplation, sama (singing), raqs (dancing).

Google it:- https://upscpdf.com


www.achieveias.co.in, YouTube Channel: http://youtube.com/c/AchieveIAS Telegram Channel: http://t.me/Achieve_Ias,
Mail: achieveias21@gmail.com, Contact Number: 8968920720
https://upscpdf.com https://upscpdf.com https://upscpdf.com
ACHIEVE IAS HISTORY MCQ SERIES, DAY 10, SOLUTIONS
Q1. Consider the following Statements:
1. Amritsar became the capital of Sikh territory under Maharaja Ranjit Singh in 1799.
2. Sawai Raja Jai Singh of Amber was governor of Gujrat during Mughal rule.

Which among them is correct?


a. 1 Only b. 2 only c. All of the above. d. None of the above.

Answer: D

Both are incorrect. Sawai Raja Jai Singh of Amber was governor of Malwa. Raja Ajit Singh of Jodhpur
held the governorship of Gujarat. Maharaja Ranjit Singh established his capital at Lahore in 1799.

Q2. Consider the following pairs:


1. Sindhias. - Indore
2. Gaekwad. - Baroda
3. Bhonsle. - Nagpur

Which of the following statement is not correctly matched?


(a) 1 only (b) 2 only (c) 3 only (d) None of the above.

Answer. A

Indore was under control of Holkars. Maratha chiefs (sardars) like Sindhia of Gwalior, Gaekwad of
Baroda and Bhonsle of Nagpur had the resources to raise powerful armies.

Q3. Consider the following statement:


1. Nizam of Hyderabad became the overload of entire deccan after invasion of Nadir shah by 1730s.
2. Afghan ruler Ahmad Shah Abdali sacked and plundered the city of Delhi in 1739.

Which of the following statement is correct?


(a) 1 only (b) 2 only (c) All of the above. (d) None of the above.

Answer: D

By the 1730s, the Maratha king was recognised as the overlord of the entire Deccan peninsula. Nadir
Shah, sacked and plundered the city of Delhi in 1739 and took away immense amounts of wealth. This
invasion was followed by a series of plundering raids by the Afghan ruler Ahmad Shah Abdali, who
invaded north India five times between 1748 and 1761.

Q4. The two taxes chauth and sardeshmukhi were introduced by:
a. Sikhs b. Nizam of Hyderabad c. Nawab of Bengal d. Marathas

Answer: D

Google it:- https://upscpdf.com


www.achieveias.co.in, YouTube Channel: http://youtube.com/c/AchieveIAS Telegram Channel: http://t.me/Achieve_Ias,
Mail: achieveias21@gmail.com, Contact Number: 8968920720
https://upscpdf.com https://upscpdf.com https://upscpdf.com
Chauth and Sardeshmukhi were taxes conceived during the times of the Great Maratha Ruler Shivaji
Maharaj. ‘Chauth’ means basically 1/4th i.e 25% of gross revenue or produce to be paid to jagirdars of
Maratha empire from hostile or alien state.

Q5. Who among them was appointed as Governor of Awadh by Mughal Emperor Farrukh Siyar?
a. Alivardi Khan b. Murshid Quli Khan
c. Nizam-ul-Mulk Asaf Jah d. Burhan-ul-Mulk Sa‘adat Khan

Answer: C

Nizam-ul-Mulk Asaf Jah, the founder of Hyderabad state, was one of the most powerful members at
the court of the Mughal Emperor Farrukh Siyar. He was entrusted first with the governorship of Awadh,
and later given charge of the Deccan. Burhan-ul-Mulk Sa‘adat Khan was appointed subadar of Awadh
in 1722 by Md. Shah Rangeela.

Q6. Consider the following Statements:


1. Lilatilakam is a compilation of stories about Rajput heroes in poetic style.
2. The ijaradari System was introduced in Bengal as new administrative machinery for maintaining huge
army.

Which of the following is correct?


(a) 1 only (b) 2 only (c) All of the above. (d) None of the above.

Answer: D

Lilatilakam is a 14th century Sanskrit-language treatise on the grammar and poetics of the
Manipravalam language form, a precursor of the modern Malayalam language spoken in the Kerala
state of India. It has no connection with Rajputs. Ijaradari system was associated with collection of
revenue. According to this the right to collect revenue was given to the highest bidder.

Q7. With reference to 18th century India, consider the following statement:
1. Richest merchants and bankers were gaining a stake in the new political order of states like Awadh
and Bengal.
2. During Alivardi Khan’s reign the rich bankers of Kansari Community became extremely prosperous.

Which of the following statement is not correct?


(a) 1 only (b) 2 only (c) All of the above (d) None of the above.

Answer: B

Statement 2 is incorrect. Kansaris were bell metal workers.Many of the modest brick and terracotta
temples in Bengal were built with the support of several “low” social groups, such as the Kolu (oil
pressers) and the Kansaris. Regional states were emerging relationship with rich bankers and
merchants. These people lent money to revenue farmers, received land as security and collected taxes
from these lands through their own agents. Throughout India the richest merchants and bankers were
gaining a stake in the new political order.

Google it:- https://upscpdf.com


www.achieveias.co.in, YouTube Channel: http://youtube.com/c/AchieveIAS Telegram Channel: http://t.me/Achieve_Ias,
Mail: achieveias21@gmail.com, Contact Number: 8968920720
https://upscpdf.com https://upscpdf.com https://upscpdf.com
Q8. With reference to miniature art, consider the following statement?
1. They are small-sized paintings, generally done in water colour on cloth or paper.
2. Mughal artists developed a bold and intense style of miniature painting called Basohli.

Which of the following statement is correct?


(a) 1 only (b) 2 only (c) All of the above (d) None of the above

Answer: A

By the late seventeenth century Himachal Pradesh had developed a bold and intense style of miniature
painting called Basohli. The most popular text to be painted here was Bhanudatta’s Rasamanjari.
Miniatures (as their very name suggests) are small-sized paintings, generallydone in water colour on
cloth or paper. The earliestminiatures were on palm leaves or wood.

Q9. Consider the following:


1. Sanskrit
2. Persian
3. Magadhi Prakrit

Which of the following has influence in evolution of Bengali language:


a. 1 and 2 only b. 2 and 3 only c. All of the above d. None of the above

Answer: C

The modern Bengali vocabulary contains the vocabulary base from Magadhi Prakrit and Pali, also
tatsamas and reborrowings from Sanskrit and other major borrowings from Persian, Arabic,
Austroasiatic languages and other languages in contact with. Dr. Suniti Kumar Chatterjee referred
Bengali as an “eastern variety of Magadhi Prakrit”. Suniti Kumar Chatterji was an Indian linguist,
educationist and litterateur. He was a recipient of the second-highest Indian civilian honour of Padma
Vibhushan.

Q10. In context of dance forms, consider the following:


1. Bharatanatyam - Andhra Pradesh
2. Kathakali - Karnataka
3. Kuchipudi - Tamil Nadu

Which among these is/are not correctly matched?


(a) 1 only (b) 2 only (c) 2 and 3only (d) All of the above

Answer: D

All are incorrect. Dance forms that are recognised as classical at present are: Bharatanatyam (Tamil
Nadu), Kathakali (Kerala), Odissi (Orissa), Kuchipudi (Andhra Pradesh), Manipuri (Manipur)

Google it:- https://upscpdf.com


www.achieveias.co.in, YouTube Channel: http://youtube.com/c/AchieveIAS Telegram Channel: http://t.me/Achieve_Ias,
Mail: achieveias21@gmail.com, Contact Number: 8968920720
https://upscpdf.com https://upscpdf.com https://upscpdf.com
ACHIEVE IAS HISTORY MCQ SERIES, DAY 11, SOLUTIONS
Q1. Consider the following statement:
1. " A history of British India" was published by James mill in 1817.
2. He was first to divide lndian history into Ancient, Medieval and Modern phase

Which among them is not correct?


(a) 1only (b) 2 only (c) All of the above. (d) None of the above.

Answer: B

James Mill wrote the monumental work History of British India in 1817. He was the first writer to
divide Indian history into three parts: Hindu, Muslim and British He didnt divide lndian history into
Ancient, Medieval and Modern phase.

Q2. Consider the following pairs:


1. Nagpur. - 1853
2. Awadh. - 1855
3. Sambhalpur - 1850

Which of the following is not correctly matched as per year of their annexation by British?
(a) 1 only (b) 2 only (c) 3 only (d) None of the above.

Answer. B

Awadh was annexed in 1856. Rest are correct.

Q3. Consider the following statement:


1. East India Company acquired a charter from ruler of England.
2. Despite of this charter they faced huge competition from English traders in East.

Which of the following statement is/are correct?


(a) 1 only (b) 2 only (c) All of the above. (d) None of the above.

Answer: A

The company received a Royal Charter from Queen Elizabeth I on 31 December 1600, coming
relatively late to trade in the Indies. It granted EIC sole right to trade with East this means no other
English company was allowed to compete them in East.

Q4. Who among the following was defeated in Battle of Buxar, 1764?
a. Mir jafar b. Mir Qasim
c. British d. Alivardi khan

Google it:- https://upscpdf.com


www.achieveias.co.in, YouTube Channel: http://youtube.com/c/AchieveIAS Telegram Channel: http://t.me/Achieve_Ias,
Mail: achieveias21@gmail.com, Contact Number: 8968920720
https://upscpdf.com https://upscpdf.com https://upscpdf.com
Answer: B

Mir Qaim was defeated in Battle of Buxar, 1764. The Battle of Buxar was fought on 22 October 1764,
between the forces under the command of the British East India Company, led by Hector Munro, and
the combined armies of Mir Qasim, the Nawab of Bengal till 1763. After being defeated in 4 battles in
Katwa, Giria and Udaynala, the Nawab of Awadh Shuja-ud-Daula and the Mughal Emperor Shah
Alam II, accompanied by Raja Balwant Singh of Kashi made an alliance with Mir Qasim.The battle
was fought at Buxar. It was a decisive victory for the British East India Company. The war was
brought to an end by the Treaty of Allahabad in 1765.

Q5. Under which Governor General “Policy of Paramountcy" was initiated by British?
(a) Lord Hastings
(b) Warren Hastings
(c) Lord Richard Wellesley
(d) Lord Dalhousie

Answer: A

Lord Hastings was appointed as the Governor-General of India, on 11 November 1813.Under him
"Policy of Paramountcy" was initiated by British.

Q6. Consider the following Statements:


1. British government official documents helped us to know the mindset of general public in colonial
period.
2. Advent of Europeans led to fall in price of goods that were imported by them.

Which of the following is/are correct?


(a) 1 only (b) 2 only (c) All of the above (d) None of the above.

Answer: D

Both are incorrect. British government official documents did not helps in knowing the mind-set of
general public in colonial period. Advent of Europeans pushed up the price of goods that were
imported by them. This happened due to huge competition between European traders.

Q7. Consider the following statement:


1. Nawab of Awadh lost his half territory in 1801 because of war against Britishers.
2. Aurangzeb abolished the east india company's right to trade duty free.

Which of the following statement is/are incorrect?


(a) 1 only (b) 2 only (c) All of the above. (d) None of the above.

Answer: C

Google it:- https://upscpdf.com


www.achieveias.co.in, YouTube Channel: http://youtube.com/c/AchieveIAS Telegram Channel: http://t.me/Achieve_Ias,
Mail: achieveias21@gmail.com, Contact Number: 8968920720
https://upscpdf.com https://upscpdf.com https://upscpdf.com
Both are incorrect. Aurangzeb issued a farman according to which the east india company was
granted right to trade duty free. Nawab of Awadh was forced to give his half territory to Britishers as
he failed to pay for 'subsidiary forces' Earlier nawab of awadh was forced to sign subsidiary Alliance.

Q8. With reference to Anglo-Mysore war, consider the following statement?


1. English defeated Tipu sultan in the battle of Manglore.
2. After Tipu's defeat, Mysore was placed under Nizam of Hyderabad.

Which of the following statement is correct?


(a) 1 only
(b) 2 only
(c) All of the above
(d) None of the above

Answer: D

English defeated Tipu sultan in the battle of Seringapatnam. After Tipu's defeat, Mysore was placed
under Wodeyars, the former ruling dynasty of Mysore.

Q9. Consider the following in context of East India Company's activities in Bengal :
1. Fortification of factories
2. Revenue dues
3. Political interference

Which of the following is not responsible for fall of Calcutta in Sirajuddaulah's reign?
a. 1 and 2 only
b. 2 and 3 only
c. All of the above
d. None of the above

Answer: D

All are responsible for fall of Calcutta in Sirajuddaulah's reign. When Siraj became Nnawab he asked
East India Company to stop meddling in political affairs, stop fortifications and pay dues. When the
British refused to cease their constructions, the Nawab led a detachment of 3,000 men to surround
the fort and factory of Cossimbazar and took several British officials as prisoners, before moving to
Calcutta. The defences of Calcutta were weak and negligible. The city was occupied on 16 June by
Siraj's force and the fort surrendered after a brief siege on 20 June.

Q10. In context of early British Administration, consider the following :


1. Qazi - Military commander
2. Faujdari adalat - Criminal court
3. Diwani adalat - Civil court

Which among these is/are not correctly matched?

Google it:- https://upscpdf.com


www.achieveias.co.in, YouTube Channel: http://youtube.com/c/AchieveIAS Telegram Channel: http://t.me/Achieve_Ias,
Mail: achieveias21@gmail.com, Contact Number: 8968920720
https://upscpdf.com https://upscpdf.com https://upscpdf.com
(a) 1 only
(b) 2 only
(c) 2 and 3only
(d) All of the above

Answer: A

Only 1 is incorrect. A Qazi is not a Military commander. He is a magistrate or judge of a Sharia court,
who also exercises extrajudicial functions, such as mediation, guardianship over orphans and minors,
and supervision and auditing of public works.

Google it:- https://upscpdf.com


www.achieveias.co.in, YouTube Channel: http://youtube.com/c/AchieveIAS Telegram Channel: http://t.me/Achieve_Ias,
Mail: achieveias21@gmail.com, Contact Number: 8968920720
https://upscpdf.com https://upscpdf.com https://upscpdf.com
ACHIEVE IAS HISTORY MCQ SERIES, DAY 12, SOLUTIONS

Q1. Consider the following statement in context of Blue Rebellion:


1. Local zamindars supported the ryots in rebellion.
2. Zamindars were annoyed with the increasing power of the planters

Which among them is not correct?


(a) 1only (b) 2 only (c) All of the above. (d) None of the above.

Answer: D
All are correct. In many villages, headmen who had been forced to sign indigo contracts, mobilised
the indigo peasants and fought pitched battles with the lathiyals. In other places even the zamindars
went around villages urging the ryots to resist the planters. These zamindars were unhappy with the
increasing power of the planters and angry at being forced by the planters to give them land on long
leases.

Q2. Consider the following pairs:


1. Jute - Bengal
2. Tea - Assam
3. Sugarcane - Maharashtra
4. Rice - Punjab

Which of the following is correctly matched as per crop produced in India for Europeans?
(a) 1 only (b) 3 and 4 only (c) 1 and 2 only (d) None of the above.

Answer: C
Jute in Bengal, tea in Assam, sugarcane in the United Provinces (now Uttar Pradesh), wheat in
Punjab, cotton in Maharashtra and Punjab, rice in Madras.

Q3. Consider the following statement with reference of Permanent Settlement.


1. It was introduced by British in 1793.
2. Revenue was collected without involvement of intermediaries.

Which of the following statement is correct?


(a) 1 only (b) 2 only (c) All of the above. (d) None of the above.

Answer: A
The Company introduced the Permanent Settlement in 1793. By the terms of the settlement, the rajas
and taluqdars were recognised as zamindars. They were asked to collect rent from the peasants and
pay revenue to the Company. So there was involvement of intermediaries.

Q4. Why Mughal Empire appointed East India Company as Diwan of Bengal in 1765?
(connect the dots from chapter 2)
a) Because native rulers got defeated by East India Company in battle of Buxar.

Google it:- https://upscpdf.com


www.achieveias.co.in, YouTube Channel: http://youtube.com/c/AchieveIAS Telegram Channel: http://t.me/Achieve_Ias,
Mail: achieveias21@gmail.com, Contact Number: 8968920720
https://upscpdf.com https://upscpdf.com https://upscpdf.com
b) Because Mir Qasim rebelled against Mughals.
c) Because East India Company was loyal to Mughals.
d) Because they introduced an effective revenue system.

Answer: A

Shah Alam II was the Mughal emperor who appointed the East India company as the diwan of Bengal
on 12 August , 1765. This was done as a part of the "Treaty of Allahabad". This treaty was signed
after the combined forces of "Shah Alam II", "Nawab of Bengal" and "Nawab of Awadh" were
defeated in the Battle of Buxar. As per the treaty, the "East India Company" got the "rights to collect
taxes" from the "east province of Bengal

Q5. Which of the following is not correct about Mahalwari Settlement?


(a) Under this settlement villages were organized as Mahals.
(b) It was devised by Warren Hastings.
(c) Revenue was not fixed
(d) It was implemented in North Western Provinces of Bengal.

Answer: B
Holt Mackenzie devised the new system known as Mahalwari Settlement which came into effect in
1822. Rest are correct

Q6. Consider the following Statements:


1. Price of Indian indigo was low in European market.
2. Woad plant was expensive in Europe as it was rarely available there.
3. Dye made from woad plant was of better quality as compared to Indian indigo.

Which of the following is/are correct?


(a) 2 and 3only
(b) 1 and 2 only
(c) All of the above.
(d) None of the above.

Answer: D

All are incorrect. Price of Indian indigo in the European market was very high. Woad plants were also
used to make violet and blue dyes. Being a plant of the temperate zones, it was more easily available
in Europe. Cloth dyers preferred indigo as a dye because indigo produced a rich blue colour, whereas
the dye from woad was pale and dull.

Q7. Which of the following is correct reason for shift of indigo plantation to Bihar?
(a) Availability of cheap labourers. (b) Optimum weather conditions.
(c) Soil fertilty was much better. (d) Due to rebellion by Bengal ryots.

Google it:- https://upscpdf.com


www.achieveias.co.in, YouTube Channel: http://youtube.com/c/AchieveIAS Telegram Channel: http://t.me/Achieve_Ias,
Mail: achieveias21@gmail.com, Contact Number: 8968920720
https://upscpdf.com https://upscpdf.com https://upscpdf.com
Answer: D
In March 1859 thousands of ryots in Bengal refused to grow indigo. As the rebellion spread, ryots
refused to pay rents to the planters, and attacked indigo factories armed with swords and spears,
bows and arrows. Women turned up to fight with pots, pans and kitchen implements. After the revolt,
indigo production collapsed in Bengal. But the planters now shifted their operation to Bihar.

Q8. Consider the following statement.


1. Bengal dominated the world in terms of indigo cultivation in 18th century.
2. Under nij system planters forced cultivators to sign an agreement.

Which of the following statement is not correct?


(a) 1 only (b) 2 only (c) Both (d) None of the above

Answer: B
From the last decades of the eighteenth century Indigo Cultivation in Bengal expanded rapidly and
Bengal indigo came to dominate the world market. In 1788 only about 30 per cent of the indigo
imported into Britain was from India. By 1810, the proportion had gone up to 95 per cent. There were
two main systems of indigo cultivation – nij and ryoti. Within the system of nij cultivation, the planter
produced indigo in lands that he directly controlled. He either bought the land or rented it from other
zamindars and produced indigo by directly employing hired labourers. Under ryot system, planters
forced cultivators to sign an agreement.

Q9. Champaran Satyagraha (1917) of Mahatama Gandhi was associated with:


a. Mill workers. b. Indigo cultivators.
c. Opium cultivators. d. Traders.

Answer: B
When Mahatma Gandhi returned from South Africa, a peasant from Bihar persuaded him to visit
Champaran and see the plight of the indigo cultivators there. Mahatma Gandhi’s visit in 1917
marked the beginning of the Champaran movement against the indigo planters.

Q10. In context of plantation in early modern phase, consider the following:


1. Satta - Advance payment
2. Gomasthas - Agents of planters
3. Mahal - Group of farmers.

Which among these is/are correctly matched?


(a) 1 only (b) 2 only (c) 2 and 3only (d) 3 only

Answer: B
Satta was a contract or an agreement which the planters forced the ryots to sign. Those who signed
the contract got cash advances from the planters at low rates of interest to produce indigo.
Gomasthas were the agents of planters who collect rent. As per Mahalwari settlement, Mahals refers
to villages.

Google it:- https://upscpdf.com


www.achieveias.co.in, YouTube Channel: http://youtube.com/c/AchieveIAS Telegram Channel: http://t.me/Achieve_Ias,
Mail: achieveias21@gmail.com, Contact Number: 8968920720
https://upscpdf.com https://upscpdf.com https://upscpdf.com
ACHIEVE IAS HISTORY MCQ SERIES, DAY 13, SOLUTIONS

Q1. With reference to Jhum cultivation, Consider the following statements :


1. Jhum cultivation was done on large farms.
2. Ashes of crops were used as fertilisers.

Which of the following is/are correct?


a. 1 Only b. 2 only c. All of the above d. None of the above.

Answer: B

Jhum cultivation also known shifting cultivation was done on small patches of land. They spread the
ash from the firing, which contained potash to fertilise the soil.

Q2. Consider the following pairs:


1. Gujjars - Punjab
2. Labadis - Rajasthan
3. Gaddis - Andhra Pradesh
4. Bakarwals - Kashmir

Which of the following is not correctly matched?


a. 1 only b. 1 and 4 only c. 2 and 3 only d. None of the above.

Answer: C

The Van Gujjars of the Punjab hills and the Labadis of Andhra Pradesh were cattle herders, the Gaddis
of Kulu were shepherds, and the Bakarwals of Kashmir reared Goats.

Q3. Consider the following statement with reference to Baigas Tribe.


1. They often needed to buy and sell in order to be able to get the goods that were not produced within
the locality.
2. In search of work many of them became labourers.

Which of the following statement is not correct?


a. 1 only b. 2 only c. All of the above. d. None of the above.

Answer: C

All are incorrect. The Baigas saw themselves as people of the forest who could only live on the produce
of the forest. It was below the dignity of a Baiga to become a labourer. They did not prefer to do trade
outside their community.

Q4. Which of the following is not correct in context of reason for Birsa movement?
a. Land policies of Britishers.
b. Suppressions by moneylenders.
c. Forceful recruitment in army.
d. Activities of missionaries.

Google it:- https://upscpdf.com


www.achieveias.co.in, YouTube Channel: http://youtube.com/c/AchieveIAS Telegram Channel: http://t.me/Achieve_Ias,
Mail: achieveias21@gmail.com, Contact Number: 8968920720
https://upscpdf.com https://upscpdf.com https://upscpdf.com
Answer: C

The Birsa movement identified all these forces as the cause of the misery the Mundas were suffering.
The land policies of the British were destroying. Their traditional land system, Hindu landlords and
Moneylenders were taking over their land, and Missionaries were criticising their traditional culture.

Q5. Which of the following tribe belonged to Orissa and they were mainly hunters and
gatherers?
a. Khonds b. Santhals c. Khasis d. Kolis

Answer: A

The Khonds were such a community living in the forests of Orissa. They regularly went out on collective
hunts and then divided the meat amongst themselves.

Q6. Consider the following Statements in context of condition of tribal chiefs in British Rule.
1. They lost their rights on land.
2. They were forced to follow laws made by British officials in India.

Which of the following is/are correct?


a. 1 only b. Both are correct c. 2 only d. None of the above.

Answer: C

Tribal chiefs were important people in many areas. They enjoyed a certain amount of Economic power
and were allowed to keep their land titles. But they lost much of their administrative power and were
forced to follow laws made by British officials in India.

Q7. Revolt of Songram Sangma, 1906 took place in which of the following region?
a. Central Provinces b. Assam
c. Jharkhand d. Orissa

Answer: B

The revolt of Songram Sangma in 1906 in Assam, and the forest Satyagraha of the 1930s in the Central
Provinces.

Q8. Consider the following statement.


1. Santhals rose in revolt in 1855.
2. Warli Revolt took place in Orissa in 1940.

Which of the following statement is correct?


a. 1 only b. 2 only c. Both d. None of the above

Answer: A

Santhals rose in revolt in Jharkhand in 1855 and the Warli Revolt in Maharashtra in 1940.
Google it:- https://upscpdf.com
www.achieveias.co.in, YouTube Channel: http://youtube.com/c/AchieveIAS Telegram Channel: http://t.me/Achieve_Ias,
Mail: achieveias21@gmail.com, Contact Number: 8968920720
https://upscpdf.com https://upscpdf.com https://upscpdf.com
Q9. Birsa, a tribesman was considered as God. From which of the following tribe he belonged?
a. Munda b. Santhals c. Ahoms d. Oraons

Answer: A

Birsa himself declared that God had appointed him to save his people from trouble, free them from the
slavery. Soon thousands began following Birsa, believing that he was Bhagwan (God) and had come
to solve all their problems. He was born in a family of Mundas – a tribal group that lived in Chottanagpur.

Q10. Which of the following was correct reason behind the Britishers effort of settling down the
tribes?
1. For easier control and administer.
2. For regular revenue source for the state.

a. 1 only b. 2 only c. Both A and B d. None of these

Answer: C

The British wanted tribal groups to settle down and become peasant cultivators. Settled peasants were
easier to control and administer than people who were always on the move. They also wanted a regular
revenue source for the state.

Google it:- https://upscpdf.com


www.achieveias.co.in, YouTube Channel: http://youtube.com/c/AchieveIAS Telegram Channel: http://t.me/Achieve_Ias,
Mail: achieveias21@gmail.com, Contact Number: 8968920720
https://upscpdf.com https://upscpdf.com https://upscpdf.com
ACHIEVE IAS HISTORY MCQ SERIES, DAY 14, SOLUTIONS

Q1. Consider the following statement in context of 1857 revolit:


1. Revolt of 1857 began at Barrackpore regiment with Mangal Pandey.
2. Local chieftain declared themselves as governor of Mughal Authority.
3. Introduction of new riffle and its cartridge rose the discontentment of Sepoy against Britishers.

Which among them is correct?


(a) 1 and 2only (b) 1 and 3only (c) All of the above. (d) 2 and 3only

Answer: D

Sepoys mutinied in several places beginning from Meerut. On 29 March 1857, a young soldier, Mangal
Pandey, was hanged to death for attacking his officers in Barrackpore. Most Smaller rulers and
chieftains controlled different territories on behalf of the Mughal ruler. Threatened by the expansion of
British rule, many of them felt that if the Mughal emperor could rule again, they too would be able to
rule their own territories once more, under Mughal authority. Sepoys of the regiment at Meerut refused
to do the army drill using the new cartridges which were suspected of being coated with the fat of cows
and pigs.

Q2. Consider the following pairs:


1. Jhansi - Rani Laxmibai
2. Lucknow - Khan Bahadur
3. Bihar - Kunwar Singh
4. Faizabad - Bakht Khan

Which of the following are correctly matched with reference to storm Centres of 1857 revolt.
(a) 1 only (b) 1 and 3 only (c) 2 and 4 only (d) All of the above.

Answer: B

1 and 3 are correct. In Lucknow, Birjis Qadr, the son of the deposed Nawab Wajid Ali Shah, was
proclaimed the new Nawab. In Faizabad, Ahmadullah Shah a maulvi was proclaimed as new leaders.

Q3. Consider the following statement with reference of Aftermath of 1857 revolt?
1. British crown was assumed as direct responsible for Indian Administration.
2. Policy of annexation ended.
3. Land of Zamindars and Landlords were confiscated on large scale.

Which of the following statement is correct?


(a) 1 and 2 only (b) 2 and 3 only (c) All of the above. (d) 1 and 3 only

Answer: A

Google it:- https://upscpdf.com


www.achieveias.co.in, YouTube Channel: http://youtube.com/c/AchieveIAS Telegram Channel: http://t.me/Achieve_Ias,
Mail: achieveias21@gmail.com, Contact Number: 8968920720
https://upscpdf.com https://upscpdf.com https://upscpdf.com
1 and 2 are correct. Land and property of Muslims was confiscated on a large scale. Policies were
made to protect landlords and zamindars and give them security of rights over their lands.

Q4. On what grounds Awadh was annexed by Britishers?


a) On grounds of Civil War.
b) Nawab died without a male Hier.
c) Conspiracy against British government.
d) On grounds of maladministration

Answer: D

Awadh was one of the last territories to be annexed. In 1801, a subsidiary alliance was imposed on
Awadh and in 1856 it was taken over. Governor General Dalhousie declared that the territory was being
misgoverned and British rule was needed to ensure proper administration.

Q5. Who declared Bahadur Shah Zafar I I as the last Mughal Emperor?
(a) Lord Dalhousie (b) Lord Canning.
(c) Lord Hardinge (d) British Monarch

Answer: B

In 1856, Governor-General Canning decided that Bahadur Shah Zafar would be the Mughal king.

Q6. Consider the following Statements:


1. Secretary of state was appointed as personal representative of the British Crown.
2. Sovreignty of ruling cheif of India was returned to them after 1857.

Which of the following is/are not correct?


(a) 1 only (b) 2 only (c) All of the above. (d) None of the above.

Answer: C

Secretary of State for India and made responsible for all matters related to the governance of India. He
was member of British cabinet. He was given a council to advise him, called the India Council.
The Governor-General of India was given the title of Viceroy, that is, a personal representative of the
Crown. The local ruling chief were allowed to pass on their kingdoms to their heirs, including adopted
sons. However, they were made to acknowledge the British Queen as their Sovereign Paramount. Thus
the Indian rulers were to hold their kingdoms as subordinates of the British Crown.

Q7. Lahore Gate Improvement Scheme was introduced in?


(a) 1888 (b) 1936 (c) 1886 (d) 1911

Answer: A

Google it:- https://upscpdf.com


www.achieveias.co.in, YouTube Channel: http://youtube.com/c/AchieveIAS Telegram Channel: http://t.me/Achieve_Ias,
Mail: achieveias21@gmail.com, Contact Number: 8968920720
https://upscpdf.com https://upscpdf.com https://upscpdf.com
In 1888 an extension scheme called the Lahore Gate Improvement Scheme was planned by
Robert Clarke for the Walled City residents

Q8. Consider the following statement.


1. Delhi was made an important presidency in late 18th century.
2. Surat witnessed fast urbanization during 19th century.

Which of the following statement is not correct?


(a) 1 only (b) 2 only (c) Both (d) none of the above

Answer: C

Delhi was not a presidency in late 18th century. In the late eighteenth century, Calcutta, Bombay and
Madras rose in importance as Presidency cities. Cities such as Machlipatnam, Surat and Seringapatam
were deurbanised during the nineteenth century.

Q9. Which of the following is not a correct reason for discontentment of Sepoy against
Britishers?
a. Overseas Service b. Low pay and allowances.
c. Violation of regious beliefs. d. Annexation of Satarna.

Answer: D

The Indian sepoys in the employ of the Company also had reasons for discontent. They were unhappy
about their pay, allowances and conditions of service. Some of the new rules, moreover, violated their
religious sensibilities and beliefs

Q10. Consider the following statement:


1. In Delhi living spaces of Indians and the British were sharply separated
2. In 1877, Viceroy Lytton organised a Durbar to acknowledge queen Victoria as the Empress of India.
3. In 1877 Delhi was made capital of India.

Which among these is/are correct statement?


(a) 1 only (b) 2 only (c) 2 and 3 only (d) All of them.

Answer: B

In 1911, when King George V was crowned in England, a Durbar was held in Delhi to celebrate the
occasion. The decision to shift the capital of India from Calcutta to Delhi was announced at this Durbar.
In 1877, Viceroy Lytton organised a Durbar to acknowledge Queen Victoria as the Empress of India. In
Madras, Bombay or Calcutta, the living spaces of Indians and the British were sharply separated.

Google it:- https://upscpdf.com


www.achieveias.co.in, YouTube Channel: http://youtube.com/c/AchieveIAS Telegram Channel: http://t.me/Achieve_Ias,
Mail: achieveias21@gmail.com, Contact Number: 8968920720
https://upscpdf.com https://upscpdf.com https://upscpdf.com
ACHIEVE IAS HISTORY MCQ SERIES, DAY 15, SOLUTIONS

Q1. With reference to Wood's Despatch, Consider the following statements:


1. It argued that literature of East India Company would improve the moral character of Indians
2. Education departments of the government was abolished under this system.

Which of the following is/are correct?


(a) 1only (b) 2 only (c) all of the above. (d) None of the above.

Answer: D

Wood’s Despatch argued that European learning would improve the moral character of Indians.
It would make them truthful and honest, and thus supply the Company with civil servants who could
be trusted and depended upon. The literature of the East was not only full of grave errors.
Following the 1854 Despatch, several measures were introduced by the British. Education
departments of the government were set up to extend control over all matters regarding education.

Q2. Consider the following pairs with reference to weaver communities of India:
1. Tanti - Bengal
2. Momin - Hyderabad
3. Devangs - South India

Which of the following is not correctly matched?


(a) 1 only (b) 2 only (c) 3 only (d) None of the above.

Answer: B

The tanti weavers of Bengal, the julahas or momin weavers of north India, sale and kaikollar and
devangs of south India are some of the communities famous for weaving.

Q3. Consider the following Statements:


1. Calico Act was passed by British government to increase duty on import of cotton textiles to
England.
2. When the English East India Company gained political power in Bengal, they used copper coins
instead of silver to buy Indian goods.

Which of the following statement is correct?


(a) 1 only (b) 2 only (c) All of the above. (d) None of the above.

Answer: D

In 1720, the British government enacted a legislation banning the use of printed cotton textiles –
chintz – in England. Interestingly, this Act was known as the Calico Act. When the English East India
Company gained political power in Bengal, it no longer had to import precious metal to buy Indian

Google it:- https://upscpdf.com


www.achieveias.co.in, YouTube Channel: http://youtube.com/c/AchieveIAS Telegram Channel: http://t.me/Achieve_Ias,
Mail: achieveias21@gmail.com, Contact Number: 8968920720
https://upscpdf.com https://upscpdf.com https://upscpdf.com
goods. Instead, they collected revenues from peasants and Zamindars in India, and used this
revenue to buy Indian textiles.

Q4. Word "Calico" coined by Europeans was in general refers to:


a) Cotton textiles b) Silk c) Spices d) Artworks of Calicut

Answer: A

When the Portuguese first came to India in search of spices they landed in Calicut on the Kerala
coast in south-west India. The cotton textiles which they took back to Europe, along with the
spices, came to be called “calico” (derived from Calicut), and subsequently calico became the
general name for all cotton textiles.

Q5. Consider the following pairs:


1. William Carey- Serampore Mission
2. William Jones - Asiatic Society
3. James Mill - An orientalist

Which of the following is correctly matched?


(a) 1 and 2only (b) 2 and 3only (c) 1 and 3 only (d) All of the above.

Answer: A

William Carey was a Scottish missionary who helped establish the Serampore Mission. William Jones
discovered that his interests were shared by many British officials living in Calcutta at the time.
Englishmen like Henry Thomas Colebrooke and Nathaniel Halhed were also busy discovering the
ancient Indian heritage, mastering Indian languages and translating Sanskrit and Persian works into
English. Together with them, Jones set up the Asiatic Society of Bengal, and started a journal called
Asiatic Researches. James Mill was one of those who attacked the Orientalists. The British effort, he
declared, should not be to teach what the natives wanted, or what they respected, in order to please
them and “win a place in their heart”. The aim of education ought to be to teach what was useful and
practical. James mill was not an orientalist.

Q6. Which of the following is correct in context of decline of Indian textiles? Which of the
following is correct in context of decline of Indian textiles?
1. Competition with English made clothes in their traditional markets.
2. By the 1830s British cotton cloth flooded in Indian markets.
3. Handloom weaving completely died in India due to introduction of machine made works from
Britain.

Which of the following is/are correct?


(a) 1 and 2only (b) All are correct (c) 2 and 3only (d) 1 and 3only

Answer: A

Google it:- https://upscpdf.com


www.achieveias.co.in, YouTube Channel: http://youtube.com/c/AchieveIAS Telegram Channel: http://t.me/Achieve_Ias,
Mail: achieveias21@gmail.com, Contact Number: 8968920720
https://upscpdf.com https://upscpdf.com https://upscpdf.com
By the beginning of the nineteenth century, English made cotton textiles successfully ousted Indian
goods from their traditional markets in Africa, America and Europe. By the 1830s British cotton cloth
flooded Indian markets. In fact, by the 1880s two-thirds of all the cotton clothes worn by Indians were
made of cloth produced in Britain. Handloom weaving did not completely die in India. This was
because some types of cloths could not be supplied by machines.

Q7. The view that "Education in English crippled Indians" was held by?
a) Gopal Krishna Gokhale b) Aurobindo Ghosh
c) Mahatama Gandhi d) Bal gangadhar Tilak

Answer: C

Mahatma Gandhi strongly felt that Indian language sought to be the medium of teaching.
Education in English crippled Indians, distanced them from their own social surroundings, and
made them “strangers in their own lands”.

Q8. Consider the following statement.


1. Mahatama Gandhi and Rabindranath Tagore were highly critical of element of western civilization
in Education.
2. First cotton mill in India was set up in Ahmedabad in 1854

Which of the following statement is correct?


(a) 1 only
(b) 2 only
(c) Both
(d) None of the above

Answer: D
Rabindranath Tagore was not critical of element of western civilization in Education.
In many senses Tagore and Mahatma Gandhi thought about education in similar ways. There were,
however, differences too. Gandhiji was highly critical of Western civilisation and its worship of
machines and technology. Tagore wanted to combine elements of modern western civilisation
with what he saw as the best within Indian tradition. He emphasised the need to teach science and
technology at Santiniketan, along with art, music and dance. The first cotton mill in India was set up
as a spinning mill in Bombay in 1854. From the early nineteenth century, Bombay had grown as an
important port for the export of raw cotton from India to England and China.

Q9. According to which personality "Goal of national education was to awaken the spirit of
nationality and education should be imparted in the vernacular language?
a) Rabindranath Tagore b) Aurobindo Ghosh
c) Mahatama Gandhi d) Bal gangadhar Tilak

Google it:- https://upscpdf.com


www.achieveias.co.in, YouTube Channel: http://youtube.com/c/AchieveIAS Telegram Channel: http://t.me/Achieve_Ias,
Mail: achieveias21@gmail.com, Contact Number: 8968920720
https://upscpdf.com https://upscpdf.com https://upscpdf.com
Answer: B
In a speech delivered on January 15, 1908 in Bombay, Aurobindo Ghose stated that the goal of
national education was to awaken the spirit of nationality among the students. This required a
contemplation of the heroic deeds of our ancestors. The education should be imparted in the
vernacular so as to reach the largest number ofpeople.

Q10. Consider the following statements:


1. Henry Thomas Colebrooke was not an orientalist.
2. Wootz Steel was rarely produced in South India.

Which of the following statement is not correct?


(a)1 only (b) 2 only (c) Both a and b (d) None of these

Answer: C

Henry Thomas Colebrooke was a scholar of Sanskrit and ancient sacred writings of
Hinduism.Orientalists are those with a scholarly knowledge of the language and culture of Asia.
The sword of Tipu sultan was incredibly hard and sharp edge that could easily rip through the
opponent's armour. The quality of the sword came from a specialtype of high carbon steel called
Wootz which was produced all over south India.

Google it:- https://upscpdf.com


www.achieveias.co.in, YouTube Channel: http://youtube.com/c/AchieveIAS Telegram Channel: http://t.me/Achieve_Ias,
Mail: achieveias21@gmail.com, Contact Number: 8968920720
https://upscpdf.com https://upscpdf.com https://upscpdf.com
ACHIEVE IAS HISTORY MCQ SERIES, DAY 16, SOLUTIONS

Q1. Consider the following statement.


1. Book 'Stripurushtulna' was published by Pandit Ramabai.
2. Tarabai Shinde founded a widows’ home at Poona to provide shelter to widows.

Which of them is/are correct?


(a) 1 only (b) 2 only (c) Both of them (d) None of them

Answer: D

Tarabai Shinde, a woman educated at home at Poona, published a book, Stripurushtulna, (A


Comparison between Women and Men), criticising the social differences between men and women.
Pandita Ramabai, a great scholar of Sanskrit, felt that Hinduism was oppressive towards women, and
wrote a book about the miserable lives of upper-caste Hindu women. She founded a widows’ home at
Poona to provide shelter to widows who had been treated badly by their husbands’ relatives.

Q2. Painting of Robert Clive and Mir Jafar was painted by?
a) Abanindranath Tagore b) George Willson
c) Rober Kerr Porter d) Francis Hayman

Answer: D

Painting of Robert Clive and Mir Jafar was painted by Francis Hayman in 1762 and placed on public
display in the Vauxhall Gardens in London. The British had just defeated Sirajuddaulah in the famous
Battle of Plassey and installed Mir Jafar as the Nawab ofMurshidabad. It was a victory won through
conspiracy, and the traitor Mir Jafar was awarded the title of Nawab. In the painting by Hayman this
act of aggression and conquest is not depicted. It shows Lord Clive being welcomed by Mir Jafar and
his troops after the Battle of Plassey.

Q3. Consider the following statement in context of Raja Rammohan Roy.


1. He founded a reform association known as the Brahmo Sabha in 1828.
2. He started a movement to bring equality for women and against western education.
3. He is known as father of Indian Renaissance.

Which of the following statement is correct?


(a) 1 and 2 only (b) 2 and 3 only (c) All of the above. (d) 1 and 3 only

Answer: D

Raja Rammohan Roy is known as father of Indian Renaissance. He was supporter of western
education. Raja Rammohan Roy (1772-1833). He founded a reform association known as the
Brahmo Sabha (later known as the Brahmo Samaj) in Calcuttain 1828. Rammohun Roy was keen to
spread the knowledge of Western education in the country and bring about greater freedom and

Google it:- https://upscpdf.com


www.achieveias.co.in, YouTube Channel: http://youtube.com/c/AchieveIAS Telegram Channel: http://t.me/Achieve_Ias,
Mail: achieveias21@gmail.com, Contact Number: 8968920720
https://upscpdf.com https://upscpdf.com https://upscpdf.com
equality for women. He wrote about the way women were forced to bear the burden of domestic work,
confined to the home and the kitchen, and not allowed to move out and become educated.

Q4. Who among the following reinterpreted verses from the Quran to argue for women's
education?
a) Mahmud-ul-hasan b) Mirza Ghulam Ahmad
c) Mumtaz Ali d) Sir Syed Ahmad Khan

Answer: C

Mumtaz Ali reinterpreted verses from the Koran to argue for women’s education. The first Urdu
novels began to be written from the late nineteenth century. Amongst other things, these were meant
to encourage women to read about religion and domestic management in a language they could
understand.

Q5. Which of the following statements is correct about book Ghulamgiri?


(a) Gulamgiri is one of the finest work of Dr. Bhim Rao Ambedkar.
(b) The writer has dedicated this book to Africans.
(c) The aim of the writer was to link condition of 'lower castes' and 'slaves'
(d) Self-respect movement was initiated by the writer of this book.

Answer: C

In 1873, Jyotirao Phule wrote a book named Gulamgiri meaning slavery. Some ten years before this,
the American Civil W ar had been fought, leading to the end of slavery in America. Phule dedicated
his book to all those Americans who had fought to free slaves, thus establishing a link between the
conditions of the “lower” castes in India and the black slaves in America.
The Satyashodhak Samaj was initiated by Jyotirao phule.

Q6. Consider the following Statements:


1. Due to efforts of Ishwarchandra Vidyasagar Hindu Widow's Remarriage Act 1856 was passed.
2. Arya Samaj advocated hindus traditional values and were against widow remarriage.

Which of the following is/are not correct?


(a) 1 only (b) 2 only (c) All of the above (d) None of the above.

Answer: B
Ishwarchandra Vidyasagar used the ancient texts to suggest that widows could remarry. His
suggestion was adopted by British officials, and a law was passed in 1856 permitting widow
remarriage. In the north, Swami Dayanand Saraswati, who founded the reform association called
Arya Samaj, also supported widow remarriage.

Q7. Which of the following organisation or movement was not associated with Dr. B. R. A
mbedkar?

Google it:- https://upscpdf.com


www.achieveias.co.in, YouTube Channel: http://youtube.com/c/AchieveIAS Telegram Channel: http://t.me/Achieve_Ias,
Mail: achieveias21@gmail.com, Contact Number: 8968920720
https://upscpdf.com https://upscpdf.com https://upscpdf.com
(a) Temple entry movement (b) Satya Shodhak Samaj
(c) Independent Labour Party (d) Bahishkrit Hitakarini Sabha

Answer: B

The Satyashodhak Samaj was an association founded with Jyotiba Phule. He founded
Satyashodhak Samaj in 1873. Rest are associated with Dr. BR Ambedkr

Q8. Consider the following statement.


1. Raja Ravi Verma declared that western style of painting was unsuitable for depicting the nation’s
ancient myths and legends.
2. Painting of Famous battle of Polilur was painted by court painter Murshidabad.

Which of the following statement is not correct?


(a) 1 only (b) 2 only (c) Both of them (d) None of the above

Answer: C

Abanindranath Tagore the nephew of Rabindranath Tagore rejected the art of Ravi Varma as
imitative and westernised, and declared that such a style was unsuitable for depicting the nation’s
ancient myths and legends. In Mysore, Tipu Sultan also resisted the cultural traditions associated with
them and had the walls of his palace at Seringapatam covered with Mural paintings done by local
artists such as painting of the famous battle of Polilur of 1780 in which Tipu and Haidar Ali defeated
the English troops.

Q9. Which of the following movement was not associated with anti-caste Struggle?
a) Paramhans Mandli b) Self-respect Movement
c) Bharat Dharma Mahamandal d) Prarthana Samaj

Answer: C

In Bombay, the Paramhans Mandali was founded in 1840 to work for the abolition of caste. Many of
these reformers and members of reform associations were people of upper castes. Often, in secret
meetings, these reformers would violate caste taboos on food and touch, in an effort to get rid of the
hold of caste prejudice in their lives. Periyar founded the Self Respect Movement. He argued that
untouchables were the true upholders of an original Tamil and Dravidian culture which had been
subjugated by Brahmans. Orthodox Hindu society also reacted by founding Sanatan Dharma Sabhas
and the Bharat Dharma Mahamandal in the north, and associations like the Brahman Sabha in
Bengal. The object of these associations was to uphold caste distinctions as a cornerstone of
Hinduism, and show how this was sanctified by scriptures. The Prarthana Samaj was established in
1867 at Bombay, the Prarthana Samaj sought to remove caste restrictions, abolish child marriage,
encourage the education of women, and end the ban on widow remarriage. Its religious meetings
drew upon Hindu, Buddhist and Christian texts.

Q10) Consider the following statement:

Google it:- https://upscpdf.com


www.achieveias.co.in, YouTube Channel: http://youtube.com/c/AchieveIAS Telegram Channel: http://t.me/Achieve_Ias,
Mail: achieveias21@gmail.com, Contact Number: 8968920720
https://upscpdf.com https://upscpdf.com https://upscpdf.com
1. Sir Syed Ahmad Khan was supporter traditional way of religion and was against western
education.
2. Ramakrishna Mission was only limited to religion.

Which among these is/are correct statement?


(a) 1 only (b) 2 only (c) None of them (d) Both of them

Answer: C

The Mohammedan Anglo-Oriental College was founded by Sayyid Ahmed Khan in 1875 at Aligarh,
later became the Aligarh Muslim University. The institution offered modern education, including
Western science, to Muslims. The Aligarh Movement, as it was known, had an enormous impact in
the area of educational reform.

Swami Vivekananda was the first Indian in modern times, who re-established the spiritual pre-
eminence of the Vedanta philosophy on a global scale. But his mission was not simply to talk of
religion. He was extremely pained at the poverty and the misery of his country men.He firmly
believed that any reform could become successful only by uplifting the condition of the
masses. Swami vivekanand was founder of Ramakrishna Mission.

Google it:- https://upscpdf.com


www.achieveias.co.in, YouTube Channel: http://youtube.com/c/AchieveIAS Telegram Channel: http://t.me/Achieve_Ias,
Mail: achieveias21@gmail.com, Contact Number: 8968920720
https://upscpdf.com https://upscpdf.com https://upscpdf.com
ACHIEVE IAS HISTORY MCQ SERIES, DAY 17, SOLUTIONS

Q1. With reference to Partition of Bengal, Consider the following statements:


1. Real motive behind the partition of Bengal was to weaken Bengal, the nerve centre of Indian
nationalism.
2. All India Muslim League supported congress in anti-partition struggle.

Which of the following is/are correct?


(a) 1 only (b) 2 only (c) All of the above (d) None of the above.

Answer: A

Statement 1 is correct. A group of Muslim landlords and nawabs formed the All India Muslim League
at Dacca in 1906. The League supported the partition of Bengal. And they were against the Congress.

Q2. Consider the following pairs:


1. Pact between Congress and the Muslim League - 1915
2. Congress split - 1907
3. Ahmedabad millworkers’ strike - 1917

Which of the following is not correctly matched?


(a) 1 and 2only (b) 2 and 3only (c) 1 and 3 only (d) None of the above.

Answer: C

The two groups of Congress reunited in December 1915. Next year (1916) the Congress and the
Muslim League signed the historic LucknowPact and decided to work together for representative
government in the country. In Ahmedabad Mahatma Gandhi led a successful millworkers’ strike in 1918.

Q3. Consider the following statement:


1. Demand of separate electorates for Muslims was conceded by the British government in 1919.
2. Bal Gangadhar Tilak criticised the Moderates for their“politics of prayers”

Which of the following statement is not correct?


(a) 1 only (b) 2 only (c) All of the above (d) None of the above.

Answer: A

In Bengal, Maharashtra and Punjab, leaders such as Bipin Chandra Pal, Bal Gangadhar Tilak and Lala
Lajpat Rai were beginning to explore more radical objectives and methods. They criticised the
Moderates for their “politics of prayers”. A group of Muslim landlords and nawabs formed the All India
Muslim League at Dacca in 1906. The League supported the partition of Bengal. It desired separate
electorates forMuslims, a demand conceded by the government in1909.

Google it:- https://upscpdf.com


www.achieveias.co.in, YouTube Channel: http://youtube.com/c/AchieveIAS Telegram Channel: http://t.me/Achieve_Ias,
Mail: achieveias21@gmail.com, Contact Number: 8968920720
https://upscpdf.com https://upscpdf.com https://upscpdf.com
Q4. Why Lord Ripon introduced Ilbert Bill:
a) For decentralisation of governance
b) For reforms in education
c) To reduce import duty
d) To ensure common system of Judiciary for both Indian and European convicts.

Answer: D

In 1883, there was a furore over the attempt by the government to introducethe Ilbert Bill. The bill
provided for the trial of Britishor European persons by Indians, and sought equality between British and
Indian judges in the country. This bill was introduced to ensure common system of Judiciary for both
Indian and European convicts.

Q5. Consider the following pairs:


1. Direct Action Day - Congress
2. Bhagat Singh - Hindustan Socialist Republican Association
3. Purna Swaraj - 1929

Which of the following is correctly matched?


(a) 1 and 2only (b) 2 and 3only (c) 1 and 3 only (d) All of the above.

Answer: B
After the failure of the Cabinet Mission, the Muslim League decided on mass agitation for winning its
Pakistan demand. It announced 16 August 1946 as “Direct Action Day”. Revolutionary nationalists
such as Bhagat Singh, Chandra Shekhar Azad, Sukhdev and others wanted to fight against the colonial
rule and the rich exploiting classes through a revolution of workers and peasants. For this purpose they
founded the Hindustan Socialist Republican Association (HSRA) in 1928 at Ferozeshah Kotla in
Delhi. On 17 December, 1928, Bhagat Singh, Azad and Rajguru assassinated Saunders, a police
officer who was involved in the lathi-charge that had caused the death of Lala Lajpat Rai. Congress
resolved to fight for Purna Swaraj (complete independence) in 1929 under the presidentship of
Jawaharlal Nehru. Consequently, “Independence Day” was observed on26 January 1930 all over the
country.

Q6. Who expressed the pain and anger for Jallianwala Bagh atrocities by renouncing his
knighthood?
(a) Rabindranath Tagore
(b) Mahatma Gandhi
(c) Mohammad Ali Jinnah
(d) Vallabhbhai Patel

Answer: A

The Jallianwala Bagh atrocities, inflicted by General Dyer in Amritsar on Baisakhi Day (13 April), were
a part of this repression. On learning about the massacre, Rabindranath Tagore expressed the pain

Google it:- https://upscpdf.com


www.achieveias.co.in, YouTube Channel: http://youtube.com/c/AchieveIAS Telegram Channel: http://t.me/Achieve_Ias,
Mail: achieveias21@gmail.com, Contact Number: 8968920720
https://upscpdf.com https://upscpdf.com https://upscpdf.com
and anger of the country by renouncing his knighthood. Knighthood is an honour granted by the British
Crown for exceptional personal achievement or public service.

Q7. With reference to Indian National Congress which of the following is not correct:
a) Congress was formed by A. O HUME.
b) First session of Congress was held at Bombay.
c) Its first session was attended by 72 delegates.
d) The early leadership of Congress included Mahatma Gandhi, Dadabhai Naoroji, PherozeshahMehta
etc.

Answer: D

Mahatma Gandhi was not among early leaders of Congress. He joined congress in 1920s
Rest are correct.

Q8. Consider the following statement.


1. Potti Sriramulu went on a hunger strike demanding the formation of a Tamil state.
2. In 1956, the Second Five Year Plan was formulated. This focused strongly on the development of
Education and health sector.

Which of the following statement is correct?


(a) 1 only (b) 2 only (c) Both of them (d) None of the above

Answer: D

In October of 1952, a veteran Gandhian named Potti Sriramulu went on a hunger strike demanding the
formation of Andhra state to protect the interests of Telugu speakers. As the fast went on, it attracted
much support. Hartals and bandhs were observed in many towns. In 1956, the Second Five Year Plan
was formulated. This focused strongly on the development of heavy industries such as steel, and on
the building of largedams.

Q9. Which of the following is not among the early demands of Moderates?
a) Repeal of the Arms Act and the freedom of speech and expression.
b) Indianisation civil service examinations
c) Expansion Legislative Councils.
d) Demand for self-government

Answer: D

It was 1906 in Calcutta session that for the first time congress demanded self-government.

Q10. Consider the following statements in context of Post-Independence:


1. Dr B.R. Ambedkar was Chairman of the Drafting Committee in Constituent Assembly.
2. Deputy Prime Minister Vallabhbhai Patel was in support of the creation of linguistic states.

Google it:- https://upscpdf.com


www.achieveias.co.in, YouTube Channel: http://youtube.com/c/AchieveIAS Telegram Channel: http://t.me/Achieve_Ias,
Mail: achieveias21@gmail.com, Contact Number: 8968920720
https://upscpdf.com https://upscpdf.com https://upscpdf.com
Which of the following statement is not correct?
(a) 1 only (b) 2 only (c) Both a and b (d) None of these

Answer: B

Statement 1 is correct. Both Prime Minister Nehru and Deputy Prime Minister Vallabhbhai Patel were
against the creation of linguistic states. After the Partition, Nehru said - “disruptionist tendencies had
come to the fore”; to check them, the nation had to be strong and united.

Google it:- https://upscpdf.com


www.achieveias.co.in, YouTube Channel: http://youtube.com/c/AchieveIAS Telegram Channel: http://t.me/Achieve_Ias,
Mail: achieveias21@gmail.com, Contact Number: 8968920720
https://upscpdf.com https://upscpdf.com https://upscpdf.com
ACHIEVE IAS HISTORY MCQ SERIES, DAY 18, SOLUTIONS

Q1. Consider the following statement in context of Bastar :


1. In 1905 colonial government proposed to stop hunting and collection of forest produce but allowed
shifting agriculture.
2. Dhurwas are not among those communities who were inhabited to Bastar.

Which among them is/are not correct?


(a) 1 only (b) 2 only (c) All of the above (d) None of the above.

Answer: C

A number of different communities live in Bastar such as Maria and Muria Gonds, Dhurwas, Bhatras
and Halbas. They speak different languages but share common customs and beliefs. When the colonial
government proposed to reserve two-thirds of the forest in 1905, and stop shifting cultivation, hunting
and collection of forest produce, the people of Bastar were very worried.

Q2. Consider the following in context of opium cultivation in India:


1. If cultivators planted opium on their land, then pulses could not be grown there.
2. The price the government paid to the cultivators for the opium they produced was very low.

Which of the above is/are not correct?


(a) 1 only (b) 2 only (c) All of the above (d) None of the above.

Answer: D

Both are correct. For a variety of reasons, cultivators were unwilling to turn their fields over to poppy.
First, the crop had to be grown on the best land, on fields that lay near villages and were well manured.
On this land peasants usually produced pulses. If they planted opium on this land, then pulses could
not be grown there. The price the government paid to the cultivators for the opium they produced was
very low. It was unprofitable for cultivators to grow opium at that price.

Q3. With reference to colonial rule and pastrol life, consider the following statement:
1. Colonial state wanted to transform all grazing lands into cultivated farms.
2. However under Waste Land Rules uncultivated lands were taken over and given to pastoralists.

Which of the following statement is correct?


(a) 1 only (b) 2 only (c) All of the above (d) None of the above.

Answer: A

Under colonial rule, the life of pastoralists changed dramatically. First, the colonial state wanted to
transform all grazing lands into cultivated farms. Land revenue was one of the main sources of its
finance. By expanding cultivation it could increase its revenue collection. Then From the mid-nineteenth
century, Waste Land Rules were enacted in various parts of the country. By these Rules uncultivated
Google it:- https://upscpdf.com
www.achieveias.co.in, YouTube Channel: http://youtube.com/c/AchieveIAS Telegram Channel: http://t.me/Achieve_Ias,
Mail: achieveias21@gmail.com, Contact Number: 8968920720
https://upscpdf.com https://upscpdf.com https://upscpdf.com
lands were taken over and given to select individuals. These individuals were granted various
concessions and encouraged to settle these lands. Some of them were made headmen of villages in
the newly cleared areas. In most areas the lands taken over were actually grazing tracts used regularly
by pastoralists. So expansion of cultivation inevitably meant the decline of pastures and a problem for
pastoralists.

Q4. Like many nomads Gaddi shepherds too had a cycle of seasonal movement. They belongs
to?
a. Jammu and Kashmir b. Himachal Pradesh
c. Arunachal Pradesh d. Punjab

Answer: B

In a different area of the mountains, the Gaddi shepherds of Himachal Pradesh had a cycle of seasonal
movement. They too spent their winter in the low hills of Siwalik range, grazing their flocks in scrub
forests. By April they moved north and spent the summer in Lahul and Spiti.

Q5. Who among them were not a Pastoral nomad?


(a) Bhatras (b) Bhotiyas
(c) Gujjar (d) Kinnauris

Answer: A

Bhatras were inhabitant of Bastar, Chattisgarh and they were not a Pastrol nomad. Rest were Pastrol
nomad.

Q6. Consider the following Statements:


1. Under Forest Acts, forests which produced commercially valuable timber like Deodar or Sal were
declared 'Protected'. No pastoralist was allowed access to these forests.
2. Other forests were classified as ‘Reserved’ and in these, some customary grazing rights of
pastoralists were granted.

Which of the following is/are correct?


(a) 1 only (b) 2 only (c) All of the above (d) None of the above.

Answer: D

Both are incorrect. By the mid-nineteenth century, various Forest Acts were also being enacted in the
different provinces. Through these Acts some forests which produced commercially valuable timber
like Deodar or Sal were declared ‘Reserved’. No pastoralist was allowed access to these forests. Other
forests were classified as ‘Protected’. In these, some customary grazing rights of pastoralists were
granted but their movements were severely restricted.

Q7. Consider the following statement:


1. In 1871, the colonial government in India passed the Criminal Tribes Act.
Google it:- https://upscpdf.com
www.achieveias.co.in, YouTube Channel: http://youtube.com/c/AchieveIAS Telegram Channel: http://t.me/Achieve_Ias,
Mail: achieveias21@gmail.com, Contact Number: 8968920720
https://upscpdf.com https://upscpdf.com https://upscpdf.com
2. Once this Act came into force, many communities were classified as Criminal Tribes and were put
into concentration camps.

Which of the following statement is/are incorrect?


(a) 1 only (b) 2 only (c) All of the above (d) None of the above.

Answer: A

Statement 2 is incorrect. In 1871, the colonial government in India passed the Criminal Tribes Act.
Under this many communities were classified as Criminal Tribes. Once this Act came into force, these
communities were expected to live only in notified village settlements.

Q8. With reference to Grazing tax, consider the following statement?


1. Pastoralists had to pay tax on every animal they grazed on the pastures.
2. By the 1880s the government abolished this tax.

Which of the following statement is/are correct?


(a) 1 only (b) 2 only (c) All of the above (d) None of the above

Answer: A

To expand its revenue income, the colonial government looked for every possible source of taxation.
So tax was imposed on land, on canal water, on salt, on trade goods, and even on animals. Pastoralists
had to pay tax on every animal they grazed on the pastures. In most pastoral tracts of India, grazing
tax was introduced in the mid-nineteenth century. By the 1880s the government began collecting
taxes directly from the pastoralists.

Q9. Why did colonial government of india put too much emphasis on opium production?
1. Because they were in great demand in Europe.
2. English pharmaceutical companies needed this for production of certain medicines.
3. To balance their trade with China.

Which of the above statements are not correct?


a. 1 and 2 only b. 2 and 3 only c. 1 and 3 only d. All of the above

Answer: A

The history of opium production in India was linked up with the story of British trade with China. In the
late eighteenth century, the English East India Company was buying tea and silk from China for sale in
England. England at this time produced nothing that could be easily sold in China. They could buy tea
only by paying in silver coins or bullion. In such a situation in order to finance the tea trade and balance
their trade They searched for a commodity they could sell in China, something they could persuade the
Chinese to buy. Opium was such a commodity. When the British conquered Bengal, they made a
determined effort to produce opium in the lands under their control.

Google it:- https://upscpdf.com


www.achieveias.co.in, YouTube Channel: http://youtube.com/c/AchieveIAS Telegram Channel: http://t.me/Achieve_Ias,
Mail: achieveias21@gmail.com, Contact Number: 8968920720
https://upscpdf.com https://upscpdf.com https://upscpdf.com
Q10. How did colonial government made unwilling cultivators to produce opium?

(a) By giving advances


(b) By increasing purchasing price of opium
(c) By granting rent free land
(d) By abolishing monopoly on opium trade

Answer: A

Unwilling cultivators were made to produce opium through a system of advances. In the rural areas of
Bengal and Bihar, there were large numbers of poor peasants. They never had enough to survive. It
was difficult for them to pay rent to the landlord or to buy food and clothing. From the 1780s, such
peasants found their village headmen (mahato) giving them money advances to produce opium. When
offered a loan, the cultivators were tempted to accept, hoping to meet their immediate needs and pay
back the loan at a later stage. But the loan tied the peasant to the headman and through him to the
government. It was the government opium agents who were advancing the money to the headmen,
who in turn gave it to the cultivators. By taking the loan, the cultivator was forced to grow opium on a
specified area of land and hand over the produce to the agents once the crop had been harvested.

Google it:- https://upscpdf.com


www.achieveias.co.in, YouTube Channel: http://youtube.com/c/AchieveIAS Telegram Channel: http://t.me/Achieve_Ias, Mail:
achieveias21@gmail.com, Contact Number: 8968920720
https://upscpdf.com https://upscpdf.com https://upscpdf.com
ACHIEVE IAS HISTORY MCQ SERIES, DAY 19, SOLUTIONS

Q1. Consider the following statements.


1. Kheda Satyagrah of Gujarat 1917 was first satyagrah of Mahatama Gandhi in India.
2. In Ahmedabad, 1918 Mahatama Gandhi organise a satyagrah in support of peasants.

Which of them is/are correct?


(a) 2 only (b) 1 only (c) Both of them (d) None of them

Answer: D

In 1916 he travelled to Champaran in Bihar to inspire the peasants to struggle against the oppressive
plantation system. Then in 1917, he organised a satyagraha to support the peasants of the Kheda
district of Gujarat. Affected by crop failure and a plague epidemic, the peasants of Kheda could not
pay the revenue, and were demanding that revenue collection be relaxed. In 1918, Mahatma Gandhi
went to Ahmedabad to organise a satyagraha movement amongst cotton mill workers.

Q2. When did Mahatama Gandhi returned from South Africa?


a) 1915 b) 1917 c) 1916 d) 1914

Answer: A

Q3. Consider the following statements.


1. In Bombay Session of Congress 1920, Mahatama Gandhi convinced leader to start non-
cooperation movement in support of Khilafat movement.
2. Khilafat issue was taken up in support of Ottoman Turkey.

Which of the following statement is not correct?


(a) 1 only (b) 2 only (c) Both of them (d) None of them

Answer: A
The First World War had ended with the defeat of Ottoman Turkey. And there were rumours that a
harsh peace treaty was going to be imposed on the Ottoman emperor – the spiritual head of the
Islamic world (the Khalifa). To defend the Khalifat’s temporal powers, a Khilafat Committee was
formed in Bombay in March 1919. A young generation of Muslim leaders like the brothers
Muhammad Ali and Shaukat Ali, began discussing with Mahatma Gandhi about the possibility of a
united mass action on the issue. Gandhiji saw this as an opportunity to bring Muslims under the
umbrella of a unified national movement. At the Calcutta session of the Congress in September
1920, he convinced other leaders of the need to start a non-cooperation movement in support of
Khilafat as well as for swaraj.

Q4. Who was the writer of the book Hind Swaraj?


a) Bal gangadhar Tilak b) Motilal nehru
c) Gopalkrishna Gokhale d) Mahatama Gandhi

Answer: D
In his famous book Hind Swaraj (1909) Mahatma Gandhi declared that British rule was established in
India with the cooperation of Indians, and had survived only because of this cooperation. If Indians
refused to cooperate, British rule in India would collapse within a year, and Swaraj would come.

Q5. Inspired by Mahatma Gandhi Who led the Rampa rebellion of Andhra Pradesh ?
(a) Turia Bhagat (b) Chakra Bisoi
Google it:- https://upscpdf.com
www.achieveias.co.in, YouTube Channel: http://youtube.com/c/AchieveIAS Telegram Channel: http://t.me/Achieve_Ias, Mail:
achieveias21@gmail.com, Contact Number: 8968920720
https://upscpdf.com https://upscpdf.com https://upscpdf.com
(c) Alluri Sitaram Raju (d) Tirath Singh

Answer: C

Rampa rebellion of Andhra Pradesh was led by Alluri Sitaram Raju. He proclaimed that he was an
incarnation of God. Raju talked of the greatness of Mahatma Gandhi, said he was inspired by the
Non-Cooperation Movement, and persuaded people to wear khadi.

Q6. Consider the following Statements:


1. Distrust between India communities led to failure of Civil Disobedience Movement.
2. Dr. Bhim Rao Ambedkar demanded separate electorate for dalits in Second Round Table
Conference.

Which of the following is/are correct?


(a) 1 only (b) 2 only (c) Both of them (d) None of the above.

Answer: C

Not all social groups were moved by the abstract concept of swaraj. One such group was the nation’s
‘untouchables’, who from around the 1930s had begun to call themselves dalit or oppressed. For long
the Congress had ignored the dalits, for fear of offending the sanatanis, the conservative high-caste
Hindus. In the north, Swami Dayanand Saraswati, who founded the reform association called Arya
Samaj, also supported widow remarriage. Dr B.R. Ambedkar, who organised the dalits into the
Depressed Classes Association in 1930, clashed with Mahatma Gandhi at the second Round Table
Conference by demanding separateelectorates for dalits. When the British government conceded
Ambedkar’s demand, Gandhiji began a fast unto death

Q7. Who was the viceroy of India during Civil Disobedience Movement of 1930?
(a) Lord Irwin (b) Lord willingdon
(c) Lord Linlithgow (d) Lord Lansdowne

Answer: A
Mahatma Gandhi found in salt a powerful symbol that could unite the nation. On 31 January 1930, he
sent a letter to Viceroy Lord Irwin stating eleven demands. Some of these were of general interest;
others were specific demands of different classes, from industrialists to peasants.

Q8. Consider the following statement.


1. In Non-Cooperation Movement and Civil Disobediece Movement, Mahatama Gandhi asked people
to refuse cooperate with British government as well as break colonial laws.
2. Mahatama Gandhi relaunched Civil disobedience Movement in 1932 after failure of Second Round
Table Conference.

Which of the following statement is correct?


(a) 1 only
(b) 2 only
(c) Both of them
(d) None of the above

Answer: B

In only Civil Disobediece Movement Mahatama Gandhi asked people to break colonial laws.

Google it:- https://upscpdf.com


www.achieveias.co.in, YouTube Channel: http://youtube.com/c/AchieveIAS Telegram Channel: http://t.me/Achieve_Ias, Mail:
achieveias21@gmail.com, Contact Number: 8968920720
https://upscpdf.com https://upscpdf.com https://upscpdf.com
Mahatama Gandhi relaunched Civil disobedience Movement in 1932 after failure of Second Round
Table Conference. By the Gandhi-Irwin Pact, Gandhiji consented to participate in aRound Table
Conference in London and the government agreed torelease the political prisoners. In December
1931, Gandhiji went toLondon for the conference, but the negotiations broke down andhe returned
disappointed. Back in India, he discovered that thegovernment had begun a new cycle of repression.
So he relaunched Civil disobedience Movement in 1932.

Q9. Consider the following statement:


1. Oudh Kisan Sabha formed in 1920 was headed by Jawaharlal Nehru.
2. In Madras, Justice Party boycotted council election during Non-Cooperation movement.

Which of the following statement is correct?


(a) 1 only
(b) 2 only
(c) Both of them
(d) None of them

Answer: A

Oudh Kisan Sabha was set up headed by Jawaharlal Nehru, Baba Ramchandra and a few others.
Within a month, over 300 branches had been set up in the villages around the region. So when the
Non-Cooperation Movement began the following year, the effort of the Congress was to integrate the
Awadh peasant struggle into the wider struggle. During Non-Cooperation Movement the council
elections were boycotted in most provinces except Madras, where the Justice Party, the party of the
non-Brahmans, felt that entering the council was one way of gaining some power – something that
usually only Brahmans had access to.

Q10. Consider the following statement:


1. Jawaharlal Nehru and Subhash Chandra Bose were founder of Swaraj Party.
2. Lahore Session of Congress 1929, was presided by Subhash Chandra Bose.

Which among these is/are correct statement?


(a) 1 only
(b) 2 only
(c) Both of them
(d) None of them

Answer: C

C. R. Das and Motilal Nehru formed the Swaraj Party within the Congress to argue for a return to
council politics. But younger leaders like Jawaharlal Nehru and Subhas Chandra Bose pressed for
more radical mass agitation and for full independence. In December 1929, under the presidency of
Jawaharlal Nehru, the Lahore Congress formalised the demand of ‘Purna Swaraj’ or full
independence for India. It was declared that 26 January 1930, would be celebrated as the
Independence Day when people were to take a pledge to struggle for complete independence.

Google it:- https://upscpdf.com


www.achieveias.co.in, YouTube Channel: http://youtube.com/c/AchieveIAS Telegram Channel: http://t.me/Achieve_Ias, Mail:
achieveias21@gmail.com, Contact Number: 8968920720
https://upscpdf.com https://upscpdf.com https://upscpdf.com
ACHIEVE IAS HISTORY MCQ SERIES, DAY 20, SOLUTIONS

Q1. With reference to India and Great depression, Consider the following statements :
1. India's export increased during great depression due to shortage of agricultural goods in other
countries.
2. India's Imports nearly halved during Great depression.

Which of the following is/are correct?


(a) 1only (b) 2 only (c) All of the above (d) None of the above.

Answer: B

The depression immediately affected Indian trade. India’s exports and imports nearly halved
between 1928 and 1934. As international prices crashed, prices in India also plunged.
Between 1928 and 1934, wheat prices in India fell by 50 per cent.

Q2. Consider the following statements.


1. After World War I India witnessed a growth in Industrialisatiion.
2. In 1921 Indentured labour migration was abolished.

Which of the following is correct?


(a) 1 only (b) 2 only (c) All of the above (d) None of the above.

Answer: C

After World War I India witnessed a growth in Industrialisation as war led to shortage of essential
items. To tackle this issue new industries were established in India. From the 1900s India’s nationalist
leaders began opposing the system of indentured labour migration as abusive and cruel. It was
abolished in 1921.

Q3. Consider the following Statement:


1. Products like indigo, jute and tea were mainly produced to be sold in India to earn high revenues.
2. Advertisements became a vehicle of the nationalist message of swadeshi.

Which of the following statement is/are not correct?


(a) 1 only (b) 2 only (c) All of the above (d) None of the above.

Answer: A

European Managing Agencies, which dominated industrial production in India, were interested in
certain kinds of products. They established tea and coffee plantations, acquiring land at cheap rates
from the colonial government; and they invested in mining, indigo and jute. Most of these were
products required primarily for export trade and not for sale in India.

Google it:- https://upscpdf.com


www.achieveias.co.in, YouTube Channel: http://youtube.com/c/AchieveIAS Telegram Channel: http://t.me/Achieve_Ias, Mail:
achieveias21@gmail.com, Contact Number: 8968920720
https://upscpdf.com https://upscpdf.com https://upscpdf.com
When Indian manufacturers advertised the nationalist message was clear and loud. If you care for the
nation then buy products that Indians produce. Advertisements became a vehicle of the nationalist
message of Swadeshi.

Q4. Which of the following was principal western port of East India Company in mid of 17th
century?
a) Surat b) Bombay
c) Bangaluru d) Coramandel coast

Answer: A

In the seventeenth century, Bombay was a group of seven islands under Portuguese control.
In 1661, control of the islands passed into British hands after the marriage of Britain’s King
Charles II to the Portuguese princess. The East India Company quickly shifted its base from
Surat, its principal western port, to Bombay. In mid-17th centuary i.e. 1650s, Bombay was under full
control of Portuguese.

Q5. Consider the following statements in context of World War I


1. Manchester imports into India declined.
2. Industrial production boomed in India.

Which of the following statement is correct?


(a) 1 only (b) 2 only (c) All of the above (d) None of these.

Answer: C

Manchester imports into India declined as British mills busy with war production to meet the needs of
the army. Indian factories were called upon to supply war needs: jute bags, cloth for army uniforms,
tents and leather boots, horse and mule saddles and a host of other items. Many new workers were
employed and everyone was made to work longer hours. Over the war years industrial production
boomed

Q6. Debganer Martye Aagaman a famous novel of 1880s was written by?
(a) Baba Padman ji (b) Lakshaman Moreshwar Holbe
(c) Durgacharan Roy (d) Harinarayan Apte

Answer: C

In 1880, Durgacharan Ray wrote a novel, Debganer Martye Aagaman (The Gods Visit Earth), in
which Brahma, the Creator in Hindu mythology, took a train to Calcutta with some other gods.

Q7. Which Indian city was first to get Smoke Nuisance Legislation?
a) Calcutta b) Bombay c) Madras d) Surat

Answer: A
Google it:- https://upscpdf.com
www.achieveias.co.in, YouTube Channel: http://youtube.com/c/AchieveIAS Telegram Channel: http://t.me/Achieve_Ias, Mail:
achieveias21@gmail.com, Contact Number: 8968920720
https://upscpdf.com https://upscpdf.com https://upscpdf.com
The high content of ash in Indian coal was a problem. Many pleas were made to banish the dirty mills
from the city, with no effect. However, in 1863, Calcutta became the first Indian city to get smoke
nuisance legislation.

Q8. Consider the following statement.


1. East India Company appointed Gomasthas to supervise weavers consolidating its power in region.
2. In 1760, East India Company captured textile market in India.

Which of the following statement is not correct?


(a) 1 only (b) 2 only (c) Both of them (d) None of the above

Answer: B

East India Company in order to establish its monopoly right to trade, it established appointed a paid
servant called the Gomastha to supervise weavers, collect supplies, and examine the quality of cloth.
Before establishing political power in Bengal and Carnatic in the 1760s and 1770s, the East India
Company had found it difficult to ensure a regular supply of goods for export. The French, Dutch,
Portuguese as well as the local traders competed in the market to secure woven cloth.

Q9. Which of the following were highly industrialized region of India?


a) Hyderabad and Madras b) Bengal and Bombay
c) Bombay and Delhi d) Delhi and Madras

Answer: B

While factory industries grew steadily after the war, large industries formed only a small segment of
the economy. Most of them – about 67 per cent in 1911 – were located in Bengal and Bombay. Over
the rest of the country, small-scale production continued to predominate.

Q10. Consider the following statements:


1. Low price.
2. Cheap quality.
3. Machine made

Which of the following statement is/are correct about Manchester made clothes?
(a) 1 and 2 only (b) 2 and 3 only (c) 1 and 3 only (d) None of these

Answer: C

Clothes made in Manchester were cheap in price compared to Indian made cloth. The label was also
to be a mark of quality. When buyers saw ‘MADE IN MANCHESTER’ written in bold on the label, they
were expected to feel confident about buying the cloth. Clothes made in Manchester were machine
made that’s why they were low in price.

Google it:- https://upscpdf.com


www.achieveias.co.in, YouTube Channel: http://youtube.com/c/AchieveIAS Telegram Channel: http://t.me/Achieve_Ias,
Mail: achieveias21@gmail.com, Contact Number: 8968920720
https://upscpdf.com https://upscpdf.com https://upscpdf.com
ACHIEVE IAS HISTORY MCQ SERIES, DAY 21, SOLUTIONS

Q1. Consider the following statements:


1. Marathi novel 'Yamuna Paryatan' was written Lakshman Moreshwar Halbe.
2. Muktamale of 1861 was written by Baba Padmanji.

Which of the following is/are correct?


(a) 1only (b) 2 only (c) All of the above (d) None of the above.

Answer: D

The earliest novel in Marathi was Baba Padmanji’s Yamuna Paryatan (1857), which used a simple style
of storytelling to speak about the plight of widows. This was followed by Lakshman Moreshwar Halbe’s
Muktamala (1861). This was not a realistic novel; it presented an imaginary ‘romance’ narrative with a
moral purpose.

Q2. Who among them introduced printing press in India?


a) Mughals b) Turks c) Portuguese d) English

Answer: C

From 1780, James Augustus Hickey began to edit the Bengal Gazette, a weekly magazine that
described itself as ‘a commercial paper open to all, but influenced by none’. So it was private English
enterprise, proud of its independence from colonial influence that began English printing in India.

Q3. Which of the following was first Novel of Bankim Chandra Chattopadhya?
(a) Durgeshnandini (b) Ananda Math (c) Mrinalini (d) Kapalkundala

Answer: A

Bankim Chandra Chattopadhyay would host a jatra in the courtyard where members of the family would
be gathered. In Bankim’s room, however, a group of literary friends would collect to read, discuss and
judge literary works. Bankim read out Durgeshnandini (1865), his first novel, to such a gathering of
people who were stunned to realise that the Bengali novel had achieved excellence so quickly.

Q4. Who published the first modern novel in Malayalam in 1889?


a) Kandavalli Ramachandra Rudre b) Chandu Menon
c) C.V. Raman Pilla d) Lakshman Rao Godakar

Answer: B

Chandu Menon, a subjudge from Malabar, tried to translate an English novel called Henrietta
Temple written by Benjamin Disraeli into Malayalam. But he quickly realised that his readers in Kerala
were not familiar with the way in which the characters in English novels lived: their clothes, ways of

Google it:- https://upscpdf.com


www.achieveias.co.in, YouTube Channel: http://youtube.com/c/AchieveIAS Telegram Channel: http://t.me/Achieve_Ias,
Mail: achieveias21@gmail.com, Contact Number: 8968920720
https://upscpdf.com https://upscpdf.com https://upscpdf.com
speaking, and manners were unknown to them. They would find a direct translation of an English novel
dreadfully boring. So, he gave up this idea and wrote instead a story in Malayalam in the ‘manner of
English novel books’. This delightful novel called Indulekha, published in 1889, was the first modern
novel in Malayalam.

Q5. Which of the following novel was written by Devaki Nandan Khatri?
a) Amar Jiban b) Istri Dharamvichar c) Chandrakanta d) Gulamgiri

Answer: C

The writings of Devaki Nandan Khatri created a novel-reading public in Hindi. His best-seller,
Chandrakanta – a romance with dazzling elements of fantasy – is believed to have contributed
immensely in popularising the Hindi language and the Nagari script among the educated classes of
those times. Although it was apparently written purely for the ‘pleasure of reading’, this novel also gives
some interesting insights into the fears and desires of its reading public.

Q6. What is Kabirlarai?


a) A novel b) Musical soirees c) Dance performance d) Poetry contest

Answer: D

The old merchant elite of Calcutta patronised public forms of entertainment such as kabirlarai (poetry
contests) and many more forms of entertainments.

Q7. 'Padmarag' is a novel that showed the need for women to reform their condition by their
own actions. Who was the writer of this novel?
a) Prem chand b) Bankim chandra chattopadhya
c) Rokeya Hossein d) Sarojini Naidu

Answer: C

Rokeya Hossein showed the need for women to reform their condition by their own actions. To express
her views she wrote a novel- Padmarag

Q8. Which of the following newspaper was published by Raja Rammohan Roy?
(a) Sambad Kaumudi (b) Samachar chandrika
(c) Voice of India (d) Amrita Bazar Patrika

Answer: A

To reach a wider audience, the ideas were printed in the everyday, spoken language of ordinary people.
Rammohun Roy published the Sambad Kaumudi from 1821.

Q9. In which year the supressive act "The Vernacular Press Act" was passed?

Google it:- https://upscpdf.com


www.achieveias.co.in, YouTube Channel: http://youtube.com/c/AchieveIAS Telegram Channel: http://t.me/Achieve_Ias,
Mail: achieveias21@gmail.com, Contact Number: 8968920720
https://upscpdf.com https://upscpdf.com https://upscpdf.com
a) 1857 b) 1878 c) 1885 d) 1905

Answer: B

After the revolt of 1857, the attitude to freedom of the press changed. Enraged Englishmen demanded
a clamp down on the ‘native’ press. As vernacular newspapers became assertively nationalist, the
colonial government began debating measures of stringent control. In 1878, the Vernacular Press Act
was passed, modelled on the Irish Press Laws. It provided the government with extensive rights to
censor reports and editorials in the vernacular press.

Q10. Who started the first newspaper in India "The Bengal Gazette"?
a) Dadabhai Naroji b) James Augustus Hickey
c) N. N. Sen d) John Adams

Answer: B

From 1780, James Augustus Hickey began to edit the Bengal Gazette, a weekly magazine that
described itself as ‘a commercial paper open to all, but influenced by none’. So it was private English
enterprise, proud of its independence from colonial influence, that began English printing in India. Refer
spectrum for more information.

Google it:- https://upscpdf.com


https://upscpdf.com https://upscpdf.com https://upscpdf.com
ACHIEVE IAS HISTORY MCQ SERIES, DAY 22, SOLUTIONS

Q1. Consider the following statements:


1. The Vijayanagar Kings claimed to rule on behalf of Lord Venkateswara.
2. Vijayanagar Rulers used the title " Hindu Suratrana " which literally means Hindu Sultan.

Which of the following is/are not correct?


(a) 1 only (b) 2 only (c) All of the above (d) None of the above.

Answer: A

It is likely that the very choice of the site of Vijayanagara was inspired by the existence of
the shrines of Virupaksha and Pampadevi. In fact the Vijayanagara kings claimed to rule on
behalf of the god Virupaksha. All royal orders were signed “Shri Virupaksha”, usually in the
Kannada script. Rulers also indicated their close links with the gods by using the title “Hindu
Suratrana”. This was a Sanskritisation of the Arabic term Sultan, meaning king, so it literally
meant Hindu Sultan.

Q2. Consider the following :


i) Tuluvas dynasty
ii) Saluvas dynasty
iii) Sangam dynasty
iv) Aravidu dynasty

Arrange the following dynasties in correct chronology:


(a) (iii) , (i) , (ii) , (iv)
(b) (iii) , (ii) , (i) , (iv)
(c) (iv) , (iii) , (i) , (ii)
(d) (i) , (iii) , (ii) , (iv)

Answer: B

The Vijayanagar Empire was founded in 1336 AD by Harihar I and his brother Bukka Raya I in
Deccan in the wake of the rebellions against Tughluq rule. The empire is named after its capital city of
Vijayanagar. There were four dynasties which ruled over Vijayanagar- Sangama Dynasty, Saluva
Dynasty, Tuluva Dynasty and Aravidu Dynasty.

Q3. Consider the following Statement:


1. Mahanavmi dibba was located on the highest point of Vijayanagar City.
2. Hazara Rama temple was built by Vijayanagar Rulers for subjects of empire.

Which of the following statement is correct ?


(a) 1 only (b) 2 only (c) All of the above. (d) None of the above.

Google it:- https://upscpdf.com


https://upscpdf.com https://upscpdf.com https://upscpdf.com
Answer: A
Located on one of the highest points in the city, the “mahanavami dibba” is a massive
platform rising from a base of about 11,000 sq. ft to a height of 40 ft. There is evidence that it
supported a wooden structure. The base of the platform is covered with relief carvings.
The Hazara Rama temple. This was probably meant to be used only by the king and his
family.

Q4. Whose policy led to decline of Imperial city of Vijayanagar?


a) Rama Raya b) Sada Siva Raya c) Achyuta Deva Raya d) Venkata II

Answer: A

Although the armies of the Sultans were responsible for the destruction of the city of
Vijayanagara but It was the adventurous policy of Rama Raya who tried to play off one Sultan
against another that led the Sultans to combine together and decisively defeat him.

Q5. Who was responsible for bringing ruins of Hampi to light?


a) John M. Fritz b) Colonel Colin Mackenzie
c) George Michell d) John Marshall

Answer: B

The ruins at Hampi were brought to light in 1800 by an engineer and antiquarian named
Colonel Colin Mackenzie. An employee of the English East India Company, he prepared the
first survey map of the site.

Q6. Krishnadeva Raya, the greatest Vijayanagar ruler who was also known as ' Abhinava
Bhoja' belonged to which dynasty?

a) Tuluvas dynasty b) Saluvas dynasty


c) Sangam dynasty d) Aravidu dynasty

Answer: A

Krishnadeva Raya belonged to the Tuluva dynasty. Krishnadeva Raya’s rule was characterised by
expansion and consolidation. This was the time when the land between the Tungabhadra and
Krishna rivers (the Raichur doab) was acquired (1512), the rulers of Orissa were subdued
(1514) and severe defeats were inflicted on the Sultan of Bijapur (1520).

Q7. System of Amara-nayaka was inspired from?


a) Muqti system b) Diwan Riyasat system
c) Chahalgani system d) Iqta system

Google it:- https://upscpdf.com


https://upscpdf.com https://upscpdf.com https://upscpdf.com
Answer: D

The amara-nayaka system was a major political innovation of the Vijayanagara Empire. It is
likely that many features of this system were derived from the iqta system of the Delhi
Sultanate.

Q8. Consider the following statement.


1. Gopuram were pavillion that ran around the shrines within the temple complex.
2. Mandapas were placed at the top of Virupaksha temple.

Which of the following statement is correct?


(a) 1 only (b) 2 only (c) Both of them (d) None of the above

Answer: D

Raya gopurams were royal gateways (not a pavilion) that often dwarfed the towers on the
central shrines, and signalled the presence of the temple from a great distance. This was tricky
question to check your grip on terms. Mandapas WERE pavilions and long, pillared corridors (so
this can’t be placed at the top of Virupaksha temple) that often ran around the shrines within the
temple complex. Let us look at two temples more closely – the Virupaksha temple and the
Vitthala temple.

Q9. With respect to Vijayanagar Empire "Kudirai Chettis" were?


a) Millitary Officers b) Peasants
c) Village Headman d) Horse Traders

Answer: D

The trade in Vijaynagar Empire was initially controlled by Arab traders. Local communities of
merchants known as kudirai chettis or horse merchants also participated in these exchanges.

Q10. Consider the following statements:


1. Dam on the river Tungabhadra was built by Krishnadeva Raya.
2. Fortification was only done around city.

Which of the following statement is not correct?


(a) 1 only (b) 2 only (c) Both a and b (d) None of these

Answer: C

Google it:- https://upscpdf.com


https://upscpdf.com https://upscpdf.com https://upscpdf.com
Dam across the Tungabhadra and irrigated the cultivated valley that separated the “sacred
centre” from the “urban core”. This was apparently built by kings of the Sangama dynasty. But we
know Krishnadeva Raya belonged to Tuluva dynasty.
About fortifications it is mentioned about seven lines of forts. These encircled not only the city
but also its agricultural hinterland and forests. The outermost wall linked the hills surrounding the
city. The massive masonry construction was slightly tapered. No mortar or cementing agent was
employed anywhere in the construction. The stone blocks were wedge shaped, which held them in
place, and the inner portion of the walls was of earth packed with rubble. Square or rectangular
bastions projected outwards.

Google it:- https://upscpdf.com


www.achieveias.co.in, YouTube Channel: http://youtube.com/c/AchieveIAS Telegram Channel: http://t.me/Achieve_Ias,
Mail: achieveias21@gmail.com, Contact Number: 8968920720
https://upscpdf.com https://upscpdf.com https://upscpdf.com
ACHIEVE IAS HISTORY MCQ SERIES, DAY 23, SOLUTIONS

Q1. Consider the following statements:


1. Pahi kashta were residents of the village in which they held their lands.
2. Khud khasta were non-resident cultivators who belonged to some other village, but cultivated lands
elsewhere on a contractual basis.

Which of the following is/are correct?


(a) 1 only (b) 2 only (c) All of the above (d) None of the above.

Answer: D

Sources of the seventeenth century refer to two kinds of peasants – khud-kashta and pahi-kashta. The
former were residents of the village in which they held their lands. The latter were non-resident
cultivators who belonged to some other village, but cultivated lands elsewhere on a contractual basis.
People became pahi-kashta either out of choice – for example, when terms of revenue in a distant
village were more favourable – or out of compulsion – for example, forced by economic distress after
a famine.

Q2. Which of the following is not correct in context of Panchayat System of Mughal Period?
(a) Agricultural worker was unlikely to be represented there.
(b) Village headman was chosen through the consensus of the village elders.
(c) The panchayat derived all its funds from Zamindars and Royal treasury.
(d) Panchayats ensured caste boundaries among various communities inside the village.

Answer: C

The panchayat derived its funds from contributions made by individuals to a common financial pool.
These funds were used for defraying the costs of entertaining revenue officials who visited the village
from time to time.

Q3. Consider the following statement with respect to the third book of Ain-e-Akbari , Mulk Abadi.
1. This gives a detail picture of 'suba' level but the details of 'sarkars' is missing.
2. Major merit of Ain-e-Akbari is in its autherncity of quantative data.

Which of the following statement is correct?

(a) 1 only (b) 2 only (c) All of the above (d) None of the above.

Answer: D

The third book, mulk-abadi, is the one which deals with the fiscal side of the empire and provides rich
quantitative information on revenue rates, followed by the “Account of the Twelve Provinces”. This
section has detailed statistical information, which includes the geographic, topographic and economic

Google it:- https://upscpdf.com


www.achieveias.co.in, YouTube Channel: http://youtube.com/c/AchieveIAS Telegram Channel: http://t.me/Achieve_Ias,
Mail: achieveias21@gmail.com, Contact Number: 8968920720
https://upscpdf.com https://upscpdf.com https://upscpdf.com
profile of all subas and their administrative and fiscal divisions (sarkars, parganas and mahals), total
measured area, and assessed revenue (jama).After setting out details at the suba level, the Ain goes
on to give a detailed picture of the sarkars below the suba.

Q4. Expansion of trade led to abundance of which metal in India?


a) Silver b) Gold c) Cooper d) Bronze

Answer: A

An expanding trade brought in huge amounts of silver bullion into Asia to pay for goods procured from
India, and a large part of that bullion gravitated towards India. This was good for India as it did not have
natural resources of silver. As a result, the period between the sixteenth and eighteenth centuries was
also marked by a remarkable stability in the availability of metal currency, particularly the silver rupya
in India.

Q5. Consider the following:


1. Sugarcane
2. Cotton
3. Tobacco

Which of the following crop was/were considered as Jins-i-kamil?


(a) 1 and 2only (b) 2 and 3only (c) 1 and 3 only (d) All of the above.

Answer: A

The term jins-i kamil (literally, perfect crops) in our sources. The Mughal state also encouraged
peasants to cultivate such crops as they brought in more revenue. Crops such as cotton and sugarcane
were jins-i kamil par excellence.

Q6. What is the meaning of Amil-guzar in context of Mughal revenue system?


(a) Millitary commander (b) Revenue collector
(c) Court poet (d) Zamindar

Answer: B

The jama was the amount assessed, as opposed to hasil, the amount collected. In his list of duties of
the amil-guzar which was revenue collector.

Q7. Which Mughal Emperor was first to ban tabacco?


a) Akbar b) Jahangir c) Shahjahan d) Aunrangzeb

Answer: B

Google it:- https://upscpdf.com


www.achieveias.co.in, YouTube Channel: http://youtube.com/c/AchieveIAS Telegram Channel: http://t.me/Achieve_Ias,
Mail: achieveias21@gmail.com, Contact Number: 8968920720
https://upscpdf.com https://upscpdf.com https://upscpdf.com
Akbar and his nobles came across tobacco for the first time in 1604. At this time smoking tobacco (in
hookahs or chillums) seems to have caught on in a big way. Jahangir was so concerned about its
addiction that he banned it.

Q8. Consider the following statement in context of condition of women during Mughal period.
1. Remarriage was considered legitimate both among divorced and widowed women.
2. Hindu and Muslim women inherited zamindaris which they were free to sell or mortgage.

Which of the following statement is/are not correct?


(a) 1 only (b) 2 only (c) Both of them (d) None of the above

Answer: C

Both are correct as

Marriages in many rural communities required the payment of bride-price rather than dowry to the
bride’s family. Remarriage was considered legitimate both among divorced and widowed women.

Amongst the landed gentry, women had the right to inherit property. Instances from the Punjab show
that women, including widows, actively participated in the rural land market as sellers of property
inherited by them. Hindu and Muslim women inherited zamindaris which they were free to sell or
mortgage. Women zamindars were known in eighteenth-century Bengal. In fact, one of the biggest and
most famous of the eighteenth-century zamindaris, that of Rajshahi, had a woman at the helm.

Q9. The Arabic word 'Muqaddam' was used for?


a) Peasants b) Labourers c) Village headmen d) Rich merchants

Answer: C

There were three constituents of village community – the cultivators, the panchayat, and the village
headman who were also known as muqaddam or mandal.

Q10. Consider the following statements:


1. Maize
2. Potatoes
3. Papaya

Which of the following crop has originated from India?


(a) 1 only (b) 2 only (c) 2 and 3 only (d) None of these

Answer: D

Maize (makka), for example, was introduced into India via Africa and Spain and by the seventeenth
century it was being listed as oneof the major crops of western India. Vegetables liketomatoes, potatoes

Google it:- https://upscpdf.com


www.achieveias.co.in, YouTube Channel: http://youtube.com/c/AchieveIAS Telegram Channel: http://t.me/Achieve_Ias,
Mail: achieveias21@gmail.com, Contact Number: 8968920720
https://upscpdf.com https://upscpdf.com https://upscpdf.com
and chillies were introduced from the New World at this time, as were fruits like the pineapple and the
papaya.

Google it:- https://upscpdf.com


www.achieveias.co.in, YouTube Channel: http://youtube.com/c/AchieveIAS Telegram Channel: http://t.me/Achieve_Ias, Mail:
achieveias21@gmail.com, Contact Number: 8968920720
https://upscpdf.com https://upscpdf.com https://upscpdf.com
ACHIEVE IAS HISTORY MCQ SERIES, DAY 24, SOLUTIONS

Q1. Consider the following statements:


1. Henry Bloachman was first scholar to translate all 3 volumes of Ain-e-Akbari into english.
2. Henry Bloachman was associated with Asiatic society of Bengal.

Which of the following is/are correct?


(a) 1only (b) 2 only (c) All of the above (d) None of the above.

Answer: B

Ain-e-Akbari has been translated for use by a number of scholars. Henry Blochmann edited it and the
Asiatic Society of Bengal,Calcutta (present-day Kolkata), published it in its Bibliotheca Indica series.
The book has also been translated into English in three volumes. The standard translation of Volume
1 is that of Henry Blochmann (Calcutta 1873). The other two volumes were translated by H.S. Jarrett
(Calcutta 1891 and 1894).

Q2. When did Nadir Shah invaded India?


a) 1751 b) 1739 c) 1749 d) 1748

Answer: B

In 1739, Nadir Shah invades India and sacks Delhi.

Q3. In which year Third Battle of Panipat was fought?


(a) 1760 (b) 1763 (c) 1761 (d) 1764

Answer: C

In 1761, Ahmad Shah Abdali defeats the Marathas in the third battle of Panipat

Q4. With reference to Mughal Administration, what was the role of Qanungos?
a) Incharge of fort b) Incharge of weapons
c) Officials of land record keeping d) Forest officer

Answer: C

Qanungos were the local hereditary officials of land record keeping.

Q5. In context of Mughal Empire, who is Mutasaddi?


a) Governor of port b) Statistics officer.
c) Forest officer d) Court artist.

Answer: A

Google it:- https://upscpdf.com


www.achieveias.co.in, YouTube Channel: http://youtube.com/c/AchieveIAS Telegram Channel: http://t.me/Achieve_Ias, Mail:
achieveias21@gmail.com, Contact Number: 8968920720
https://upscpdf.com https://upscpdf.com https://upscpdf.com
The port administration was independent of the provincial authority. The governor of the port was called
Mutasaddi.

Q6. Consider the following statements in context of classification of land under Mughal Emperor
Akbar:
1. Polaj is land which is annually cultivated for each crop in succession and is never allowed to lie
fallow.
2. Chachar is land that has lain fallow for three or four years.

Which of the following is/are not correct?


(a) 1only (b) 2 only (c) All of the above (d) None of the above.

Answer: D

Both are correct as the Emperor Akbar in his profound sagacity classified the lands and fixed a different
revenue to be paid by each. Polaj is land which is annually cultivated for each crop in succession and
is never allowed to lie fallow. Parauti is land left out of cultivation for a time that it may recover its
strength. Chachar is land that has lain fallow for three or four years. Banjar is land uncultivated for five
years and more. Of the first two kinds of land, there are three classes, good, middling, and bad. They
add together the produce of each sort, and the third of this represents the medium produce, one-third
part of which is exacted as the Royal dues.

Q7. In medieval India, Mansabdari system was introduced for:


(a) Making recruitment to the army (b) Facilitating revenue collection
(c) Ensuring religious harmony (d) Effecting clean administration

Answer: D

Mansabdari system was introduced in 1595–96, was a combined status showing a noble’s civil and
military capacity.

Q8. The medieval Indian writer who refers to the discovery of America is:
(a) Malik Muhammad Jayasi (b) Amir Khusrau
(c) Raskhan (d) Abul Fazl

Answer: D

Abu Fazl refers to the discovery of America.

Q9. With reference to role of Jati panchayats, consider the following statements :
a) Jati panchayats arbitrated civil disputes between members of different castes.
b) In most cases, except in matters of criminal justice, the state respected the decisions of jati
panchayats.

Which of the following is/are correct?


(a) 1 only (b) 2 only (c) Both are correct (d) None of the above.
Google it:- https://upscpdf.com
www.achieveias.co.in, YouTube Channel: http://youtube.com/c/AchieveIAS Telegram Channel: http://t.me/Achieve_Ias, Mail:
achieveias21@gmail.com, Contact Number: 8968920720
https://upscpdf.com https://upscpdf.com https://upscpdf.com
Answer: C

In addition to the village panchayat each caste or jati in the village had its own jati panchayat.These
panchayats wielded considerable power in rural society. In Rajasthan jati panchayats arbitrated civil
disputes between members of different castes. They mediated in contested claims on land, decided
whether marriages were performed according to the norms laid down by a particular caste group, and
determined who had ritual precedence in village functions, and so on. In most cases, except in matters
of criminal justice, the state respected the decisions of jati panchayats.

Q10. Match List-I with List-II and select the correct answer using the codes given below the lists:
A. 1556 1. Battle of Haldi Ghati
B. 1600 2. Nadir Shah's capture of Delhi
C. 1686 3. Death of Shivaji
D. 1739 4. Grant of Charter to East India Company
5. Accession of Akbar

(a) A – 3; B – 4; C – 2; D – 1 (b) A – 5; B – 4; C – 3; D – 2
(c) A – 5; B – 2; C – 1; D – 4 (d) A – 1; B – 5; C – 3; D – 2

Answer: B

The Battle of Haldighati was fought between the Mughal Empire and the forces of Mewar on June 21,
1576 at Haldighati in Rajasthan, India. It was a decisive victory for the Mughal Emperor Jalal ud-Din
Muhammad Akbar’s general Raja Man Singh against the Maharana Pratap Singh of Mewar. Akbar
was 14 years old when he was crowned at Kalanaur in 1556.

Google it:- https://upscpdf.com


www.achieveias.co.in, YouTube Channel: http://youtube.com/c/AchieveIAS Telegram Channel: http://t.me/Achieve_Ias,
Mail: achieveias21@gmail.com, Contact Number: 8968920720
https://upscpdf.com https://upscpdf.com https://upscpdf.com
ACHIEVE IAS HISTORY MCQ SERIES, DAY 25, SOLUTIONS

Q1. Consider the following statements:


1. Members of the Jesuit mission were impressed by Shah Jahan.
2. Shah Jahan was curious about Christianity and dispatched an embassy to Goa to invite Jesuit
priests.

Which of the following is/are correct?


a. 1 only b. 2 only c. All of the above d. None of the above.

Answer: D

The high respect shown by Akbar towards the members of the Jesuit mission impressed them deeply.
They interpreted the emperor’s open interest in the doctrines of Christianity as a Sign of his
acceptance of their faith. This can be understood in the light of the prevailing Climate of religious
intolerance in Western Europe. Monserrate remarked that “the king cared little that in allowing
everyone to follow his religion he was in reality violating all”. Akbar was curious about Christianity and
dispatched an embassy to Goa to invite Jesuit Priests. The first Jesuit mission reached the Mughal
court at Fatehpur Sikri in 1580 and St. Jesuit stayed for about two years. The Jesuits spoke to Akbar
about Christianity and debated its virtues with the ulama. Two more missions were sent to the Mughal
court at Lahore, in 1591 and 1595.

Q2. Muhammad waris was a Mughal historian who completed Lahori’s ‘Badshahnama’ in
1656. Which Mughal ruler patronised Muhammad Waris?
a. Akbar b. Jahangir c. Shah Jahan d. Aurangzeb

Answer: C

Shah Jahan in his eighth regnal year asked Muhammad Amin Qazvini to write an official History of his
reign and he completed his Badshahnama in 1636, which covers the first ten (lunar) years of Shah
Jahan’reign. Jalaluddin Tabatabai wrote another Badshahnama, but the extant portion of the text
covers Only four years, from fifth to eighth regnal year of the emperor. The project was later given to
Abdul Hamid Lahori, who wrote his Badshahnama in two volumes. The first volume of this Work is
based upon Qazvini’s work but has more details. The second volume covers the next Ten (lunar)
years of Shah Jahan’s reign. He completed his work in 1648. Lahori died in 1654. Muhammad Waris,
a pupil of Lahori was given the responsibility to complete the task and his Badshahnama (completed
in 1656) covers the rest of the period of Shah Jahan’s reign. His Work was published by the Asiatic
Society as the third volume of the Badshahnama of Lahori.

Q3. Consider the following statement:


1. The Ramayana was translated into persian as the Razmnama.
2. Abu’l-Hasan was a painter in the court of Jahangir.

Which of the following statement is correct?


(a) 1 only (b) 2 only (c) All of the above (d) None of the above.

Google it:- https://upscpdf.com


www.achieveias.co.in, YouTube Channel: http://youtube.com/c/AchieveIAS Telegram Channel: http://t.me/Achieve_Ias,
Mail: achieveias21@gmail.com, Contact Number: 8968920720
https://upscpdf.com https://upscpdf.com https://upscpdf.com
Answer: B

Mughal chronicles such as the Akbar Nama werewritten in Persian, others, like Babur’s Memoirs,
were translated from the Turkish into the Persian Babur Nama. Translations of Sanskrit texts such as
the Mahabharata and the Ramayana into Persian were commissioned By the Mughal emperors. The
Mahabharata was translated as the Razmnama (Book of Wars) Abu’l-Hasan (or Abu al-Hasan) (1589
– c. 1630), from Delhi, India, was a Mughal painter of Miniatures in the reign of Jahangir.Abu al-
Hasan was initially trained by the emperor himself in his large studios and workshops But soon
surpassed his father and his employer. Jahangir said of him that he had no equal and Bestowed the
title Nadir-uz-Saman (“Wonder of the Age”) on him.

Q4. Consider the following:


1. Aghas
2. Chaudhari
3. Khwajasara

Who among them was not a part of Mughal household?


(a) 1 only (b) 2 only (c) 3 only (d) None of the above.

Answer: B

The chaudhuri means in charge of revenue collection. They were not a part of Mughal Household.
The term “harem” is frequently used to refer to the domestic world of the Mughals. It
Originates in the Persian word haram, meaning a sacred place. The Mughal household
Consisted of the emperor’s wives and concubines, his near and distant relatives (mother, step-
and foster-mothers, sisters, daughters, daughters-in-law, aunts, children, Etc.), and female
servants and slaves. In the Mughal household a distinction was maintained between wives
who came from Royal families (begams), and other wives (aghas) who were not of noble
birth. Apart from wives, numerous male and female slaves populated the Mughal household.
The tasks they performed varied from the most mundane to those requiring skill, tact And
intelligence. Slave eunuchs (khwajasara) moved between the external and internal Life of the
household as guards, servants, and also as agents for women dabbling in Commerce.

Q5. Consider the following statement:


1. The bazaar of Chandni Chowk, the throbbing centre of Shahjahanabad, was designed by
Roshanara.
2. The book ‘Char Chaman’ was written during reign of Akbar.

Which of the following statement is/are correct?


(a) 1 only (b) 2 only (c) Both (d) None of the above

Answer: D

Google it:- https://upscpdf.com


www.achieveias.co.in, YouTube Channel: http://youtube.com/c/AchieveIAS Telegram Channel: http://t.me/Achieve_Ias,
Mail: achieveias21@gmail.com, Contact Number: 8968920720
https://upscpdf.com https://upscpdf.com https://upscpdf.com
The bazaar of Chandni Chowk,the throbbing centre of Shahjahanabad, was designed by Jahanara.
Chandrabhan Barahman described the Mughal nobility in his book Char Chaman (Four Gardens) was
written during the reign of Shah Jahan.

Q6. Which Mughal emperor introduced Jharoka darshan?


a. Humayun b. Akbar c. Jahangir d. Shah Jahan

Answer: B

Jharoka darshan was introduced by Akbar with the objective of broadening the acceptance of The
imperial authority as part of popular faith.

Q7. Consider the following pairs:


1. Robe of honour - Khilat
2. Sash - Patka
3. Gift of large amount of money - Nazr

Which of the following is not correctly matched?


(a) 1 only (b) 2 only (c) 3 only (d) None of these

Answer: C

Awards included the robe of honour (khilat), a Garment once worn by the emperor and imbued with
his Benediction. One gift, the sarapa (“head to foot”), consisted Of a tunic, a turban and a sash
(patka). Jewelled ornaments Were often given as gifts by the emperor. The lotus blossom Set with
jewels (padma murassa) was given only in Exceptional circumstances.A courtier never approached
the emperor empty handed:He offered either a small sum of money (nazr ) or a large Amount
(peshkash).

Q8. Consider the following statement:


1. Qandahar was a bone of contention between the Ottamons and the Mughals..
2. Mughals lost Qandahar during the reign of Jhanagir.

Which of the following statement is/are correct?


(a) 1 only (b) 2 only (c) Both of them (d) None of the above

Answer: B

Qandahar was a bone of contention between the Safavids and the Mughals. The Fortress-town
had initially been in the possession ofHumayun, reconquered in 1595 by Akbar.In the winter
of 1622 during reign of Jahangir, Persian army besieged Qandahar. The ill-Prepared Mughal
garrison was defeated and had to surrender the fortress and the city To the Safavids.

Perfecting prelims: Previous year questions from this chapter-

Google it:- https://upscpdf.com


www.achieveias.co.in, YouTube Channel: http://youtube.com/c/AchieveIAS Telegram Channel: http://t.me/Achieve_Ias,
Mail: achieveias21@gmail.com, Contact Number: 8968920720
https://upscpdf.com https://upscpdf.com https://upscpdf.com
Q9. Nastaliq was: (1996)
(a) A persian script used in medieval India
(b) A raga composed by Tansen
(c) A cess levied by the Mughal rulers
(d) A manual of code of conduct for the Ulemas

Answer: A

Nastaliq was a persian script during Mughal period.

Q10. Mughal painting reached its zenith under: (1996)


(a) Humayun (b) Akbar (c) Jahangir (d) Shahjahan

Answers: C

Jahangir was the Mughal Emperor from 1605–1627.

Q11. The loss of Qandhar was a big blow to the Mughal empire from the view point of : (1998)
(a) Natural resources (b) Buffer territory
(c) Communication (d) Strategic stronghold

Answer: D

Shah Jahan recovered Kandhar in 1638 from the Iranians but lost it again in 1649 despite three
Campaigns. The loss of Qandhar was a big blow as it was a strategic stronghold

Q12. Assertion (A): During the reign of Shahjahan, Dara Sikoh was sent on expedition to
Balkha, Badakhshan and Qandahar. (1998)
Reason (R): The expedition sent by Shahjahan to the Middle-East was a marvellous success.

(a) Both A and R are true but R is the correct explanation of A


(b) Both A and R are true but R is not a correct explanation of A
(c) A is true but R is false
(d) A is false but R is true

Answer: C

R is incorrect as this expedition failed miserably

Q13. Assertion (A): During the time of Akbar, for every ten cavalrymen, the mansabdars had to
maintain twenty horses.

Google it:- https://upscpdf.com


www.achieveias.co.in, YouTube Channel: http://youtube.com/c/AchieveIAS Telegram Channel: http://t.me/Achieve_Ias,
Mail: achieveias21@gmail.com, Contact Number: 8968920720
https://upscpdf.com https://upscpdf.com https://upscpdf.com
Reason (R): Horses had to be rested while on march and replacements’ were necessary in
times of war. (1999)

(a) Both A and R are true but R is the correct explanation of A


(b) Both A and R are true but R is not a correct explanation of A
(c) A is true but R is false
(d) A is false but R is true

Answer: D

A is incorrect as Du aspa Sih aspa system was introduced by Jahangir (not Akbar). So answer is d.

Google it:- https://upscpdf.com


www.achieveias.co.in, YouTube Channel: http://youtube.com/c/AchieveIAS Telegram Channel: http://t.me/Achieve_Ias, Mail:
achieveias21@gmail.com, Contact Number: 8968920720
https://upscpdf.com https://upscpdf.com https://upscpdf.com
ACHIEVE IAS HISTORY MCQ SERIES, DAY 26, SOLUTIONS

Q1. Consider the following statements:


1. According to the Sunset Law, if payment did not come in by sunset of the specified date, the
zamindari was liable to be auctioned.
2. A low demand was imposed in the 1790s, a time when the prices of agricultural produce were
depressed.

Which of the following is/are not correct?


(a) 1 only (b) 2 only (c) All of the above (d) None of the above.

Answer: B
A high revenue demand was imposed in the 1790s, a time when the prices of agricultural
produce were depressed, making it difficult for the ryots to pay their dues to the zamindar.
According to the Sunset Law, if payment did not come in by sunset of the specified date, the
zamindari was liable to be auctioned.

Q2.Who were Jotedars?


(a) Village headmen (b) Rich ryots
(c) Merchants (d) An officer at collectorate

Answer: B

While many zamindars were facing a crisis at the end of the eighteenth century, a group of rich peasants
were consolidating their position in the villages. In Francis Buchanan’s survey of the Dinajpur district in
North Bengal we have a vivid description of this class of rich peasants known as Jotedars.

Q3. Consider the following statement:


1. Collectors emerged as an alternative of Zamindars in respect to authority.
2. Zamindars collected rent with the help of an officer 'Amlah'.

Which of the following statement is correct?


(a) 1 only (b) 2 only (c) Both of them (d) None of the above.

Answer: C

Zamindars lost their power to organise local justice and the local police. Over time the collectorate
emerged as an alternative centre of authority, severely restricting what the zamindar could do. At the
time of rent collection, an officer of the zamindar, usually the amlah, came around to the village.

Q4. Which of the following is correct about Aftermath of Santhal Revolt?


(a) Santhals were listed as criminal tribe.
(b) Santhals were expelled from their lands.
(c) Santhals were granted a santhal paragane.

Google it:- https://upscpdf.com


www.achieveias.co.in, YouTube Channel: http://youtube.com/c/AchieveIAS Telegram Channel: http://t.me/Achieve_Ias, Mail:
achieveias21@gmail.com, Contact Number: 8968920720
https://upscpdf.com https://upscpdf.com https://upscpdf.com
(d) Adult members of santhals were made slaves.

Answer: C

By the 1850s, the Santhals felt that the time hadcome to rebel against zamindars, moneylenders and
the colonial state, in order to create an ideal world for themselves where they would rule. It was after
the Santhal Revolt (1855-56) that the Santhal Pargana was created, carving out 5,500 square miles
from the districts of Bhagalpur and Birbhum. The colonial state hoped that by creating a new territory
for the Santhals and imposing some special laws within it, the Santhals could be conciliated.

Q5. For what purpose British East India Company hired Francis Buchanan?
(a) For detailed survey of areas under EIC jurisdiction.
(b) For position of commanding officer.
(c) For position of district collector.
(d) For Zamindar.

Answer: A

Francis Buchanan was a physician who came to India and served in the Bengal Medical Service (from
1794 to 1815). For a few years he was surgeon to the Governor-General of India, Lord
Wellesley. During his stay in Calcutta (present-day Kolkata), he organised a zoo that became the
Calcutta Alipore Zoo; he was also in charge of the Botanical Gardens for a short period. On the request
of the Government of Bengal, he undertook detailed surveys of the areas under the jurisdiction of the
British East India Company. In 1815 he fell ill and returned to England. Upon his mother’s death, he
inherited her property and assumed her family name Hamilton.
So he is often called Buchanan-Hamilton.

Q6. In which year ryots of Supa rose to revolt?


a) 1856 b) 1875 c) 1852 d) 1878

Answer: B

The movement began at Supa, a large village in Poona (present-day Pune) district. It was a market
centre where many shopkeepers and moneylenders lived. On 12 May1875, ryots from surrounding
rural areas gathered and attacked the shopkeepers, demanding their bahi khatas (account books) and
debt bonds. They burnt the khatas, looted grain shops, and in some cases set fire to the houses of
sahukars.

Q7. Why British govt. did not expand Permanent Settlement beyond Bengal?
(a) Permanent Settlement was ineffective.
(b) Due to frequent revolt.
(c) Zamindars cost their authority.
(d) To get claim over enhanced income.

Google it:- https://upscpdf.com


www.achieveias.co.in, YouTube Channel: http://youtube.com/c/AchieveIAS Telegram Channel: http://t.me/Achieve_Ias, Mail:
achieveias21@gmail.com, Contact Number: 8968920720
https://upscpdf.com https://upscpdf.com https://upscpdf.com
Answer: D

As British rule expanded from Bengal to other parts of India, new systems of revenue were imposed.
The Permanent Settlement was rarely extended to any region beyond Bengal. Reason was that after
1810, agricultural prices rose, increasing the value of harvest produce, and enlarging the income of the
Bengal zamindars. Since the revenue demand was fixed under the Permanent Settlement, the colonial
state could not claim any share of this enhanced income. Keen on expanding its financial resources,
the colonial government had to think of ways to maximise its land revenue. So in territories annexed in
the nineteenth century, temporary revenue settlements were made.

Q8. In which year Deccan Riots Report was present to the British parliament?
(a) 1858 (b) 1875 (c) 1878 (d) 1877

Answer: C

When the revolt spread in the Deccan, the Government of Bombay was initially unwilling to see
it as anything serious. But the Government of India, worried by the memory of 1857, pressurised
the Government of Bombay to set up a commission of enquiry to investigate into the causes
of the riots. The commission produced a report that was presented to the British Parliament in
1878. This report, referred to as the Deccan Riots Report, provides historians with a range of
sources for the study of the riot.

Q9. Consider the following statements regarding the early years of the East India Company
establishment in India:
(1). The colonial rule was first established in The Bengal.
(2). In the beginning, the earliest attempts were made to reorder rural society and establish a new
regime of land rights and a new revenue system.
(3). The Permanent Settlement had come into operation in 1793. The East India Company had fixed
the revenue that each zamindar had to pay.

Which of the following statement(s) is/are correct?


(a) Only 1 (b) 1 and 2 only (c) 1 and 3 (d) All of the above

Answer: D
The English East India Company (E.I.C.) established its raj in the countryside, implemented its revenue
policies, these policies meant to different sections of people, and how they changed everyday lives.
The colonial rule was first established in The Bengal. In the beginning, the earliest attempts were made
to reorder rural society and establish a new regime of land rights and a new revenue system.

Q10. Consider the following statements regarding after the establishment of British East India
Company in Bengal:
1. In 1797 there was an auction in Burdwan (present day Bardhaman) during which A number of mahals
(estates) held by the Raja of Burdwan were being sold.
2. The Permanent Settlement had come into operation in 1793.
3. The estates of those who failed to pay were to be auctioned to recover the revenue.

Google it:- https://upscpdf.com


www.achieveias.co.in, YouTube Channel: http://youtube.com/c/AchieveIAS Telegram Channel: http://t.me/Achieve_Ias, Mail:
achieveias21@gmail.com, Contact Number: 8968920720
https://upscpdf.com https://upscpdf.com https://upscpdf.com
Which of the following statement(s) is/are correct?
(A). Only 1 (B) 1 and 2 (C) 1 and 3 (D) All of the above

Answer: D

Explanation: The East India Company had fixed the revenue that each zamindar had to pay. The
estates of those who failed to pay were to be auctioned to recover the revenue. Since the raja had
accumulated huge arrears, his estates had been put up for auction.

Google it:- https://upscpdf.com


www.achieveias.co.in, YouTube Channel: http://youtube.com/c/AchieveIAS Telegram Channel: http://t.me/Achieve_Ias,
Mail: achieveias21@gmail.com, Contact Number: 8968920720
https://upscpdf.com https://upscpdf.com https://upscpdf.com
ACHIEVE IAS HISTORY MCQ SERIES, DAY 27, SOLUTIONS

Q1. Consider the following statements:


1. According to limitation law loan bonds signed between moneylenders and ryots would have validity
for only three years.
2. This law was meant to regulate the wealth of Jotedars.

Which of the following is/are correct?


(a) 1 only (b) 2 only (c) All of the above (d) None of the above.

Answer: A
The ryots came to see the moneylender as devious and deceitful. They complained of moneylenders
manipulating laws and forging accounts. In 1859 the British passed a Limitation Law that stated that
the loan bonds signed between moneylenders and ryots would have validity for only three years. This
law was meant to check the accumulation of interest over time.

Q2. Under the Permanent Settlement, 1893, the Zamindars are required to issue pattas to the
farmers which were not issued by many of the Zamindars.
The reason was:
(a) The Zamindars were trusted by the farmers.
(b) There was no officials check upon the Zamindars.
(c) It was the responsibility of the British government.
(d) The farmers were not interested in getting pattas

Answer: B

John Shore planned Permanent Settlement and it was introduced in 1793 by Lord Cornwallis.
Zamindars were made the owners of the land and the British got a fixed share of 10/11th of the revenue
collected by the zamindars.

Q3. After the Santhal Uprising subsided, what was/were the measure/measures taken by the
colonial government?
1. The territories called 'Santhal Paraganas' were create.
2. It became illegal for a Santhal to transfer land to a non-Santhal.

Choose from following options:


(a) 1 only (b) 2 only (c) Both 1 and 2 (d) Neither 1 nor 2

Answer: C

It was after the Santhal Revolt (1855-56) that the Santhal Pargana was created, carving out 5,500
square miles from the districts of Bhagalpur and Birbhum. The colonial state hoped that by creating a
new territory for the Santhals and imposing some special laws within it, the Santhals could be conciliate.
So statement 1 is right. After the revolt was suppressed, the British government passed the Santhal
Parganas Tenancy Act (SPT), which prohibits the transfer of land. So, statement 2 also right, At the
time of rent collection, an officer of the zamindar, usually the amlah, came around to the village.

Google it:- https://upscpdf.com


www.achieveias.co.in, YouTube Channel: http://youtube.com/c/AchieveIAS Telegram Channel: http://t.me/Achieve_Ias,
Mail: achieveias21@gmail.com, Contact Number: 8968920720
https://upscpdf.com https://upscpdf.com https://upscpdf.com
Q4. Under which one of the following systems of assessment, the British Government collected
revenue directly from the farmers?
(a) Zamindari (b) Annawari (c) Ryotwari (d) Desaiwari

Answer: (c)
Ryotwari System was introduced by Thomas Munro in 1820. In this System, the ownership rights were
handed over to the peasants. British Government collected taxes directly from the peasants.

Q5. After Deccan Riots as a conciliatory measure, the Deccan Agriculturists Relief Act was
passed in which year?
(a) 1858 (b) 1878 (c) 1879 (d) 1880

Answer: C

The Government succeeded in repressing the Deccan movement. As a conciliatory measure, the
Deccan Agriculturists Relief Act was passed in 1879, This time also, the modern nationalist intelligentsia
of Maharashtra supported the peasants' cause. Changed Nature of Peasant Movements after 1857.
Peasants emerged as the main force in agrarian movements, fighting directly for their own demands.

Q6. In which year Cotton Supply Association was founded?


a) 1863 b) 1867 c) 1873 d) 1857

Answer: D

In 1857 the Cotton Supply Association was founded.

Q7. Who was the Governor General of Bengal when the Permanent Settlement was introducing
in 1793?
A. Warren Hastings B. The Charles Cornwallis
C. The Lord Minto D. The Lord Auckland

Answer: B

Charles Cornwallis (1738-1805), was the commander of the British forces during the American
War of Independence and the Governor General of Bengal when the Permanent Settlement was
introduced there in 1793.

Q8. Consider the following events:


(Past prelims)
1. Indigo Revolt
2. Santhal Rebellion
3. Deccan Riot
4. Mutiny of the Sepoys

The correct chronological sequence of these events is:


Google it:- https://upscpdf.com
www.achieveias.co.in, YouTube Channel: http://youtube.com/c/AchieveIAS Telegram Channel: http://t.me/Achieve_Ias,
Mail: achieveias21@gmail.com, Contact Number: 8968920720
https://upscpdf.com https://upscpdf.com https://upscpdf.com
(a) 4, 2, 1, 3 (b) 4, 2, 3, 1 (c) 2, 4, 3, 1 (d) 2, 4, 1, 3

Answer: D

Indigo revolt – 1860; Santhal rebellion – 1855-56; Deccan riot – 1875; Mutiny of the Sepoys - 1857

Q9. Consider the following statements:


(1). As per customary norms the interest charged on loan should be half of the principal amount.
(2). By the 1770s, the rural economy in Bengal was in crisis, with recurrent famines and declining
agricultural output.

Which of the following statement(s) is/are correct?


(a) Only 1 (b) Only 2 (c) Both of them (d) None of the above

Answer: B

A variety of customary norms regulated the relationship between the moneylender and the ryot. One
general norm was that the interest charged could not be more than the principal. This was meant to
limit the moneylender’s exactions and defined what could be counted as “fair interest”. Under colonial
rule this norm broke down. In one of the many cases investigated by the Deccan Riots Commission,
the moneylender had charged over Rs 2,000 as interest on a loan of Rs 100. In petition after petition,
ryots complained of the injustice of such exactions and the violation of custom. By the 1770s, the rural
economy in Bengal was in crisis, with recurrent famines and declining agricultural output. Officials felt
that agriculture, trade and the revenue resources of the state could all be developed by encouraging
investment in agriculture. This could be done by securing rights of property and permanently fixing the
rates of revenue demand.

Q10. Consider the following statements:


1. As the Civil War ended in America , cotton production in America revived however Indian cotton
exports to Britain increased due to its quality.
2. Ryots in Bengal always cultivate the land directly, but never leased it out to under-ryots.

Which of the following statement(s) is/are not correct?


(A) Only 1 (B) Only 2 (C) Both of them (D) None of the above

Answer: C

During cotton boom, cotton merchants in India had visions of capturing the world market in raw
cotton, permanently displacing America. The editor of the Bombay Gazette had asked in 1861,
“What can prevent India from supplanting the Slave States (of U.S.A.) as the feeder of
Lancashire?” By 1865 these dreams were over. As the Civil War ended, cotton production in
America revived and Indian cotton exports to Britain steadily declined. Usually Ryots in Bengal
leased the land to under-ryots.

Google it:- https://upscpdf.com


www.achieveias.co.in, YouTube Channel: http://youtube.com/c/AchieveIAS Telegram Channel: http://t.me/Achieve_Ias,
Mail: achieveias21@gmail.com, Contact Number: 8968920720A
https://upscpdf.com https://upscpdf.com https://upscpdf.com
ACHIEVE IAS HISTORY MCQ SERIES, DAY 28, SOLUTIONS

Q1. Consider the following statements:


1. Governor General Lord Dalhousie described the kingdom of Poona as “a cherry that will drop into
our mouth one day
2. Satara was first princely state to be annexed under Doctrine of lapse.

Which of the following is/are correct?


(a) 1 only (b) 2 only (c) All of the above (d) None of the above.

Answer: B

In 1851 Governor General Lord Dalhousie described the kingdom of Awadh as “a cherry that will drop
into our mouth one day”. Five years later, in 1856, the kingdom was formally annexed to the British
Empire. Satara state was a short-lived Princely state created by the British in 1818 after the Third Anglo-
Maratha War and annexed by them in 1849 using the Doctrine of lapse. The state was ruled by
descendants of Chhatrapati Shivaji, the founder of the Maratha Empire. At the time of its adoption, the
British East India Company had imperial administrative jurisdiction over wide regions of the
subcontinent. The company took over the princely states of Satara (1848), Jaitpur and Sambalpur
(1849), Nagpur and Jhansi (1854), Tore and Arcot (1855) and Udaipur (Chhattisgarh) under the terms
of the doctrine of lapse. Oudh (1856)

Q2. .“The Sepoys Mutiny and the rebellion of 1857” written by:
A. Karl Marx B. Syed Ahmad Khan C. R. C. Mazumdar D. S. N. Sen

Answer: C

Explanation: Sepoys were the Indian soldiers under the command of British, the book “The Sepoys
Mutiny and the rebellion of 1857” written by R. C. Mazumdar.

Q3.Who was the prominent leader in Delhi during the Revolt of 1857?
A. Begum Hazrat Mahal B. Rani Laxmi Bai
C. Kunwar Singh D. Bahadur Shah Zafar

Answer: D

Explanation: In Delhi, Bahadur Shah Zafar, then the Mughal ruler, was the prominent rebel leader,
suppressed by John Nicholas.

Q4. With which uprising is Mangal Pandey associated?


(a) Barrackpur (b) Delhi (c) Meerut (d) Berhampur

Answer: A

The Mangal Pandey is associated with Barrackpur uprising. He started uprising on 29 March 1857 in
Barrackpur Cantonment.
Google it:- https://upscpdf.com
www.achieveias.co.in, YouTube Channel: http://youtube.com/c/AchieveIAS Telegram Channel: http://t.me/Achieve_Ias,
Mail: achieveias21@gmail.com, Contact Number: 8968920720A
https://upscpdf.com https://upscpdf.com https://upscpdf.com
Q5. In which of the centre of revolt resistance to the British lasted longest in 1857 Revolt:
(a) Delhi (b) Awadh (c) Jhansi (d) Bihar

Answer: B

The administrative structures established by the rebels were primarily aimed at meeting the demands
of war. However, in most cases these structures could not survive the British on slaught. But in Awadh,
where resistance to the British lasted longest, plans of counter-attack were being drawn up by the
Lucknow court and hierarchies of command were in place as late as the last months of 1857 and the
early part of 1858.

Q6. During 1857 revolt which proclamation criticized the attitude of British towards native public
servant?
a) Delhi proclamation b) Lucknow proclamation
c) Azamgarh proclamation d) Kanpur proclamation

Answer: D

Azamgarh proclamation criticized the attitude of British towards native public servant.
Section (iii) of proclamation was regarding public servant. It is not a secret thing, that under the British
Government, natives employed in the civil and military services have little respect, low pay, and no
manner of influence; and all the posts of dignity and emolument in both the departments are exclusively
bestowed on Englishmen. Therefore, all the natives in the British service ought to be alive to their
religion and interest, and abjuring their loyalty to the English, side with the Badshahi Government, and
obtain salaries of 200 and 300 rupees a month for the present, and be entitled to high posts in the
future.

Q7. After the Revolt of 1857 why Governor General Canning was widely criticized in Britain?
A. Because he failed to control the revolt in its initial phase.
B. Because of his barbarism towards Indians.
C. Because he revoked doctrine of lapse.
D. Because of his policy of Clemency towards Indian sepoys.

Answer: D
At a time when the clamour was for vengeance, pleas for moderation were ridiculed. When Governor
General Canning declared that a gesture of leniency and a show of mercy would help in winning back
the loyalty of the sepoys, he was mocked in the British press. In one of the cartoons published in the
pages of Punch, a British journal of comic satire, Canning is shown as a looming father figure, with his
protective hand over the head of a sepoy who still holds an unsheathed sword in one hand and a dagger
in the other, both dripping with blood (Fig.11.17) – an imagery that recurs in a number of British pictures
of the time.

Q8. Which one of the following territories was not affected by the Revolt of 1857 ?
(a) Jhansi (b) Chittor (c) Jagdishpur (d) Lucknow

Google it:- https://upscpdf.com


www.achieveias.co.in, YouTube Channel: http://youtube.com/c/AchieveIAS Telegram Channel: http://t.me/Achieve_Ias,
Mail: achieveias21@gmail.com, Contact Number: 8968920720A
https://upscpdf.com https://upscpdf.com https://upscpdf.com
Answer: B
Jhansi - Rani Laxmibai;
Lucknow - Begum Hazrat Mehal;
Jagdishpur (Bihar)- Kunwar Singh.

Most of the Punjab, Rajputana (Chittor) and Kashmir remained peaceful Nizam. of Hyderabad, Gulab
Singh of Kashmir, Sikh rulers of Patiala, Nabha and Jind, Holkar of Indore, Scindia of Gwalior, Nawab
of Bhopal, Rulers of Tehri and Tikargarh in order to retain their position and state, helped the British to
suppress the revolt of 1857.

Q9. Exile of which ruler was observed as “The life was gone out of the body”?
(a) Rani Laxmibai (b) Nana Saheb
(c) Bahadur Shah Zafar (d) Nawab Wajid Ali Shah

Answer: D
Lord Dalhousie’s annexations created disaffection in all the areas and principalities that were annexed
but nowhere more so than in the kingdom of Awadh in the heart of North India. Here, Nawab Wajid Ali
Shah was dethroned and exiled to Calcutta on the plea that the region was being misgoverned. The
British government also wrongly assumed that Wajid Ali Shah was an unpopular ruler. On the contrary,
he was widely loved, and when he left his beloved Lucknow, there were many who followed him all the
way to Kanpur singing songs of lament. The widespread sense of grief and loss at the Nawab’s exile
was recorded by many contemporary observers. One of them wrote: “The life was gone out of the body,
and the body of this town had been left lifeless … there was no street or market and house which did
not wail out the cry of agony in separation of Jan-i-Alam.

Q10. Siege of Lucknow became a story of survival, heroic resistance and the ultimate triumph
of British power. Who was the Commander of British forces in India who successfully rescued
the besieged British garrison?
(A) Henry Harelock (B) Colin Campbell
(C) Colonel Inglis (D) James Outram

Answer: B

Colin Campbell was appointed as the new Commander of British forces in India, came with his forces
and rescued the besieged British garrison. In British accounts the siege of Lucknow became a story of
survival, heroic resistance and the ultimate triumph of British power.

Google it:- https://upscpdf.com


www.achieveias.co.in, YouTube Channel: http://youtube.com/c/AchieveIAS Telegram Channel: http://t.me/Achieve_Ias,
Mail: achieveias21@gmail.com, Contact Number: 8968920720
https://upscpdf.com https://upscpdf.com https://upscpdf.com
ACHIEVE IAS HISTORY MCQ SERIES, DAY 29, SOLUTIONS
Q1. Consider the following statements :
1. Pitched Roof is term used by Architect to describe a plain roof.
2. Bombay Municipal Corporation Building is fusion of Oriental gothic design.

Which of the following is/are correct?


(a) 1 only (b) 2 only (c) All of the above (d) None of the above.

Answer: B

Pitched roof is a term used by architects to describe a sloping roof. By the early twentieth century
pitched roofs became less common in bungalows, although the general plan remained the same. The
Municipal Corporation Building, Bombay, designed by F. W. Stevens in 1888, this became perfect
example of fusion of Oriental and Gothic designs.

Q2. Which of the following was called 'Nursery of Benagal Army'?


(a) Calcutta (b) Murshidabad
(c) Meerut (d) Awadh

Answer: D

The large majority of the sepoys of the Bengal Army were recruited from the villages of Awadh and
eastern Uttar Pradesh. Many of them were Brahmins or from the “upper” castes. Awadh was, in fact,
called the “nursery of the Bengal Army’’.

Q3. From which of the following 1857 rebels term " Chaurasee des" is associated?
(a) Maulvi Ahmadullah (b) Shah Mal
(c) Kunwar Singh (d) General Bakht Khan

Answer: B

Shah Mal lived in a large village in pargana Barout in Uttar Pradesh. He belonged to a clan of Jat
cultivators whose kinship ties extended over chaurasee des (eighty-four villages). The lands in the
region were irrigated and fertile, with rich dark loam soil. Many of the villagers were prosperous and
saw the British land revenue system as oppressive: the revenue demand was high and its collection
inflexible. Consequently cultivators were losing land to outsiders, to traders and moneylenders who
were coming into the area. Shah Mal mobilised the headmen and cultivators of chaurasee des, moving
at night from village to village, urging people to rebel against the British.

Q4. The term 'Saracen' was designated to?


(a) European (b) Muslim (c) Hindu (d) African

Answer: B

Google it:- https://upscpdf.com


www.achieveias.co.in, YouTube Channel: http://youtube.com/c/AchieveIAS Telegram Channel: http://t.me/Achieve_Ias,
Mail: achieveias21@gmail.com, Contact Number: 8968920720
https://upscpdf.com https://upscpdf.com https://upscpdf.com
Towards the beginning of the twentieth century a new hybrid architectural style developed which
combined the Indian with the European. This was called Indo-Saracenic. “Indo” was shorthand for
Hindu and “Saracen” was a term Europeans used to designate Muslim.

Q5. What does Ganj mean?


(a) Big market in country side (b) Small fixed - market.
(c) Residential area of Agra (d) Park in urban settlement.

Answer: B

Ganj refers to a small fixed market. Both qasbah and ganj dealt in cloth, fruit, vegetables and milk
products. They provided for noble families and the army.

Q6. According to Colonial government which of the following was considered as 'Safe Heaven
Away From Epidemics'.
a) Civil lines b) Port cities.
c) Presidency towns d) Hill station.

Answer: D

Hill stations became strategic places for billeting troops, guarding frontiers and launching campaigns
against enemy rulers. The temperate and cool climate of the Indian hills was seen as an advantage,
particularly since the British associated hot weather with epidemics.

Q7. On what basis, Towns were labelled as 'Black and White' ?


A. Religious Discrimination B. Economic Discrimination
C. Racial Discrimination D. Social Discrimination

Answer: C

Indian merchants, artisans and other workers who had economic dealings with European merchants
lived outside these forts in settlements of their own. Thus, from the beginning there were separate
quarters for Europeans and Indians, which came to be labelled in contemporary writings as the “White
Town” and “Black Town” respectively. Once the British captured political power these racial distinctions
became sharper.

Q8. Consider the following statements:


1. Dhaka
2. Madras
3. Surat

Which of these commercial centre declined during colonial rule?

(a) 1 and 2 only (b) 2 only (c) 1 and 3 only (d) None of these

Answer B
Google it:- https://upscpdf.com
www.achieveias.co.in, YouTube Channel: http://youtube.com/c/AchieveIAS Telegram Channel: http://t.me/Achieve_Ias,
Mail: achieveias21@gmail.com, Contact Number: 8968920720
https://upscpdf.com https://upscpdf.com https://upscpdf.com
From the mid-eighteenth century, there was a new phase of change. Commercial centres such as
Surat, Masulipatnam and Dhaka, which had grown in the seventeenth century, declined when trade
shifted to other places.

Q9. Consider the following statements :


(1). People believed that enquiries were being conducted to impose new taxes.
(2). Upper-caste people were unwilling to give any information regarding the women of their household.

Which of the following statement(s) is/are not correct?


(a) 1 Only (b) 2 Only (c) Both of them (d) None of the above

Answer: D

For a long while they were suspicious of census operations and believed that enquiries were being
conducted to impose new taxes. Upper-caste people were also unwilling to give any information
regarding the women of their household: women were supposed to remain secluded within the interior
of the household and not subjected to public gaze or public enquiry.

Q10. Consider the following statements:


1. Mirzapur developed to cotton trade when railway line was made to Bombay which actually faciliated
in trade.
2. Bareilly declined due to introduction of Railways.

Which of the following statement(s) is/are correct?


(A) 1 Only (B) 2 Only (C) Both of them (D) None of the above

Answer: D

Mirzapur on the Ganges, which specialised in collecting cotton and cotton goods from the Deccan,
declined when a railway link was made to Bombay. With the expansion of the railway network, railway
workshops and railway colonies were established. Railway towns like Jamalpur, Waltair and Bareilly
developed.

Google it:- https://upscpdf.com


www.achieveias.co.in, YouTube Channel: http://youtube.com/c/AchieveIAS Telegram Channel: http://t.me/Achieve_Ias,
Mail: achieveias21@gmail.com, Contact Number: 8968920720
https://upscpdf.com https://upscpdf.com https://upscpdf.com
ACHIEVE IAS HISTORY MCQ SERIES, DAY 30, SOLUTIONS

Q1. Consider the following statements :


1. During 18th century of Colonial rule, pre-colonial commercial centre declined.
2. In pre-colonial town separation between town and country was fluid.

Which of the following is/are correct?


(a) 1 only (b) 2 only (c) All of the above (d) None of the above.

Answer: C

In most parts of the Western world, modern cities emerged during and due to Industrialisation. In
Britain, for example, industrial cities like Leeds and Manchester grew rapidly in the 19th and 20th
centuries. In the late 18th century, Calcutta, Bombay, and Madras rose in importance as Presidency
cities, centres of British power in India. At the same time, historically important ports and cities such
as Machilipatnam, Surat, and Seringapatam declined. The historic Mughal imperial city of Delhi also
became a dusty provincial town in the 19th century before it was rebuilt as the capital of British India.

Towns were often defined in opposition to rural areas. They came to represent specific forms of
economic activities and cultures. In the countryside peoplesubsisted by cultivating land, foraging in
the forest, or rearing animals. Towns by contrast were peopled with artisans, traders, administrators
and rulers. Towns dominated over the rural population, thriving on the surplus and taxes derived from
agriculture. Towns and cities were often fortified by walls which symbolised their separation from the
countryside. However, the separation between town and country was fluid. Peasants travelled long
distances on pilgrimage, passing through towns; they also flocked to towns during times of famine

Q2. Which of the following monument was built in Gujarati style to welcome King George V in
1911?
(a) National library of India, Kolkata.
(b) Viceroy's House (Rashtrapati bhawan), Delhi
(c) India gate, Delhi
(d) Gateway of India, Mumbai

Answer: D

Towards the beginning of the twentieth century a new hybrid architectural style developed which
combined the Indian with the European. This was called Indo-Saracenic. “Indo” was shorthand for
Hindu and “Saracen” was a term Europeans used to designate Muslim. The inspiration for this style
was medieval buildings in India with their domes, chhatris, jalis, and arches. By integrating Indian and
European styles in public architecture the British wanted to prove that they were legitimate rulers of
India. The Gateway of India, built in the traditional Gujarati style to welcome King George V and
Queen Mary to India in 1911.

Q3. The three Presidency cities Madras, Calcutta, Bombay were originally:
(a) Fishing and weaving villages (b) Commercial cities.
Google it:- https://upscpdf.com
www.achieveias.co.in, YouTube Channel: http://youtube.com/c/AchieveIAS Telegram Channel: http://t.me/Achieve_Ias,
Mail: achieveias21@gmail.com, Contact Number: 8968920720
https://upscpdf.com https://upscpdf.com https://upscpdf.com
(c) Power centre of native rulers (d) Abandoned locations.

Answer: A

The three big colonial cities Madras (Chennai), Calcutta (Kolkata) and Bombay (Mumbai), all three
were originally fishing and weaving villages. Bombay was given to the Company in 1661 by the
English king, who had got it as part of his wife’s dowry from the king of Portugal. The Company
established trading and administrative offices in each of these settlements.

Q4. Which of the following town was not an important centre of Mughal imperial
administration and control?
(a) Delhi
(b) Agra
(c) Lahore
(d) Islamabad

Answer: D

During the sixteenth and seventeenth centuries the towns built by the Mughals were famous for their
concentration of populations, their monumental buildings and their imperial grandeur and wealth.
Agra, Delhi and Lahore were important centres of imperial administration and control

Q5. Writers' Building which later became a government office was situated in?
(a) Delhi (b) Calcutta (c) Bombay (d) Madras

Answer: B

The Writers' Building, often shortened to just Writers', is the secretariat building of the State
Government of West Bengal in India. It is located in West Bengal's capital city of Kolkata.

Q6. 'Amar Katka' serialised between 1910, 1913 was based on?
a) Social problems faced by women.
b) Economic issues of peasants.
c) Life of middle class in urban town.
d) Love stories of Indian elite class couples.

Answer: A

Binodini Dasi (1863-1941) was a pioneering figure in Bengali theatre in the late nineteenth and early
twentieth centuries and worked closely with the dramatist and director Girish Chandra Ghosh (1844-
1912). She was one of the prime movers behind the setting up of the Star Theatre (1883) in Calcutta
which became a centre for famous productions. Between 1910 and 1913 she serialised her
autobiography, Amar Katha (My Story).

Q7. Which incident changed the British attitude towards planning of cities?
Google it:- https://upscpdf.com
www.achieveias.co.in, YouTube Channel: http://youtube.com/c/AchieveIAS Telegram Channel: http://t.me/Achieve_Ias,
Mail: achieveias21@gmail.com, Contact Number: 8968920720
https://upscpdf.com https://upscpdf.com https://upscpdf.com
A. Siege of Calcutta B. Deccan Riots.
C. Revolt of 1857 D. Industrial Revolution.

Answer: C

The nature of the colonial city changed further in the mid-nineteenth century. After the Revolt of 1857
British attitudes in India were shaped by a constant fear of rebellion. They felt that towns needed to
be better defended, and white people had to live in more secure and segregated enclaves, away from
threat of the “natives”.

Q8. Who was/were the architect(s) of Delhi?


A. Sir Herbert Baker B. Sir Edwin Lutyens
C. Sobha Singh D. Both A & B

Answer: D

The two british architects Sir Herbert Baker and Sir Edwin Lutyens had executed the planning and
architect of Delhi the contract of building the city was bestowed on Sobha Singh.

Q9. Consider the following statements :


(1) To keep track of life in the growing cities colonial rule carried out regular surveys, gathered
statistical data, and published various official reports.
(2) Colonial rule did not emphasize on mapping.

Which of the following statement(s) is/are correct?


(a) 1 Only (b) 2 Only (c) Both (d) None of the above

Answer: A

Colonial rule was based on the production of enormous amounts of data. The British kept detailed
records of their trading activities in order to regulate their commercial affairs. To keep track of life in
the growing cities, they carried out regular surveys, gathered statistical data, and published various
official reports. From the early years, the colonial government was keen on mapping. It felt that good
maps were necessary to understand the landscape and know the topography.

Q10. Consider the following statements :


1. Indians were not allowed to hold any position in Municipal Corporation.
2. Census Data became invaluable source for studying urbanisation in India.

Which of the following statement(s) is/are not correct?


(A) Only 1 (B) Only 2 (C) Both (D) None of the above

Answer: A

From the late nineteenth century the British tried to raise money for administering towns through the
systematic annual collection of municipal taxes. To avoid conflict they handed over some

Google it:- https://upscpdf.com


www.achieveias.co.in, YouTube Channel: http://youtube.com/c/AchieveIAS Telegram Channel: http://t.me/Achieve_Ias,
Mail: achieveias21@gmail.com, Contact Number: 8968920720
https://upscpdf.com https://upscpdf.com https://upscpdf.com
responsibilities to elected Indian representatives. The growth of cities was monitored through regular
headcounts. By the mid-nineteenth century several local censuses had been carried out in different
regions. The first all-India census was attempted in 1872. Thereafter, from 1881, decennial
(conducted every ten years) censuses became a regular feature. This collection of data is an
invaluable source for studying urbanisation in India.

Google it:- https://upscpdf.com


www.achieveias.co.in, YouTube Channel: http://youtube.com/c/AchieveIAS Telegram Channel: http://t.me/Achieve_Ias,
Mail: achieveias21@gmail.com, Contact Number: 8968920720
https://upscpdf.com https://upscpdf.com https://upscpdf.com
ACHIEVE IAS HISTORY MCQ SERIES, DAY 31, SOLUTIONS

Q1. In which of the following session of Indian National Congress, Mahatma Gandhi had said,
"Gandhi may die but Gandhism will remain forever"?
(a) Calcutta Session 1928 (b) Lahore Session 1929
(c) Karachi Session 1931 (d) Ramgarh Session 1940

Answer: C

In the Karachi Session 1931 of India National Congress, Mahatma Gandhi had said "Gandhi may die
but Gandhism will remain forever."

Q2. Who was the political mentor of Mahatama Gandhi?


(a) Bal Gangadhar Tilak (b) Dadabhai Naoroji
(c) Gopal Krishna Gokhale (d) Surendranath Banerjee

Answer: C

“Moderates” who preferred a more gradual and persuasive approach. Among these Moderates was
Gandhiji’s acknowledged political mentor, Gopal Krishna Gokhale, as well as Mohammad Ali Jinnah,
who, like Gandhiji, was a lawyer of Gujarati extraction trained in London. On Gokhale’s advice,
Gandhiji spent a year travelling around British India, getting to know the land and its peoples.

Q3. Which struggle against British made Mahatama Gandhi a truly national leader?
(a) Champaran Satyagraha (b) Rowlatt Satyagraha
(c) Kheda Satyagraha (d) Non-cooperation Movement.

Answer: B

Gandhiji called for a countrywide campaign against the “Rowlatt Act”. The situation in the province
grew progressively tenser, reaching a bloody climax in Amritsar in April 1919, when a British Brigadier
ordered his troops to open fire on a nationalist meeting. More than four hundred people were killed in
what is known as the Jallianwala Bagh massacre. It was the Rowlatt satyagraha that made Gandhiji a
truly national leader. Emboldened by its success, Gandhiji called for a campaign of “non-cooperation”
with British rule.

Q4. Which was the first Congress Session to be presided by Jawaharlal Nehru?
(a) Faizpur Session, 1936 (b) Karanchi Session, 1931
(c) Calcutta Session, 1928 (d) Lahore Session, 1929

Answer: D

In the end of December 1929, the Congress held its annual session in the city of Lahore. The meeting
was significant for two things: the election of Jawaharlal Nehru as President, signifying the passing of
the baton of leadership to the younger generation
Google it:- https://upscpdf.com
www.achieveias.co.in, YouTube Channel: http://youtube.com/c/AchieveIAS Telegram Channel: http://t.me/Achieve_Ias,
Mail: achieveias21@gmail.com, Contact Number: 8968920720
https://upscpdf.com https://upscpdf.com https://upscpdf.com
Q5. Which of the following was immediate reason for choosing salt as a symbol of struggle in
Dandi March?
(a) Salt was expensive
(b) Salt was a traditional food item in India.
(c) Government had monopoly in salt manufacturing.
(d) People's sentiments were associated with salt.

Answer: C

Soon after the observance of this “Independence Day”, Mahatma Gandhi announced that he would
lead a march to break one of the most widely disliked laws in\ British India, which gave the state a
monopoly in the manufacture and sale of salt. His picking on the salt monopoly was another
illustration of Gandhiji’s tactical wisdom. For in every Indian household, salt was indispensable; yet
people were forbidden from making salt even for domestic use, compelling them to buy it from shops
at a high price. The state monopoly over salt was deeply unpopular; by making it his target, Gandhiji
hoped to mobilise a wider discontent against British rule.

Q6. Who was the Viceroy of India during Salt March?


a) Lord Reading b) Lord Irwin c) Lord Willingdon d) Lord Linlithgow

Answer: B

The Irwin Declaration of October 1929 committed Britain to eventual Dominion Status for India.
Despite such a policy having been implicit for a decade, the Declaration was denounced by many on
the Tory Right New Delhi Conference of December 1929 between Irwin and the Indian leaders failed
to reach agreement. Gandhi now began a campaign of civil disobedience with a view to achieving
complete independence

Q7.From where Mahatama Gandhi started his journey toward the ocean to break Salt law?
A. Rajabai clock tower, Bombay B. Kheda, Gujarat
C. Sabarmati Ashram, Gujarat D. Belur Math, Kolkata

Answer: C

On 12 March 1930, Gandhiji began walking from his ashram at Sabarmati towards the ocean. He
reached his destination three weeks later, making a fistful of salt as he did and thereby making
himself a criminal in the eyes of the law. Meanwhile, parallel salt marches were being conducted in
other parts of the country.

Q8. Which one of the following was a very significant aspect of the Champaran Satyagraha?
A. Active all-India participation of lawyers, students and women in the National Movement
B. Active involvement of Dalit and Tribal communities of India in the National Movement
C. Joining of peasant unrest to India's National Movement.
D. Drastic decrease in the cultivation of plantation crops and commercial crops.
Google it:- https://upscpdf.com
www.achieveias.co.in, YouTube Channel: http://youtube.com/c/AchieveIAS Telegram Channel: http://t.me/Achieve_Ias,
Mail: achieveias21@gmail.com, Contact Number: 8968920720
https://upscpdf.com https://upscpdf.com https://upscpdf.com
Answer: C

India's first Civill Disobedience Movement, Champaran Satyagraha, was launched by Mahatma
Gandhi to protest against the injustice meted out to tenant farmers in Champaran district of Bihar. It is
widely regarded as the place where Gandhi made his first experiments in Satyagraha and then
replicated them elsewhere. During British rule, many tenant farmers were forced to growindigo on
part of their land, often working under oppressive conditions. This indigo was used to make dye. But
the demand for indigo dropped when the Germans invented a cheaper artificial dye. However, during
the First World War, the German dye ceased to be available and indigo once more became profitable
for the British. Many tenants were forced again into indigo cultivation - required by their lease under
British law. This led to anger and resentment among the tenants, with several alleging that the
landlords were using strong- arm tactics. A farmer named Raj Kumar Shukla appealed to Gandhiji to
organize the struggle to save the plantation workers. Shukla's persuasion paid off and Gandhi visited
the district in April 1917 and Satyagraha started. The main objective of the Satyagraha was to create
awakening among the peasants against the European planters.

Q9. With reference to Rowlatt Satyagraha, which of the following statements is / are correct?
(1). The Rowlatt Act was based on the recommendations of the 'Sedition 5. Committee
(2). In Rowlatt Satyagraha, Gandhiji tried to utilize the Home Rule League.
(3). Demonstrations against the arrival of Simon Commission coincided with Rowlatt Satyagraha

Select the correct answer using the code given below:


(a) 1 only (b) 2 and 3 only (c) 1 and 2 only (d) All of the above

Answer: B

The Rowlatt committee was a Sedition Committee appointed in 1918 by the British Indian
Government with Mr Justice Rowlatt, an English judge, as its president. The purpose of the
committee was to evaluate political terrorism in India, especially Bengal and Punjab, its impact, and
the links with the German government and the Bolsheviks in Russia. Rowlatt Act was a legislative act
passed by the Imperial Legislative Council in Delhi on March 18, 1919 on the recommendations of the
Sedition Committee. Hence, statement 1 is correct. In organizing Rowlatt Satyagraha, Gandhiji tried
to utilize three types of political networks- the Home Rule League, certain Pan-Islamist groups, and a
Satyagraha Sabha which he himself started at Bombay. Hence, statement 2 is also correct. The
Simon Commission arrived in India in 1928 to inquire into the causes of the rise of revolutionary
activities in 1920s, widespread distress caused by falling agricultural prices, the general
dissatisfaction of the people of India with the Govt. ds of India Act 1919. It was greeted with the
slogan 'Simon Go Back. Demonstrations against the Commission were held all over India supported
by all parties including the Congress and the Muslim League. These demonstrations did not coincide
with Rowlatt Satyagraha. Hence, statement 3 is not correct.

Q10. Who addressed Gandhiji as the "Father of the Nation" for the first time?
(a) Subhash Chandra Bose (b) Rabindranath Tagore
(c) Sardar Vallabhbhai Patel (d) Pandit Jawaharlal Nehru

Google it:- https://upscpdf.com


www.achieveias.co.in, YouTube Channel: http://youtube.com/c/AchieveIAS Telegram Channel: http://t.me/Achieve_Ias,
Mail: achieveias21@gmail.com, Contact Number: 8968920720
https://upscpdf.com https://upscpdf.com https://upscpdf.com
Answer: A

Subhash Chandra Bose addressed Gandhiji as the "Father of the Nation" for the first time. Mahatma
Gandhiji is revered in India as the Father of the Nation. Much before the Constitution of Free India
conferred the title of the Father of the Nation upon the Mahatma, it was Netaji Subhash Chandra
Bose who first addressed him as such in his condolence message to the Mahatma on the demise of
Kasturba.

Google it:- https://upscpdf.com


www.achieveias.co.in, YouTube Channel: http://youtube.com/c/AchieveIAS Telegram Channel: http://t.me/Achieve_Ias, Mail:
achieveias21@gmail.com, Contact Number: 8968920720
https://upscpdf.com https://upscpdf.com https://upscpdf.com
ACHIEVE IAS HISTORY MCQ SERIES, DAY 32, SOLUTIONS

Q1. Consider the following facts about Mahatma Gandhi:


1. In January 1917, Mohandas Karamchand Gandhi returned to his homeland after two decades of
residence abroad.
2. Mahatma Gandhi went South Africa as a lawyer, and became a leader of the Indian community in
that territory.
3. It was in South Africa that Mahatma Gandhi first forged the distinctive techniques of non-violent
protest known as satyagraha, first promoted harmony between religions, and first alerted upper-caste
Indians to their discriminatory treatment of low castes and women.

Which of the following is/are correct?


A. Only I B. I and II C. II and III D. All of the above

Answer: C

In January 1915, Mohandas Karamchand Gandhi returned to his homeland after two decades of
residence abroad. These years had been spent for the most part in South Africa, where he went as a
lawyer, and in time became a leader of the Indian community in that territory. As the historian
Chandran Devanesan has remarked, South Africa was “the making of the Mahatma”. It was in South
Africa that Mahatma Gandhi first forged the distinctive techniques of non-violent protest known as
satyagraha, first promoted harmony between religions, and first alerted upper-caste Indians to their
discriminatory treatment of low castes and women.

Q2. Consider the following statements:


(a) Mahatama Gandhi and Jawaharlal Nehru promised Congress support to the war effort if the
British, in return, promised to grant India independence once hostilities ended.
(b) Congress organised a series of individual satyagrahas to pressure the rulers to accept Congress
as representative of whole India.

Which of the following is/are correct?


A. Only I B. Only II C. Both D. All of the above

Answer: A

In September 1939, two years after the Congress ministries assumed office, the Second World War
broke out. Mahatma Gandhi and Jawaharlal Nehru had both been strongly critical of Hitler and the
Nazis. Accordingly, they promised Congress support to the war effort if the British, in return, promised
to grant India independence once hostilities ended. The offer was refused. In protest, the Congress
ministries resigned in October 1939. Through 1940 and 1941, the Congress organised a series of
individual satyagrahas to pressure the rulers to promise freedom once the war had ended.

Q3. "Direct Action Day" is associated with?


(a) Congress demand for complete independence.
Google it:- https://upscpdf.com
www.achieveias.co.in, YouTube Channel: http://youtube.com/c/AchieveIAS Telegram Channel: http://t.me/Achieve_Ias, Mail:
achieveias21@gmail.com, Contact Number: 8968920720
https://upscpdf.com https://upscpdf.com https://upscpdf.com
(b) Muslim league's demand for Pakistan.
(c) British' plan for granting independence to India.
(d) British' plan for suppressing Indian National Movement.

Answer: B

A Cabinet Mission sent in the summer of 1946 failed to get the Congress and the League to agree on
a federal system that would keep India together while allowing the provinces a degree of autonomy.
After the talks broke down, Jinnah called for a “Direct Action Day” to press the League’s demand for
Pakistan. On the designated day, 16 August 1946, bloody riots broke out in Calcutta.

Q4. Consider the following statements:


1. Political polarisation was witnessed in 1946 general elections.
2. Lord Wavell was viceroy of India during 1946 general election.

Which of the following is/are correct?


A. Only I B. Only II C. Both D. All of the above

Answer: C

Viceroy, Lord Wavell, brought the Congress and the League together for a series of talks. Early in
1946 fresh elections were held to the provincial legislatures. The Congress swept the “General”
category, but in the seats specifically reserved for Muslims the League won an overwhelming
majority. The political polarisation was complete. A Cabinet Mission sent in the summer of 1946 failed
to get the Congress and the League to agree on a federal system that would keep India together
while allowing the provinces a degree of autonomy. However in February 1947, Wavell was replaced
as Viceroy by Lord Mountbatten.

Q5. Why Quit India Movement was launched?


(a) To demand release of Political prisoner arrested for participating in individual Satyagraha.
(b) Because of Muslim league's demands for autonomy.
(c) Because of failure of Cripps Mission.
(d) Because of Anti-India attitude of British Conservative Party.

Answer: C

Conservative Prime Minister, Winston Churchill, was a diehard imperialist who insisted that he had
not been appointed the King’s First Minister in order to preside over the liquidation of the British
Empire. In the spring of 1942, Churchill was persuaded to send one of his ministers, Sir Stafford
Cripps, to India to try and forge a compromise with Gandhiji and the Congress. Talks broke down,
however, after the Congress insisted that if it was to help the British defend India from the Axis
powers, then the Viceroy had first to appoint an Indian as the Defence Member of his Executive
Council.After the failure of the Cripps Mission, Mahatma Gandhi decided to launch his third major
movement against British rule. This was the “Quit India” campaign, which began in August 1942

Google it:- https://upscpdf.com


www.achieveias.co.in, YouTube Channel: http://youtube.com/c/AchieveIAS Telegram Channel: http://t.me/Achieve_Ias, Mail:
achieveias21@gmail.com, Contact Number: 8968920720
https://upscpdf.com https://upscpdf.com https://upscpdf.com
Q6. In context of Gandhi - Irwin pact which of the following is not correct?
a) All political prisoners were released.
b) Civil Disobedience to be called off.
c) Salt manufacture was allowed by the Government.
d) Dominion Status to India was promised.

Answer: D

By the terms of “Gandhi-Irwin Pact’ - civil disobedience would be called off, all prisoners released,
and salt manufacture allowed along the coast. The pact was criticised by radical nationalists, for
Gandhiji was unable to obtain from the Viceroy a commitment to political independence for Indians;
he could obtain merely an assurance of talks towards that possible end.

Q7. Who persuaded Mahatama Gandhi not to restrict the protests to men alone?
A. Jawaharlal Nehru B. Kamala Devi Chattopadhyay
C. Sarojini Naidu D. Kasturba Gandhi

Answer: B

The socialist activist Kamaladevi Chattopadhyay had persuaded Gandhiji not to restrict the protests to
men alone. Kamaladevi was herself one of numerous women who courted arrest by breaking the salt
or liquor laws. Third, and perhaps most significant, it was the Salt March which forced upon the British
the realisation that their Raj would not last forever, and that they would have to devolve some power
to the Indians.

Q8. Consider the following statements :


The Non-Cooperation Movement led to the -
1. Congress becoming a mass movement for the first time.
2. Growth of Hindu-Muslim unity.
3. Removal of fear of the British 'might' from the minds of the people.
4. British Government's willingness to grant political concessions to Indians.

Of these statements:
(a) 1, 2, 3 and 4 are correct (b) 2 and 3 are correct
(c) 1 and 3 are correct (d) 3 and 4 are correct

Answer: A

The Non-Cooperation Movement launched in August, 1920 by Mahatma Gandhi, saw for the first
time, Hindu Muslim Unity in a big way and removed the fear of the British "might" from the minds of
the people.

Q9. Consider the following statements regarding Gandhiji’s countrywide campaign against
the “Rowlatt Act”:

Google it:- https://upscpdf.com


www.achieveias.co.in, YouTube Channel: http://youtube.com/c/AchieveIAS Telegram Channel: http://t.me/Achieve_Ias, Mail:
achieveias21@gmail.com, Contact Number: 8968920720
https://upscpdf.com https://upscpdf.com https://upscpdf.com
(1). In towns across North and West India, life came to a standstill, as shops shut down and schools
closed in response to the bandh call.
(2). The protests were particularly intense in the Punjab, where many men had served on the British
side in the War – expecting to be rewarded for their service.
(3). Gandhiji was detained while proceeding to the Punjab, even as prominent local Congressmen
were arrested.

Which of the following statement(s) is/are correct?


A. Only I B. I and II C. I and III D. All of the above

Answer: D
During the Great War of 1914-18, the British had instituted censorship of the press and permitted
detention without trial. Now, on the recommendation of a committee chaired by Sir Sidney Rowlatt,
these tough measures were continued. In response, Gandhiji called for a countrywide campaign
against the “Rowlatt Act”. In towns across North and West India, life came to a standstill, as shops
shut down and schools closed in response to the bandh call. The protests were particularly intense in
the Punjab, where many men had served on the British side in the War – expecting to be rewarded
for their service. Instead they were given the Rowlatt Act. Gandhiji was detained while proceeding to
the Punjab, even as prominent local Congressmen were arrested. The situation in the province grew
progressively tenser, reaching a bloody climax in Amritsar in April 1919, when a British Brigadier
ordered his troops to open fire on a nationalist meeting.

Q10. Consider the following statements:


1. Muslim league
2. Indian princes
3. B. R. Ambedkar

Who among them recognise Congress as representative of whole India in 2nd Round Table
Conference, 1931?
(A). Only 1 and 2 (B). Only 2 and 3
(C). Only 3 (D). None of the above

Answer: D
Second Round Table Conference was held in London in the latter part of 1931. Here, Gandhiji
represented the Congress. However, his claims that his party represented all of India came under
challenge from three parties: from the Muslim League, which claimed to stand for the interests of the
Muslim minority; from the Princes, who claimed that the Congress had no stake in their territories;
and from the brilliant lawyer and thinker B.R. Ambedkar, who argued that Gandhiji and the Congress
did not really represent the lowest castes.

Google it:- https://upscpdf.com


www.achieveias.co.in, YouTube Channel: http://youtube.com/c/AchieveIAS Telegram Channel: http://t.me/Achieve_Ias,
Mail: achieveias21@gmail.com, Contact Number: 8968920720
https://upscpdf.com https://upscpdf.com https://upscpdf.com
ACHIEVE IAS HISTORY MCQ SERIES, DAY 33, SOLUTIONS

Q1. What is the correct chronological order of movements led by Gandhi ji?
(a).Kheda Peasant Satyagraha, Khilafat and non-Co-operation Movement, Individual Civil
Disobedience started
(b). Non-Co-operation Movement, Champaran Satyagraha, Kheda Peasant Satyagraha
(c). Khilafat Movement, Kheda Peasant Satyagraha, Champaran Satyagraha
(d). Champaran Satyagraha, Khilafat Movement, Kheda Kheda Peasant Satyagraha

Answer: A

Champaran Satyagraha (1917), Kheda Peasant Satyagraha (1918), Khilafat and non-Cooperation
Movement (1920-21), Individual Civil Disobedience (1933).

Q2. Consider the following statements:


(a) In 1939 Congress Minister resigned due to rift between Swarajists and no changers.
(b) Purna Swaraj was accepted as Congress goal at the Karanchi session 1931.

Which of the following is/are correct?


A. Only I B. Only II C. Both D. None of the above

Answer: D

In September 1939, two years after the Congress ministries assumed office, the Second World War
broke out. Mahatma Gandhi and Jawaharlal Nehru had both been strongly critical of Hitler and the
Nazis. Accordingly, they promised Congress support to the war effort if the British, in return, promised
to grant India independence once hostilities ended. The offer was refused. In protest, the Congress
ministries resigned in October 1939. Purna Swaraj” was accepted as Congress goal at the Lahore
Congress (December) 1929

Q3. Why Gandhiji was not well satisfied with India's independence?
(a) Because of Drain of Wealth.
(b) Because Congress leaders disapproved to wind up.
(c) Because India got divided into two separate nation.
(d) Because Jawaharlal Nehru had a Pacificst policy towards British.

Answer: C

Mahatma Gandhi was not present at the festivities in the capital on 15 August 1947. He was in Calcutta,
but he did not attend any function or hoist a flag there either. Gandhiji marked the day with a 24-hour
fast. The freedom he had struggled so long for had come at an unacceptable price, with a nation divided
and Hindus and Muslims at each other’s throats.

Q4. Which of the following was the second Satyagrah of Gandhi ji in India?
(a). Kheda Satyagraha (b). Ahmedabad mill strike

Google it:- https://upscpdf.com


www.achieveias.co.in, YouTube Channel: http://youtube.com/c/AchieveIAS Telegram Channel: http://t.me/Achieve_Ias,
Mail: achieveias21@gmail.com, Contact Number: 8968920720
https://upscpdf.com https://upscpdf.com https://upscpdf.com
(c). Champaran Satyagraha (d). None of the above

Answer: B

Ahmedabad Mill Strike, 1918 was second movements led by Gandhi ji in India. The Mill Owners wanted
to withdraw the bonus while the workers demanded a 50% wage hike against 20% offered by the Mill
Owners.

Q5. Which of the following is correct about Mahatama Gandhi's perception about divided India?
(a) Division of nation will led to war in coming times.
(b) Minorities will suffer in both nations.
(c) Both nation will share a relation of love and friendship.
(d) Divided India will not be that strong that it could have been.

Answer: C

Mahatma Gandhi permitted himself the hope “that though geographically and politically India is divided
into two, at heart we shall ever be friends and brothers helping and respecting one another and be one
for the outside world”.

Q6. According to whom Mahatama gandhi was "an appeaser of muslims"?


a) Hindu Mahasabha b) Nathuram godse
c) K.B. Hedgewar d) V. D. Savarkar

Answer: B

Gandhiji was shot dead by a young man. The assassin, who surrendered afterwards, was a Brahmin
from Pune named Nathuram Godse, the editor of an extremist Hindu newspaper who had denounced
Gandhiji as “an appeaser of Muslims”.

Q7. A 'Bunch of old letters' was actually a collection of letters received by editor of this book.
Who is this editior?
A. Jawaharlal Nehru B. Mahatama Gandhi
C. Subhash Chandra Bose D. Md. Jinnah

Answer: A

Nehru edited a collection of letters written to him during the national movement and published a Bunch
of Old Letters.

Q8. Which Newspaper / Magazine criticised Civil Disobedience Movement?


(a) Vividh vritt (b) Time (c) Kesari (d) Sudharak

Answer: A

Google it:- https://upscpdf.com


www.achieveias.co.in, YouTube Channel: http://youtube.com/c/AchieveIAS Telegram Channel: http://t.me/Achieve_Ias,
Mail: achieveias21@gmail.com, Contact Number: 8968920720
https://upscpdf.com https://upscpdf.com https://upscpdf.com
Moderate paper Vividh Vritt pointed out the futility of the movement and opined that it could not achieve
the end in view. It, however, reminded the government that repression would defeat its purpose.

Q9. Who formed All Bengal Civil Disobedience Council?


A. Annanda Shankar Ray B. Binoy Basu
C. Subhash Chandra Bose D. J. M. Sengupta

Answer: D

Mr. J.M. Sengupta has formed an All-Bengal Civil Disobedience Council, and the Bengal Provincial
Congress Committee has formed an All Bengal Council of Disobedience. But beyond forming councils
no active steps have yet been taken in the matter of civil disobedience in Bengal.

Q10. Who among them was ideologically different from other three?
(A). Jawaharlal Nehru (B). Jayprakash Narayan
(C). Sardar Patel (D). N.G. Ranga

Answer: C

In the 1920s, Jawaharlal Nehru was increasingly influenced by socialism, and he returned from Europe
in 1928 deeply impressed with the Soviet Union. As he began working closely with the socialists
(Jayaprakash Narayan, Narendra Dev, N.G. Ranga and others), a rift developed between the socialists
and the conservatives within the Congress. After becoming the Congress President in 1936, Nehru
spoke passionately against fascism, and upheld the demands of workers and peasants. Worried by
Nehru’s socialist rhetoric, the conservatives, led by Rajendra Prasad and Sardar Patel, threatened to
resign from the Working Committee, and some prominent industrialists in Bombay issued a statement
attacking Nehru.

Google it:- https://upscpdf.com


www.achieveias.co.in, YouTube Channel: http://youtube.com/c/AchieveIAS Telegram Channel: http://t.me/Achieve_Ias,
Mail: achieveias21@gmail.com, Contact Number: 8968920720
https://upscpdf.com https://upscpdf.com https://upscpdf.com
ACHIEVE IAS HISTORY MCQ SERIES, DAY 34, SOLUTIONS

Q1. Consider the following with respect to 1937 election:


1. Muslim league captured majority of seats in North West Frontier Province.
2. Congress performed badly in constituencies reserved for muslims.

Which of the following is/are correct?


A. Only I B. Only II C. None of the above D. All of the above

Answer: B

In 1937, elections to the provincial legislatures were held for the first time. Only about 10 to 12 per
cent of the population enjoyed the right to vote. The Congress did well in the elections, winning an
absolute majority in five out of eleven provinces and forming governments in seven of them. It did
badly in the constituencies reserved for Muslims, but the Muslim League also fared poorly, polling
only 4.4 per cent of the total Muslim vote cast in this election. The League failed to win a single seat
in the North West Frontier Province (NWFP) and could capture only two out of 84 reserved
constituencies in the Punjab and three out of 33 in Sind.

Q2. Which of the following statement is not correct regarding Lucknow pact?
A. It was signed between Congress and Muslim league
B. Congress accepted separate electorate demand.
C. Extremists were readmitted to Congress.
D. Congress accepted to support Khilafat issue.

Answer: D

All are correct except D as Khilafat issue was raised after the end of World War I (1918). But
Lucknow pact was signed in 1916. So, technically option d is wrong.

Q3. Which act was considered as first step towards communal politics in India?
(a) Government of India Act 1858.
(b) Indian Council Act of 1861.
(c) Indian Council Act of 1909.
(d) Government of India Act 1919.

Answer: C

Indian Council Act of 1909 is also known as Morley Minto Reform and it was the first step towards
communal politics as for the first time separate electorate was introduced.

Q4. Why Congress rejected the offer to form coalition in United Provinces with Muslim
league?
A. Because they already had majority.
B. They do not wanted to get associated with any communal poltical party.
C. Because Muslim league supported Conservatives whereas Congress was dominated by liberals.
Google it:- https://upscpdf.com
www.achieveias.co.in, YouTube Channel: http://youtube.com/c/AchieveIAS Telegram Channel: http://t.me/Achieve_Ias,
Mail: achieveias21@gmail.com, Contact Number: 8968920720
https://upscpdf.com https://upscpdf.com https://upscpdf.com
D. Because Muslim league tended to support Landlordism.

Answer: D

In the United Provinces, the party had rejected the Muslim League proposal for a coalition
government partly because the League tended to support landlordism, which the Congress wished to
abolish, although the party had not yet taken any concrete steps in that direction.

Q5. According to whom, events of 1947 were intimately connected to the long history of
Hindu-Muslim conflict throughout medieval and modern times?
(a) Muhammad Ali Jinnah (b) Historians
(c) Jawaharlal Nehru (d) Nathuram Godse

Answer: B

Muhammad Ali Jinnah had put the theory of of saparate nation but he never stated that he never
stated that roots of this theory lies in the events happened in medieval and modern times. So, this
perception was created by Historians, both Indian and Pakistani.

Q6. Who proposed the idea of Muslim majority autonomous region with India for the first
time?
a) Muhammad Iqbal b) Chaudhary Rahmat Ali
c) Muhammad Ali Jinnah d) Liaquat Ali Khan

Answer: A

The origins of the Pakistan demand have also been traced back to the Urdu poet Mohammad Iqbal,
the writer of “Sare Jahan Se Achha Hindustan Hamara”. In his presidential address to the Muslim
League in 1930, the poet spoke of a need for a “NorthWest Indian Muslim state”. Iqbal, however, was
not visualising the emergence of a new country in that speech but a reorganisation of Muslim-
majority.

Q7. Who voiced the idea of Pakistan in his pamphlet intitled 'Now or Never; Are We to Live or
Perish Forever?
A. Chaudhary Rahmat Ali B. Khaliq-oz-zaman
C. Abul Mansur Ahmad D. Muhammad Ali Jinnah

Answer: A

Choudhary Rahmat Ali was a Pakistani nationalist who was one of the earliest proponents of the
creation of the state of Pakistan. He is credited with creating the name "Pakistan" for a separate
Muslim homeland in South Asia and is generally known as the originator of the Pakistan Movement.
Rahmat Ali's seminal contribution was when he was a law student at the University of Cambridge in
1933, in the form of a pamphlet "Now or Never; Are We to Live or Perish Forever?", also known as
the "Pakistan Declaration"

Google it:- https://upscpdf.com


www.achieveias.co.in, YouTube Channel: http://youtube.com/c/AchieveIAS Telegram Channel: http://t.me/Achieve_Ias,
Mail: achieveias21@gmail.com, Contact Number: 8968920720
https://upscpdf.com https://upscpdf.com https://upscpdf.com
Q8.Who among them came forward to restore Communal harmony during 1946-47 riots?
(a) Muhammad Jinnah (b) Lord Mountbatten
(c) Jawaharlal Nehru (d) Mahatama Gandhi

Answer: D

During riots there was a complete breakdown of authority in the city. British officials did not know how
to handle the situation: they were unwilling to take decisions, and hesitant to intervene. When panic-
stricken people appealed for help, British officials asked them to contact Mahatma Gandhi,
Jawaharlal Nehru, Vallabh Bhai Patel or M.A. Jinnah. Nobody knew who could exercise authority and
power. The top leadership of the Indian parties, barring Mahatma Gandhi, were involved in
negotiations regarding independence while many Indian civil servants in the affected provinces feared
for their own lives and property. The British were busy preparing to quit India.

Amidst the 1946-47 turmoil, one man’s valiant efforts at restoring communal harmony bore fruit. The
77-year-old Gandhiji decided to stake his all in a bid to vindicate his lifelong principle of non-violence,
and his conviction that people’s hearts could be changed. He moved from the villages of Noakhali in
East Bengal (present-day Bangladesh) to the villages of Bihar and then to the riot-torn slums of
Calcutta and Delhi, in a heroic effort to stop Hindus and Muslims kill each other, careful everywhere
to reassure the minority community. In October 1946, Muslims in East Bengal targeted Hindus.
Gandhiji visited the area, toured the villages on foot, and persuaded the local Muslims to guarantee
the safety of Hindus. Similarly, in other places such as Delhi he tried to build a spirit of mutual trust
and confidence between the two communities.

Q9. Which of the following gave the proposal of grouping Provincial Assemblies into three 3
sections A, B, C?
A. Shimla Conference, 1945 B. C. Rajagopalachari Formula.
C. Desai-Liaquat Pct D. Cabinet Mission.

Answer: D

In March 1946 the British Cabinet sent a three member mission to Delhi to examine the League’s
demand and to suggest a suitable political framework for a free India. The Cabinet Mission toured the
country for three months and recommended a loose three-tier confederation. India was to remain
united. It was to have a weak central government controlling only foreign affairs, defence and
communications with the existing provincial assemblies being grouped into three sections while
electing the constituent assembly: Section A for the Hindu majority provinces, and Sections B and C
for the Muslim-majority provinces of the north-west and the north-east (including Assam) respectively.
The sections or groups of provinces would comprise various regional units. They would have the
power to set up intermediate-level executives and legislatures of their own.

Q10. Which region of India was dominated by Unionist Party during 1923-47?
(A). NWFP (B). Sindh (C). Punjab (D). United Provinces

Answer: C

Google it:- https://upscpdf.com


www.achieveias.co.in, YouTube Channel: http://youtube.com/c/AchieveIAS Telegram Channel: http://t.me/Achieve_Ias,
Mail: achieveias21@gmail.com, Contact Number: 8968920720
https://upscpdf.com https://upscpdf.com https://upscpdf.com
Unionist party was a political party representing the interests of landholders Hindu, Muslim and Sikh –
in the Punjab. The party was particularly powerful during the period 1923-47. The party was secular in
nature and believed in a strong and united Punjabi entity, bringing together Muslims, Hindus, Sikhs
and other communities of this province.

Google it:- https://upscpdf.com


www.achieveias.co.in, YouTube Channel: http://youtube.com/c/AchieveIAS Telegram Channel: http://t.me/Achieve_Ias,
Mail: achieveias21@gmail.com, Contact Number: 8968920720
https://upscpdf.com https://upscpdf.com https://upscpdf.com
ACHIEVE IAS HISTORY MCQ SERIES, DAY 35, SOLUTIONS

Q1. Consider the following:


1. Constituent Assembly was dominated by Congress.
2. Muslim league was second most dominant party in Congress.

Which of the following is/are correct?


A. Only I B. Only II C. None of the above D. All of the above

Answer: A

Congress swept the general seats in the provincial elections, and the Muslim League captured most
of the reserved Muslim seats. But the League chose to boycott the Constituent Assembly, pressing its
demand for Pakistan with a separate constitution. The Socialists too were initially unwilling to join, for
they believed the Constituent Assembly was a creation of the British, and therefore incapable of being
truly autonomous. In effect, therefore, 82 per cent of the members of the Constituent Assembly were
also members of the Congress.

Q2. How were the members of Constituent Assembly elected?


A. On the basis of universal Franchise.
B. On the basis of limited Franchise.
C. Choosen by British Parliament.
D. Choosen by Provincial legislature.

Answer: D

The members of the Constituent Assembly were not elected on the basis of universal franchise. In the
winter of 1945-46 provincial elections were held in India. The Provincial Legislatures then chose the
representatives to the Constituent Assembly.

Q3. Bhim Rao Ambedkar was one of the prominent member of Constituent Assembly. From
which of following political party, he was associated?
(a) Congress party (b) Unionist Scheduled castes
(c) Scheduled castes federation (d) Independent candidates.

Answer: C

Bhimrao Ramji Ambedkar (14 April 1891 – 6 December 1956), also known as Babasaheb Ambedkar,
was an Indian jurist, economist, politician and social reformer, who inspired the Dalit Buddhist
movement and campaigned against social discrimination towards the untouchables (Dalits). He was
independent India's first Minister of Law and Justice, and the chief architect of the Constitution of
India. Ambedkar oversaw the transformation of his political party into the Scheduled Castes
Federation, although it performed poorly in the 1946 elections for Constituent Assembly of India.
Later he was elected into the constituent assembly of Bengal where Muslim League was in power.

Q4. Consider the following statements:


Google it:- https://upscpdf.com
www.achieveias.co.in, YouTube Channel: http://youtube.com/c/AchieveIAS Telegram Channel: http://t.me/Achieve_Ias,
Mail: achieveias21@gmail.com, Contact Number: 8968920720
https://upscpdf.com https://upscpdf.com https://upscpdf.com
(a) Jawaharlal Nehru argued for weak centre and equal share of power for states.
(b) K. Santhanam argued for story centre with no power to states.

Which of the following is/are not correct?


A. Only I B. Only II C. None of the above D. All of the above

Answer: D

One of the topics most vigorously debated in the Constituent Assembly was the respective rights of
the Central Government and the states. Among those arguing for a strong Centre was Jawaharlal
Nehru. The rights of the states were most eloquently defended by K. Santhanam from Madras. A
reallocation of powers was necessary, he felt, to strengthen not only the states but also the Centre.
“There is almost an obsession that by adding all kinds of powers to the Centre we can make it
strong.” This was a misconception, said Santhanam. If the Centre was overburdened with
responsibilities, it could not function effectively. By relieving it of some of its functions, and
transferring them to the states, the Centre could, in fact, be made stronger.

Q5. The "Objective Resolution" was moved by?


(a) Jawaharlal Nehru (b) Bhim Rao Ambedkar
(c) Sardar Vallabhbhai Patel (d) Rajendra Prasad

Answer: A

The Constituent Assembly had 300 members. Of these, six members played particularly important
roles. Three were representatives of the Congress, namely, Jawaharlal Nehru, Vallabh Bhai Patel
and Rajendra Prasad. It was Nehru who moved the crucial “Objective Resolution”

Q6. Who among them was not a part of Drafting Committee of Constituent Assempbly.
a) Dr. K M Munshi b) Dr. B R Ambedkar
c) N. Gopalaswamy Ayyangar d) Usha Nath Sen

Answer: D

During the period of British rule, Ambedkar had been a political opponent of the Congress; but, on the
advice of Mahatma Gandhi, he was asked at Independence to join the Union Cabinet as law minister.
In this capacity, he served as Chairman of the Drafting Committee of the Constitution. Serving with
him were two other lawyers, K.M. Munshi from Gujarat and Alladi Krishnaswamy Aiyar from Madras,
both of whom gave crucial inputs in the drafting of the Constitution.

Q7. Who was chosen as Constitutional Advisor to the Government of India?


A. B. N. Rau B. S. N Mukherjee
C. S. Varada Chari D. Dr. K M Munshi

Answer: A

Google it:- https://upscpdf.com


www.achieveias.co.in, YouTube Channel: http://youtube.com/c/AchieveIAS Telegram Channel: http://t.me/Achieve_Ias,
Mail: achieveias21@gmail.com, Contact Number: 8968920720
https://upscpdf.com https://upscpdf.com https://upscpdf.com
B. N. Rau was civil servant who also acted as Constitutional Advisor to the Government of India, who
prepared a series of background papers based on a close study of the political systems obtaining in
other countries.

Q8. In the post Independent India, who among the following was in support of separate
electorate?
(a) R. V. Dhulekar (b) Govind Bhallabh Pant
(c) B. Pocker Bahadur (d) Begum Aizaas Rasul

Answer: C

On 27 August 1947, B. Pocker Bahadur from Madras made a powerful plea for continuing separate
electorates. Minorities exist in all lands, argued Bahadur; they could not be wished away, they could
not be “erased out of existence”. The need was to create a political framework in which minorities
could live in harmony with others. Not all Muslims supported the demand for separate electorates.
Begum Aizaas Rasul, for instance, felt that separate electorates were selfdestructive since they
isolated the minorities from the majority.

Q9. Which of the following word was not used in proclamation of "Objective Resolution"?
A. Republic B. Democratic C. Equality D. Sovereign

Answer: B

13 December 1946: An 'Objective Resolution' was presented by Jawaharlal Nehru, laying down the
underlying principles of the constitution. In this resolution he pointed out that " The House will notice
that in this Resolution, although we have not used the word “democratic” because we thought it is
obvious that the word “republic” contains that word and we did not want to use unnecessary words
and redundant words, but we have done something much more than using the word".

Q10. Who pointed out that Constituent Assembly was British made and it is working under
British influence?
(A). M. N. Roy (B). S. V. Ghate
(C). Z. A. Ahmad (D). Somnath Lahiri

Answer: D

A Communist member, Somnath Lahiri saw the dark hand of British imperialism hanging over the
deliberations of the Constituent Assembly. He thus urged the members, and Indians in general, to
fully free themselves from the influences of imperial rule. In the winter of 1946-47, as the Assembly
deliberated, the British were still in India. An interim administration headed by Jawaharlal Nehru was
in place, but it could only operate under the directions of the Viceroy and the British Government in
London. Lahiri exhorted his colleagues to realise that the Constituent Assembly was British-made and
was “working the British plans as the British should like it to be worked out”.

Google it:- https://upscpdf.com


www.achieveias.co.in, YouTube Channel: http://youtube.com/c/AchieveIAS Telegram Channel: http://t.me/Achieve_Ias,
Mail: achieveias21@gmail.com, Contact Number: 8968920720
https://upscpdf.com https://upscpdf.com https://upscpdf.com
ACHIEVE IAS HISTORY MCQ SERIES, DAY 36, SOLUTIONS

Q1. Consider the following statements:


1. No traces of canals have been found at Harrapan site.
2. Most of the Harappan sites are located in semi-arid lands.

Which of the following is/are correct?


A. Only I B. Only II C. None of the above D. All of the above

Answer: B

Most Harappan sites are located in semi-arid lands, where irrigation was probably required for
agriculture. Traces of canals have been found at the Harappan site of Shortughai in Afghanistan

Q2. Consider the following statements:


(a) Archaeologists extrapolate that animals were not used for ploughing.
(b) Terracotta models of plough were found at Banawali but were absent in cholistan

Which of the following is/are not correct?


A. Only I B. Only II C. None of the above D. All of the above

Answer: D

Representations on seals and terracotta sculpture indicate that the bull was known, and
archaeologists extrapolate from this that oxen were used for ploughing. Moreover, terracotta
models of the plough have been found at sites in Cholistan and at Banawali (Haryana).

Q3. In which of the following sites Lapis Lajuli were found?


(a) Balakot (b) Shortughai (c) Mageshwar (d) Lothal

Answer: B

Shortughai, in far-off Afghanistan, was the best source of lapis lazuli, a blue stone that was
apparently very highly valued, and Lothal which was near sources of carnelian (from Bharuch
in Gujarat), steatite (from south Rajasthan and north Gujarat) and metal (from Rajasthan).

Q4. Mesopotamian Civilisation often used word 'Meluḫḫa'. What does 'Meluḫḫa' mean?
A. Pot B. Terracotta C. Indus coins D. Indus region

Answer: D

Meluḫḫa or Melukhkha is the Sumerian name of a prominent trading partner of Sumer during the Middle
Bronze Age. Its identification remains an open question, but most scholars associate it with the Indus
Valley Civilization.

Google it:- https://upscpdf.com


www.achieveias.co.in, YouTube Channel: http://youtube.com/c/AchieveIAS Telegram Channel: http://t.me/Achieve_Ias,
Mail: achieveias21@gmail.com, Contact Number: 8968920720
https://upscpdf.com https://upscpdf.com https://upscpdf.com
Q5. "The story of Indian Archaeology" was written by?
(a) Amlanand Ghosh (b) S N Roy (c) R S Bist (d) Y D Sharma

Answer: B

The story of Indian Archaeology was written by archaeologist S N Roy.

Q6. Regarding the Indus Valley Civilisation, consider the following statements:
1. It was predominantly a secular civilisation and the religious element, though present, did not
dominate the scene.
2. During this period, cotton was used for manufacturing textiles in India.

Which of the statements given above is/are correct?


a) Only 1 b) Only 2 c) Both d) None of the above

Answer: C

Both are correct as Indus Valley Civilisation was predominantly a secular civilisation and the religious
element, though present, did not dominate the scene and also during this period, cotton was used for
manufacturing textiles in India.

Q7. Evidences of ploughed field have been found in?


A. Kalibangan B. Rangpur C. Banawali D. Alamgirpur

Answer: A

Archaeologists have also found evidence of a ploughed field at Kalibangan (Rajasthan),


associated with Early Harappan levels. The field had two sets of furrows at right angles to
each other, suggesting that two different crops were grown together.

Q8. Which of the following animals as figures surround the male deity in the Indus Valley
Civilization?
1. Elephant
2. Tiger
3. Buffalo
4. Deer
5. Rhinoceros

(a) 1, 2, 3 and 4 are correct


(b) 2, 3 and 4 are correct
(c) 1, 3 and 5 are correct
(d) All are correct

Answer: D

Google it:- https://upscpdf.com


www.achieveias.co.in, YouTube Channel: http://youtube.com/c/AchieveIAS Telegram Channel: http://t.me/Achieve_Ias,
Mail: achieveias21@gmail.com, Contact Number: 8968920720
https://upscpdf.com https://upscpdf.com https://upscpdf.com
The 'Shiva Pashupati', believed to be the male deity of the Harappan civilization, garnered much
popularity. In a seal, he is shown seated in a yogic posture, with three visible faces, surrounded uo
paseq ortant has din Lothal by the tiger and the elephant on the right, the buffalo and rhinoceros on the
left and a horned deer appearing from under its seat.

Q9. Consider the following statements:


(1). Citadel were the lower areas of town.
(2). Important building were built on higher areas of town.

Which of the following statement(s) is/are correct?


A. Only I B. Only II C. None of the above D. All of the above

Answer: B

The settlement is divided into two sections, one smaller but higher and the other much larger
but lower. Archaeologists designate these as the Citadel and the Lower Town respectively. The
Citadel owes its height to the fact that buildings were constructed on mud brick platforms. It
was walled, which meant that it was physically separated from the Lower Town.

Q10. Which of the following site is currently within Indian Territory?


(A). Mohenjo-daro (B). Koti-diji (C). Alamgirpur (D). Amri

Answer: C

Alamgirpur is in Meerut (UP), Koti-Diji (Sindh, Pakistan), Mohenjo-Daro (Sindh, Pakistan), Amri (Sindh,
Pakistan)

Google it:- https://upscpdf.com


www.achieveias.co.in, YouTube Channel: http://youtube.com/c/AchieveIAS Telegram Channel: http://t.me/Achieve_Ias,
Mail: achieveias21@gmail.com, Contact Number: 8968920720
https://upscpdf.com https://upscpdf.com https://upscpdf.com
ACHIEVE IAS HISTORY MCQ SERIES, DAY 37, SOLUTIONS

Q1. Consider the following statements:


1. The first coins to bear the names and images of rulers were issued by the Mauryans.
2. The first gold coins were issued by the Guptas.

Which of the following is/are correct?


A. Only I B. Only II C. None of the above D. All of the above

Answer: C

The first coins to bear the names and images of rulers were issued by the Indo-Greeks,
who established control over the north-western part of the subcontinent c. second century BCE.
The first gold coins were issued c. first century CE by the Kushanas. These were virtually
identical in weight with those issued by contemporary Roman emperors and the Parthian rulers
of Iran, and have been found from several sites in north India and Central Asia.

Q2. With which kingdom ancient town Rajagaha was associated?


A. Magadha B. Matsya C. Panchala D. Anga

Answer: A

Rajagaha (the Prakrit name for presentday Rajgir in Bihar) was the capital of Magadha.
Interestingly, the old name means “house of the king”. Rajagaha was a fortified settlement,
located amongst hills. Later, in the fourth century BCE, the capital was shifted to Pataliputra,
present-day Patna, commanding routes of communication along
the Ganga.

Q3. Which of the following inscriptions of the time of Kanishka indicates that his empire was
extended up to Champa in the east?
(a) Sarnath inscription (b) Saheth-maheth inscription
(c) Rabatak inscription (d) None of the above

Answer: C

The Rabatak inscription is an inscription written on a rock in the Bactrian language and the
Greek script, which was found in 1993 at the site of Rabatak, near Surkh Kotal in Afghanistan.
The lines 4 to 7 of the inscription describe the cities which were under the rule of Kanishka, among
which six names are identifiable: Ujjain, Kundina, Saketa, Kausambi, Pataliputra, and Champa.

Q4. With reference to ancient India, Which one of the following scripts was written from right to
left?
(a) Kharoshti (b) Devanagari (c) Brahmi (d) Grantha

Answer: A
Google it:- https://upscpdf.com
www.achieveias.co.in, YouTube Channel: http://youtube.com/c/AchieveIAS Telegram Channel: http://t.me/Achieve_Ias,
Mail: achieveias21@gmail.com, Contact Number: 8968920720
https://upscpdf.com https://upscpdf.com https://upscpdf.com
The Kharosthi script is an ancient script used by the ancient Gandhara culture of South Asia primarily
in modern-day Afghanistan and Pakistan. Kharosthi is mostly written right to left but some inscriptions
show the left to right direction that was to become universal for the later South Asian scripts.

Q5. Which of the following Ashoka inscriptions is in Kharosthi script?


(a) Lumbini (b) Khalsi (c) Girnar (d) Shahbazgarhi

Answer: D
The inscriptions of Ashoka are written in two scripts known as Brahmi and Kharosthi. Kharosthi script
has been used in the inscriptions found at Shahbazgarhi in Mardan District of Khyber- Pakhtunkhwa
province of Pakistan and Manshera also in Pakistan. Three languages were used, Prakrit, Greek and
Aramaic. Four scripts were used. The edicts are composed in non-standardized and archaic forms of
Prakrit. Prakrit inscriptions were written in Brahmi and Kharosthi scripts, which even a commoner could
read and understand. The inscriptions found in the area of Pakistan are in kharoshthi script. The rest
are written in Greek or Aramaic. The Kandahar Rock Inscription is bilingual Greek-Aramaic.

Q6. Which of the following is not a major rock edicts?


a) Sannati b) Shahbazgarhi c) Kandahar d) Brahmagiri

Answer: D

The edict at Brahmagiri comprising Minor Rock-edicts is the best preserved edict of Asoka in
Karnataka. It is engraved on a huge boulder of gneiss, at the base of a hill.

Q7. The title of Devanapiya Piyadasi was assumed by?


A. Kanishka B. Chandragupta Maurya C. Ashoka D. Samundragupta

Answer: C

Ashoka, the great ruler of the Maurya Dynasty was called Devanampiya Piyadassi. Ashoka is the
earliest known ruler to have inscribed his messages to his subjects and officials on stone surface natural
rocks as well as polished pillars. When the king Devanampiya Piyadassi had been ruling for eight years,
the (country of the) Kalinga (present day coastal Orissa) was conquered by (him). One hundred and
fifty thousand men were deported, a hundred thousand were killed, and much more died.

Q8. Which of the following was major political centre of Mauryan Empire was important for
tapping gold mines?
(a) Suvarnagiri (b) Ujjayini (c) Tosali (d) Taxila

Answer: A

In Maurya times centres were carefully chosen, both Taxila and Ujjayini being situated on important
long-distance trade routes, while Suvarnagiri (literally, the golden mountain) was possibly important
for tapping the gold mines of Karnataka.

Google it:- https://upscpdf.com


www.achieveias.co.in, YouTube Channel: http://youtube.com/c/AchieveIAS Telegram Channel: http://t.me/Achieve_Ias,
Mail: achieveias21@gmail.com, Contact Number: 8968920720
https://upscpdf.com https://upscpdf.com https://upscpdf.com
Q9. The Rulers of this dynasty claimed to be "son of god". The Dynasty was:
A. Mauryans
B. Kushana
C. Sungar
D. Shakas

Answer: B
Kushanas considered themselves godlike. Many Kushana rulers also adopted the title
devaputra, or “son of god”, possibly inspired by Chinese rulers who called themselves
sons of heaven.

Q10. The first person to decipher Ashokan inscriptions was:


(A). Harace Hayman Wilson (B). James Atkinson
(C). Captain Herbert (D). James Princep

Answer: D

James Princep was an English scholar, orientalist and antiquary. Remained unread until James
Princep successfully deciphered Ashoka's edicts inscription in 1 Stories of Ashoka flourished from the
time of his death and grew more fanciful with the passing centuries.

Google it:- https://upscpdf.com


www.achieveias.co.in, YouTube Channel: http://youtube.com/c/AchieveIAS Telegram Channel: http://t.me/Achieve_Ias,
Mail: achieveias21@gmail.com, Contact Number: 8968920720
https://upscpdf.com https://upscpdf.com https://upscpdf.com
ACHIEVE IAS HISTORY MCQ SERIES, DAY 38, SOLUTIONS

Q1. Consider the following statements with reference to manusmriti:


1. Women can hold their valuables even without their husband's permission.
2. Women's wealth could be inherted by their children or can be claimed by their husband.

Which of the following is/are correct?


A. Only I B. Only II C. None of the above D. All of the above

Answer: C

Women were allowed to retain the gifts they received on the occasion of their marriage as
stridhana (literally, a woman’s wealth). This could be inherited by their children, without the
husband having any claim on it. At the same time, the Manusmriti warned women against
hoarding family property, or even their own valuables, without the husband’s permission.

Q2. Who among the following were regarded as Mlechchhas?


A. Vakatakas B. Shungas
C. Shakas D. Kanvas

Answer: C

Rulers such as the Shakas who came from Central Asia, were regarded as mlechchhas,
barbarians or outsiders by the Brahmanas. However, one of the earliest inscriptions in Sanskrit
describes how Rudradaman, the best-known Shaka ruler (c. second century CE), rebuilt
Sudarshana lake This suggests that powerful mlechchhas were familiar with Sanskritic traditions..

Q3. Inscription of Mandason is mainly about?


(a) King of Dashapura (b) Brahmenes
(c) Varna system (d) Silk weaves

Answer: D

Stone inscription (c. fifth century CE), found in Mandasor (Madhya Pradesh), records the vhistory
of a guild of silk weavers who originally lived in Lata (Gujarat), from where they migrated to
Mandasor, then known as Dashapura. It states that they undertook the difficult journey along
with their children and kinfolk, as they had heard about the greatness of the local king, and
wanted to settle in his kingdom.

Q4. With reference to ancient history of India, what is the meaning of Vanik?
(a) Brahmanas (b) Peasants
(c) Goldsmiths (d) Merchants

Answer: D

Google it:- https://upscpdf.com


www.achieveias.co.in, YouTube Channel: http://youtube.com/c/AchieveIAS Telegram Channel: http://t.me/Achieve_Ias,
Mail: achieveias21@gmail.com, Contact Number: 8968920720
https://upscpdf.com https://upscpdf.com https://upscpdf.com
Sanskrit texts and inscriptions used the term vanik to designate merchants. While trade was defined
as an occupation for Vaishyas in the Shastras, a more complex situation is evident in plays such as the
Mrichchhakatika written by Shudraka (c. fourth century CE), Here, the hero Charudatta was described
as both a Brahmana and a sarthavaha or merchant And a fifth-century inscription describes two
brothers who made a donation for the construction of a temple as kshatriya-vaniks

Q5. Which archaeologists excavated Hastinapura in Meerut (UP)?


(a) B B Lal (b) D P Aggarwal (c) R D Banerjee (d) M C Joshi

Answer: A

The Mahabharata, like any major epic, contains vivid descriptions of battles, forests, palaces
and settlements. In 1951-52, the archaeologist B.B. Lal excavated at a village named Hastinapura
in Meerut (Uttar Pradesh). Was this the Hastinapura of the epic? While the similarity in names
could be coincidental, the location of the site in the Upper Ganga doab, where the Kuru kingdom
was situated, suggests that it may have
been the capital of the Kurus mentioned in the text.

Q6. Who among them is associated with story of Majjhima Nikaya?


a) Anirudh b) Kachchana
c) Mahakasyapa d) Upali

Answer: B

Story based on a Buddhist text in Pali known as the Majjhima Nikaya, is part of a dialogue between a
king named Avantiputta and a disciple of the Buddha named Kachchana. While it may not be literally
true, it reveals Buddhist attitudes towards varna.

Q7. Consider the following statements regarding the critical edition of Mahabharata:
1. A team comprising dozens of scholars initiated the task of preparing a critical edition of the
Mahabharata.
2. Initially, it meant collecting Sanskrit manuscripts of the text, written in a variety of scripts, from
different parts of the country.
3. The team worked out a method of comparing verses from each manuscript.

Which of the following statement(s) is/are correct?


A. Only I B. I and II C. I and III D. All of the above

Answer: D

One of the most ambitious projects of scholarship began in 1919, under the leadership of a noted Indian
Sanskritist, V.S. Sukthankar. A team comprising dozens of scholars initiated the task of preparing a
critical edition of the Mahabharata. What exactly did this involve? Initially, it meant collecting Sanskrit
manuscripts of the text, written in a variety of scripts, from different parts of the country. Ultimately, they
selected the verses that appeared common to most versions and published these in several volumes,
running into over 13,000 pages.
Google it:- https://upscpdf.com
www.achieveias.co.in, YouTube Channel: http://youtube.com/c/AchieveIAS Telegram Channel: http://t.me/Achieve_Ias,
Mail: achieveias21@gmail.com, Contact Number: 8968920720
https://upscpdf.com https://upscpdf.com https://upscpdf.com
Q8. Consider the following statements:
1. Polyandry was prevalent in Himalayan region.
2. System of exogamy existed in ancient India.

Which of the following is/are correct?


A. Only I B. Only II C. None of the above D. All of the above

Answer: A

Some historians note that while the practice of polyandry may have seemed unusual or even
undesirable from the Brahmanical point of view, it was (and is) prevalent in the Himalayan
region. Others suggest that there may have been a shortage of women during times of warfare,
and this led to polyandry. In other words, it was attributed to a situation of crisis. While sons
were important for the continuity of the patrilineage, daughters were viewed rather differently
within this framework. They had no claims to the resources of the household. At the same
time, marrying them into families outside the kin was considered desirable. This system was
called exogamy.

Q9. Consider the following statements:


1. Buddhists rejected the idea of claims to status on the basis of birth.
2. Budhists considered the differences in the society as flexible.

Which of the following is/are not correct?


A. Only I B Only II C. None of the above D. All of the above

Answer: C

Brahmanical view of society was codified in the Dharmasutras and Dharmashastras, other
traditions developed critiques of the varna order. Some of the best-known of these were
developed within early Buddhism (c. sixth century BCE onwards). The Buddhists recognised that
there were differences in society, but did not regard these as natural or inflexible. They also
rejected the idea of claims to status on the basis of birth.

Q10. Consider the following statements regarding the familial ties and practices:
1. Often people belonging to the same family share food and other resources, and live, work and
perform rituals together.
2. Families are usually parts of larger networks of people defined as relatives, or to use a more technical
term, kinfolk.
3. While familial ties are often regarded as “natural” and based on blood, they are defined in many
different ways.

Which of the following statement(s) is/are correct?


A. Only I B. I and II C. I and III D. All of the above

Answer: D
Google it:- https://upscpdf.com
www.achieveias.co.in, YouTube Channel: http://youtube.com/c/AchieveIAS Telegram Channel: http://t.me/Achieve_Ias,
Mail: achieveias21@gmail.com, Contact Number: 8968920720
https://upscpdf.com https://upscpdf.com https://upscpdf.com
We often take family life for granted. However, you may have noticed that not all families are identical:
they vary in terms of numbers of members, their relationship with one another as well as the kinds of
activities they share. Often people belonging to the same family share food and other resources, and
live, work and perform rituals together. Families are usually parts of larger networks of people defined
as relatives, or to use a more technical term, kinfolk. While familial ties are often regarded as “natural”
and based on blood, they are defined in many different ways. For instance, some societies regard
cousins as being blood relations, whereas others do not.

Google it:- https://upscpdf.com


www.achieveias.co.in, YouTube Channel: http://youtube.com/c/AchieveIAS Telegram Channel: http://t.me/Achieve_Ias,
Mail: achieveias21@gmail.com, Contact Number: 8968920720
https://upscpdf.com https://upscpdf.com https://upscpdf.com
ACHIEVE IAS HISTORY MCQ SERIES, DAY 39, SOLUTIONS

Q1. Consider the following statements in context of Bodhisatta:


1. One who attains nirvana through intense efforts.
2. Bodhisattas are not worshipped.

Which of the following is/are correct?


A. Only I B. Only II C. None of the above D. All of the above

Answer: C

The concept of the Bodhisatta also developed. Bodhisattas were perceived as deeply
compassionate beings who accumulated merit through their efforts but used this not to
attain nibbana and thereby abandon the world, but to help others. The worship of
images of the Buddha and Bodhisattas became an important part of this tradition.

Q2. Who among them is a vedic deity ?


A. Dakini B. Soma C. Ambika D. Shiv

Answer: B

The Rigveda consists of hymns in praise of a variety of deities, especially Agni, Indra
and Soma. Many of these hymns were chanted when sacrifices were performed, where
people prayed for cattle, sons, good health, long life, etc.

Q3. In which of the following Buddhist sacred place, Jatakas have been depicted through
paintings?
(a) Dhamek Stupa (b) Ajanta Caves
(c) Bavikonda Stupa (d) Aurangabad Caves

Answer: B

The paintings in the Ajanta caves predominantly narrate the Jataka tales. These are Buddhist legends
describing the previous births of the Buddha. These fables embed ancient morals and cultural lores
that are also found in the fables and legends of Hindu and Jain texts. The Jataka tales are exemplified
through the life example and sacrifices that the Buddha made in hundreds of his past incarnations,
where he is depicted as having been reborn as an animal or human.

Q4. The symbol of wheel represented in Buddhism represents?


(a) Buddha's birth (b) The great departure
(c) Enlightenment (d) First sermon

Answer: D

Symbols associated with life of Buddha - Lotus and elephant: Buddha's birth,
Google it:- https://upscpdf.com
www.achieveias.co.in, YouTube Channel: http://youtube.com/c/AchieveIAS Telegram Channel: http://t.me/Achieve_Ias,
Mail: achieveias21@gmail.com, Contact Number: 8968920720
https://upscpdf.com https://upscpdf.com https://upscpdf.com
Horse: The Great Departure (Mahabhinishkramana),
Bodhi tree (Peepal): Enlightenment,
Wheel: First sermon (Dhamma chakra parivartana),
Stupa (dome-shaped monument in which ash was kept) Death (Maha Parinirvana)

Q5. The Vessantara Jātaka tells the story of one of Gautama Buddha's past lives. In this jataka
he has been mentioned as?
(a) Prince (b) Monk (c) Merchant (d) Peasant

Answer: A

The Vessantara Jātaka is one of the most popular apadānas of Theravada Buddhism. The Vessantara
Jātaka tells the story of one of Gautama Buddha's past lives, about a compassionate prince,
Vessantara, who gives away everything he owns, including his children, thereby displaying the virtue
of perfect generosity. It is also known as the Great Birth Sermon.

Q6. Elliot marbles is a series of 120 sculptures inscription. To which of the following Indian city
they initially belonged?
a) Madras b) Amaravati c) Sanchi d) Sarnath

Answer: B

The Amaravati Collection, sometimes called the Amaravati Marbles, is a series of 120 sculptures and
inscriptions in the British Museum from the Amaravathi Mahachaitya in Amaravathi, Guntur in the Indian
state of Andhra Pradesh. The Amaravati artefacts entered the Museum's collection in the 1880s. The
Amaravati sculptures have also been called the Elliot Marbles on account of their association in with
Sir Walter Elliot in the 1840s.

Q7. Consider the following statements:


1. Birth - Lumbini
2. First sermon - Kusinagar
3. Nirvana - Bodhgaya

In context of Buddhist life which of the following is not correctly matched?


A. 1 and 2 B. 1 and 3 C. 2 and 3 D. None of the above

Answer: C

Buddha's life - where he was born (Lumbini) where he attained enlightenment (Bodh Gaya) where he
gave his first sermon (Sarnath) and where he attained nibbana (Kusinagara). Gradually, each of these
places came to be regarded as sacred.

Q8. The first 'Bhikkhuni' in Buddhism was?


A. Yasodha B. Mahaprajapati Gotami C. Visakha D. Savatthi

Google it:- https://upscpdf.com


www.achieveias.co.in, YouTube Channel: http://youtube.com/c/AchieveIAS Telegram Channel: http://t.me/Achieve_Ias,
Mail: achieveias21@gmail.com, Contact Number: 8968920720
https://upscpdf.com https://upscpdf.com https://upscpdf.com
Answer: B

Mahāpajāpatī Gotamī was the step-mother and maternal aunt (mother's sister) of the Buddha. In
Buddhist tradition, she was the first woman to seek ordination for women, which she did from Gautama
Buddha directly, and she became the first bhikkhuni (Buddhist nun).

Q9. With reference to the history of Indian architecture, consider the following statements:
1. The Barabar rock-cut caves were originally made for Ajivikas by Emperor Chandragupta Maurya.
2. Harmika is a term used for pillars of stupa.

Which of the following is/are correct?


A. Only I B. Only II C. None of the above D. All of the above

Answer: C

The inscription in the Sudama cave informs that the four caves on Barabar hill were assigned by King
Ashoka to Ajivika monks in 261 BC. Another inscription on the Nagarjuni hill is of the grandson of King
Ashoka, Dasaratha Maurya, which tells that the Ajivikas continued to enjoy imperial Mauryan patronage
for long. The harmika is inspired by a square railing or fence that surrounded the mound of dirt, marking
it as a sacred burial site.

Q10. Which of the following language was patronised by the Jainas to preach their doctrines?
a. Prakrit b. Sanskrit c. Dravidian d. Pali

Answer: A

Explanation: Jainism made the first serious attempt to alleviate the evils of the varna older and the
ritualistic Vedic religion The early Jainas discarded the Sanskrit language mainly patronised by the
Brahmanas. They adopted Prakrit language of the common people to preach their doctrines.

Google it:- https://upscpdf.com


www.achieveias.co.in, YouTube Channel: http://youtube.com/c/AchieveIAS Telegram Channel: http://t.me/Achieve_Ias,
Mail: achieveias21@gmail.com, Contact Number: 8968920720
https://upscpdf.com https://upscpdf.com https://upscpdf.com
ACHIEVE IAS HISTORY MCQ SERIES, DAY 40, SOLUTIONS

Q1. Consider the following statements :


1. Al-Biruni - Egypt
2. Ibn Battuta - Morocco
3. Francois Baernies - France

Which of the following is/are correctly matched?


A. 1 and 2 only B. 2 and 3 only C. 1 and 3 only D. All are correct

Answer: B

Only 1st one is incorrect. Al-Biruni came from Uzbekistan (eleventh century), Ibn Battuta came
from Morocco, in northwestern Africa (fourteenth century) and the François Bernier was from
France (seventeenth century).

Q2. Which position was offered to Ibn Battuta in Delhi sultanate?


A. Court poet B. Wazir C. Ulema D. Qazi

Answer: D

The Sultan Muhammad bin Tughlaq was impressed by Battuta's scholarship, and appointed
him the qazi or judge of Delhi.

Q3. Ibn Batutta visited India in the reign of?


(a) Firoz Tughlaq (b) Muhammad bin Tughlaq
(c) Bahlol Lodhi (d) Alauddin Khilji

Answer: B

In 1334, Ibn Battuta arrived in India all the way through the mountains of Afghanistan, during the time
when dynasty of Muhammad bin Tughlaq was at its height. On his approach towards Sultan
Muhammad Tughlaq, he learnt that the great Sultan liked to take gifts from his visitors, and in return
the Sultan, used to give gifts, which are of far superior worth to visitors.

Q4. With respect to Kitab-ul-hind consider the following statements:


1. It was written by Al-Biruni.
2. It mainly describes conquest of India by Mahmud Ghazni.

Which of the following is/are correct?


(a) Only 1 (b) Only 2 (c) Both (d) None

Answer: A

Google it:- https://upscpdf.com


www.achieveias.co.in, YouTube Channel: http://youtube.com/c/AchieveIAS Telegram Channel: http://t.me/Achieve_Ias,
Mail: achieveias21@gmail.com, Contact Number: 8968920720
https://upscpdf.com https://upscpdf.com https://upscpdf.com
Al-Biruni’s Kitab-ul-Hind, written in Arabic, is simple and lucid. It is a voluminous text, divided
into 80 chapters. It mainly describes subjects such as religion and philosophy, festivals, astronomy,
alchemy, manners and customs, social life, weights and measures, iconography, laws and
metrology.

Q5. Which Mughal prince or emperor was closely associated with Francois Bernier?
(a) Jahan Shah (b) Dara Shikoh
(c) Shah Jahan (d) Aurangzeb

Answer is B

François Bernier, a Frenchman, was a doctor, political philosopher and historian. Like many
others, he came to the Mughal Empire in search of opportunities. He was in India for twelve
years, from 1656 to 1668, and was closely associated with the Mughal court, as a physician to
Prince Dara Shukoh, the eldest son of Emperor Shah Jahan, and later as an intellectual and
scientist, with Danishmand Khan, an Armenian noble at the Mughal court.

Q6. In context of Medieval Period who among them is an Indian traveller?


a) Abd-Al-Razzaq Samarqandi b) Nasir Khusraw
c) Muhammad-Al-Idrisi d) Mirza Abu Taleb

Answer: D

Mirza Abu Taleb grew up in Lucknow, where his father had served the nawab of Awadh. By the time
he came of age, the nawab’s power had been eclipsed by that of the British, who installed a Resident
at Lucknow in 1773. At the dawn of the 19th century, Mirza Abu Taleb Khan, the Indian voyager, offered
his own account of Paris.

Q7. According to Francois Bernier, what was the reason behind uniform ruination of
agriculture?
A. Lack of private landownership B. Bad climatic condition
C. Less innovation in agriculture. D. Absence of warehouse system

Answer: A

According to Bernier, one of the fundamental differences between Mughal India and Europe
was the lack of private property in land in the former. He was a firm believer in the virtues of
private property, and saw crown ownership of land as being harmful for both the state and its
people. The absence of private property in land had, therefore, prevented the emergence of the
class of “improving” landlords (as in Western Europe) with a concern to maintain or improve
the land. It had led to the uniform ruination of agriculture, excessive oppression of the peasantry
and a continuous decline in the living standards of all sections of society, except the ruling
aristocracy.

Google it:- https://upscpdf.com


www.achieveias.co.in, YouTube Channel: http://youtube.com/c/AchieveIAS Telegram Channel: http://t.me/Achieve_Ias,
Mail: achieveias21@gmail.com, Contact Number: 8968920720
https://upscpdf.com https://upscpdf.com https://upscpdf.com
Q8. Which historians provide detailed account of working of the Imperial workshop in Mughal
India?
A. Francois Bernier B. Al-Biruni C. Ibn Battuta D. Marco polo

Answer: A

Bernier is perhaps the only historian who provides a detailed account of the working of the imperial
karkhanas or workshops: Large halls are seen at many places, called karkhanas or workshops for the
artisans. In one hall, embroiderers are busily employed, superintended by a master. In another, you
see the goldsmiths; in a third, painters; in a fourth, varnishers in lacquer-work; in a fifth, joiners, turners,
tailors and shoe-makers; in a sixth, manufacturers of silk, brocade and fine muslins.

Q9. The merchant community in western India was known as :


A. Dola B. Wakil C. Mahajans D. Sheth.

Answer: C

Merchants had strong communities, organized into their own caste-cum occupational groups, which
were known as mahajans in western India.

Google it:- https://upscpdf.com

You might also like